Chest Pain (2024)

by Khaled Alaboud Alkheder & Muneer Al Marzooqi

You Have A New Patient!

A 67-year-old woman presents to the ED with acute chest pain. The pain is sharp and stabbing in nature. She feels nauseated and short of breath. The patient has a history of hypertension, type 1 diabetes mellitus, medullary thyroid cancer, coronary artery disease, and gastroesophageal reflux disease. She smoked half a pack of cigarettes daily for 19 years but quit 18 years ago. Her current medications include Lisinopril, Insulin Glargine, Insulin Aspart, Sertraline, Aspirin, and Ranitidine.

The image was produced by using ideogram 2.0.

She appears anxious and diaphoretic. Her temperature is 37.2°C, pulse is 62/min, respirations are 19/min, and blood pressure is 142/81 mmHg. The lungs are clear to auscultation. The chest wall and abdomen are non-tender. There is 5/5 strength in all extremities. The remainder of the examination shows no abnormalities.

How would you proceed, and what is the next step in management?

What Do You Need To Know?

Chest pain in the emergency department is reported to be the second most common complaint, comprising approximately 5% of all emergency department visits. It can indicate various underlying causes, and patients present with many signs and symptoms. The potential causes of chest pain include diseases affecting the heart, aorta, lungs, esophagus, stomach, mediastinum, pleura, and abdominal viscera.

Patients usually describe visceral pain as a squeezing, pressure-like, or dull type of pain. If the pain is visceral, it may also refer to other locations due to the nerves coursing through somatic nerve fibers as they reach the spinal cord. For example, ischemic heart pain may refer to the left or right shoulder, jaw, or left arm.

Clinicians in the ED focus on promptly identifying and ruling out life-threatening causes of chest pain. Patients with serious causes of chest pain may not exhibit any vital sign or physical examination abnormalities and may appear healthy [1,2].

Initial Assessment and Stabilization (ABCDE Approach)

The ABCDE approach is universally recognized as the safest and most efficient method for the initial assessment of patients in the Emergency Department (ED), particularly those presenting with chest pain [3]. This systematic approach ensures rapid identification and management of life-threatening conditions. It prioritizes the immediate stabilization of the patient while facilitating a structured evaluation process.

A – Airway: The first step involves assessing the airway for any signs of obstruction. Key indicators include the patient’s ability to speak without distress and the presence of paradoxical chest movements. Obstructions may result from conditions such as tongue swelling, lip swelling, or other factors impeding spontaneous breathing. Ensuring a patent airway is critical, as it serves as the foundation for effective oxygenation and ventilation.

B – Breathing: Next, the breathing assessment evaluates respiratory effectiveness by observing the patient’s respiratory rate (normal range: 10-20 breaths per minute), inspecting for signs of respiratory distress, and auscultating lung sounds. Findings such as basal crackles may indicate pulmonary edema, diminished breath sounds could suggest pneumothorax or pleural effusion. Each of these conditions requires prompt recognition and intervention.

C – Circulation: The circulation step focuses on identifying signs of cardiovascular compromise or shock. Clinical signs include abnormal extremity coloration (blue, pale, pink, or mottled), prolonged capillary refill time (normal is ≤2 seconds), and abnormal heart rates. Auscultation of the heart should confirm normal S1 and S2 sounds without murmurs or gallops. These findings guide the clinician in diagnosing conditions such as hypoperfusion or cardiac dysfunction. Muffled heart sounds may point toward pericardial tamponade. 

D – Disability: Assessment of the patient’s neurological status is crucial, including evaluating their level of alertness, Glasgow Coma Scale (GCS) score, and glucose levels. Any abnormalities here could indicate underlying conditions such as hypoglycemia, traumatic brain injury, or other causes of altered mental status.

E – Exposure: The final step involves fully exposing the patient to detect visible signs such as rashes, discoloration, or gross abnormalities. This step ensures that no critical findings, such as trauma or skin infections, are overlooked.

Once the primary assessment is complete, interventions should focus on managing hemodynamic instability, such as shock or hypertension. Simultaneously, secondary assessments and investigations are initiated, including obtaining IV access, performing a 12-lead ECG, and ordering relevant diagnostic tests to confirm the underlying cause of the presentation.

Medical History

When assessing a patient presenting with chest pain in the Emergency Department (ED), obtaining a thorough history is critical after ensuring the patient’s stability. Key aspects of the history should include [3,4]:

  • Onset of Pain: Determining whether the pain started abruptly or developed gradually provides valuable diagnostic clues.
  • Site of Pain: The location of the pain (e.g., substernal, localized, diffuse, chest wall, or back) can guide the identification of the underlying cause.
  • Character of Pain: Descriptions such as sharp, squeezing, or pleuritic help differentiate between cardiac, pulmonary, and musculoskeletal etiologies.
  • Radiation: Pain radiating to areas like the jaw, back, shoulder, or arm can indicate cardiac involvement.
  • Associated Symptoms: Symptoms such as diaphoresis, palpitations, dyspnea, nausea, or vomiting are important to document.
  • Timing: The pattern of the pain, whether constant or episodic, its duration, and the time of onset can help in distinguishing between various causes.
  • Exacerbating/Relieving Factors: Identifying activities or factors that provoke or alleviate the pain aids in narrowing down the diagnosis.

Pain Descriptions and Differential Diagnosis: The nature of the chest pain provides critical diagnostic insights:

  • Cardiac Origin: Pain described as “squeezing,” “crushing,” or “pressure-like” suggests cardiac ischemia or acute coronary syndrome (ACS). Pain during exertion is typical of stable angina, whereas progressive pain at rest suggests unstable angina or myocardial infarction (MI).
  • Aortic Dissection: “Tearing” pain radiating to the back is a hallmark of aortic dissection.
  • Pulmonary or Musculoskeletal Causes: “Sharp” or “stabbing” pain is often associated with pulmonary embolism, pneumothorax, or musculoskeletal disorders.
  • Gastrointestinal Causes: “Burning” or “indigestion-like” pain may originate from the gastrointestinal tract but could also signify visceral chest pain. Pain triggered by meals is more likely gastrointestinal in origin.
  • Acute Conditions: Sudden onset pain suggests conditions like aortic dissection, pulmonary embolism, or pneumothorax.

Medical Background and Risk Factors: A comprehensive medical history is essential to assess the risk for specific conditions:

  • Risk Factors for Acute Coronary Syndrome (ACS):
    • Male sex
    • Age over 55 years
    • Family history of coronary artery disease
    • Diabetes mellitus
    • Hypercholesterolemia
    • Hypertension
    • Tobacco use
  • Risk Factors for Pulmonary Embolism: Patients are at an increased risk if they have:
    • Prolonged immobilization (e.g., long-distance travel)
    • Recent surgery, especially orthopedic procedures lasting over 30 minutes
    • Central venous catheterization
    • Trauma
    • Pregnancy
    • Cancer
    • Lung or chronic heart disease
    • A personal or family history of hypercoagulability
    • Use of hormonal contraceptives or chemotherapeutic agents that increase estrogen and progestin levels

This detailed and systematic approach to history-taking allows for accurate and timely diagnosis, ensuring that critical conditions are addressed without delay.

Physical Examination

After obtaining a detailed history, a focused physical examination is crucial to identify any signs that may guide the clinician toward an accurate diagnosis. This examination combines general and systemic assessments, prioritizing findings that can point to life-threatening conditions [5,6].

General Examination and Vital Signs:

The initial step involves assessing vital signs, which often provide significant diagnostic clues:

  • Hypotension may indicate conditions such as tension pneumothorax, pulmonary embolism (PE), or acute myocardial infarction (MI).
  • Tachycardia is a nonspecific finding but is frequently seen in acute MI, PE, aortic dissection, or tension pneumothorax.
  • Hypoxemia suggests pulmonary conditions such as PE, tension pneumothorax, or simple pneumothorax.
  • Fever can be indicative of inflammatory or infectious processes, including PE, pericarditis, myocarditis, or even extrapulmonary causes like cholecystitis.

Cardiovascular Examination:

A detailed cardiovascular assessment should focus on specific findings that may narrow the differential diagnosis:

  • Significant blood pressure differences between upper extremities are a hallmark of aortic dissection.
  • Pericardial rub is a characteristic sign of pericarditis.
  • Jugular venous distension (JVD) may indicate tension pneumothorax, PE, or pericarditis with effusion.
  • Narrow pulse pressure can be associated with pericarditis with effusion, reflecting compromised cardiac output.
  • Pulsus paradoxus, an exaggerated drop in systolic blood pressure during inspiration, is a critical finding in cardiac tamponade and constrictive pericarditis.

Pulmonary Examination:

The pulmonary evaluation should focus on auscultation and observation:

  • Unilateral diminished or absent breath sounds point to tension pneumothorax or simple pneumothorax.
  • Pleural rub, a coarse grating sound, may be heard in PE, indicating pleural irritation.
  • Basal crackles (rales), particularly when bilateral, are often associated with acute MI or pulmonary edema, reflecting fluid overload or cardiac dysfunction.

Integration of Findings:

These physical examination findings must be interpreted in the context of the patient’s history and associated risk factors. For example:

  • A patient presenting with hypoxemia, tachycardia, and JVD warrants an immediate evaluation for PE.
  • Tension pneumothorax should be suspected in cases with hypotension, unilateral absent breath sounds, and JVD.
  • Signs of basal crackles and a pericardial rub may point to a combination of acute MI and pericarditis, necessitating rapid interventions.

By systematically combining history with these focused examination findings, clinicians can efficiently narrow their differential diagnosis and prioritize further investigations and treatments. This structured approach ensures that life-threatening conditions are promptly identified and managed.

When To Ask for Senior Help

Remember that senior residents and attendings supervise you when working in the emergency department. It is important to ask for their help when needed, especially when a patient with chest pain arrives [6]. The following are situations when you need to call for help immediately in a patient with chest pain:

  • Patients clenching their chest with ongoing chest pain and diaphoresis.
  • Chest pain with severe shortness of breath and evidence of pulmonary edema.
  • Chest pain with hypotension.
  • Chest pain with severe bradycardia or tachycardia.
  • Chest pain followed by unresponsiveness.

These examples exhibit life-threatening features of chest pain that can be lethal within minutes. You must call for help, and the team will be assembled to care for the patient and administer lifesaving interventions.

Alternative Diagnoses

Chest pain is a common presentation in the Emergency Department (ED) and requires a systematic and thorough approach to rule out life-threatening conditions. These diagnoses must be prioritized in the differential diagnosis as they carry significant morbidity and mortality if not identified and managed promptly [1,6].

Life-Threatening Diagnoses:

  1. Acute Coronary Syndrome (ACS): ACS encompasses conditions such as unstable angina, non-ST elevation myocardial infarction (NSTEMI), and ST elevation myocardial infarction (STEMI). These result from ischemia due to decreased myocardial oxygen supply, often caused by atherosclerotic plaque rupture. Rapid identification through ECG and biomarkers is critical to initiate timely treatment.

  2. Acute Aortic Dissection: This condition arises when a tear in the intimal layer of the aorta allows blood to flow between the layers, creating a false lumen. Patients often present with severe, tearing chest or back pain and may have a significant difference in blood pressure between the upper extremities. Early diagnosis via imaging such as CT angiography is essential to prevent fatal rupture.

  3. Pulmonary Embolism (PE): PE results from the occlusion of pulmonary arteries by thromboemboli, often originating from deep vein thrombosis (DVT). Symptoms include sudden onset dyspnea, chest pain, and hypoxemia. Clinical suspicion should be high in patients with risk factors like prolonged immobilization, recent surgery, or hypercoagulable states.

  4. Tension Pneumothorax: This is a critical condition where air accumulates in the pleural space under pressure, compressing the lungs and mediastinum. Patients may present with hypotension, respiratory distress, and absent breath sounds on the affected side. Immediate needle decompression is lifesaving.

  5. Pericardial Tamponade: This occurs when fluid accumulates in the pericardial sac, impairing cardiac filling and output. Classic findings include hypotension, jugular venous distension, and muffled heart sounds (Beck’s triad). Pulsus paradoxus is another critical clue. Echocardiography confirms the diagnosis, and pericardiocentesis is the treatment.

  6. Esophageal Rupture with Mediastinitis: Esophageal rupture, also known as Boerhaave syndrome, can lead to mediastinitis due to leakage of gastric contents into the mediastinum. Patients typically present with severe chest pain following vomiting, subcutaneous emphysema, and signs of sepsis. Prompt surgical intervention is required.

Other Diagnoses to Consider:

  1. Simple Pneumothorax: Unlike tension pneumothorax, simple pneumothorax lacks hemodynamic compromise but still requires prompt recognition. Patients may present with pleuritic chest pain and diminished breath sounds on the affected side. Treatment typically involves observation or chest tube placement, depending on severity.

  2. Pericarditis: This inflammatory condition of the pericardium often presents with sharp, pleuritic chest pain that is relieved by sitting up and leaning forward. A pericardial rub is the hallmark auscultatory finding. ECG changes, including diffuse ST elevation, aid in the diagnosis. Most cases are viral and self-limiting, though complications like effusion and tamponade must be monitored.

Acing Diagnostic Testing

To accurately diagnose the cause of chest pain, a combination of bedside tests and advanced investigations are essential. These tests provide critical information that can guide immediate management, particularly in identifying life-threatening conditions [1,2].

Bedside Tests

Electrocardiogram (ECG):

The 12-lead ECG is a cornerstone of chest pain evaluation and must be performed within 10 minutes of the patient’s presentation or EMS arrival. It aids in identifying acute coronary syndromes (ACS), including ST-elevation myocardial infarction (STEMI).

STEMI Criteria:
  • General Criteria: At least 1 mm of ST elevation in two contiguous leads, excluding V2 and V3.
  • Specific Criteria for V2 and V3 ST Elevation:
    • Women: ≥1.5 mm elevation.
    • Men <40 years: ≥2.5 mm elevation.
    • Men ≥40 years: ≥2 mm elevation.
Source: Hernandez JM, Glembocki MM, McCoy MA. Increasing Nursing Knowledge of ST-Elevated Myocardial Infarction Recognition on 12-Lead Electrocardiograms to Improve Patient Outcomes. The Journal of Continuing Education in Nursing. 2019;50(10):475-480. doi:10.3928/00220124-20190917-10
Inferior ST segment elevations with anterior and lateral reciprocal changes. Inferior MI, so the right side of the heart should be evaluated with right side chest leads. V2 ST depression is very prominent, therefore, posterior leads should be applied form V7 to V12 for the left side.
43 years-old patients with left sided chest pain. Courtesy of Khaled Alaboud Alkheder and Muneer Al Marzooqi
Clinical Interpretation of the ECG above:
  • For instance, an ECG from a 43-year-old male presenting with severe left-sided chest pain showed ST elevation in anteroseptal leads (V1-V4) with J point elevation >2 mm and reciprocal ST depression in inferior leads, indicative of an acute anterior STEMI. This finding underscores the importance of identifying patterns such as J point elevation, which marks the transition between the QRS complex and the ST segment.

ECG Limitations and Additional Considerations:

  • While some patients exhibit a classic STEMI pattern, many may present with a normal or non-diagnostic ECG. A normal ECG at admission cannot rule out ACS or other conditions, necessitating further testing if clinical suspicion remains high.
  • If the initial ECG is inconclusive, it should be repeated after a 10-minute interval, especially if chest pain recurs.
  • Additional leads should be utilized when clinical suspicion exists for specific myocardial infarctions:
    • Posterior leads (V7-V9): For suspected posterior MI.
    • Right-sided leads (V3R and V4R): For patients with acute inferior MI, to assess for right ventricular involvement.
  • In suspected pulmonary embolism (PE), the S1Q3T3 pattern (prominent S wave in lead I, Q wave in lead III, and inverted T wave in lead III) may suggest right heart strain, though it is neither sensitive nor specific for PE [5].
S1Q3T3 - Courtesy of Khaled Alaboud Alkheder and Muneer Al Marzooqi
ECG 54-yo male chest pain for the last 3 days. S1 Q3 T3, Tachycardia, minor ST depressions on lateral leads (V5-6)

The ECG is a highly valuable tool for ruling in STEMI or other acute conditions. However, its limitations in ruling out conditions underscore the necessity of adjunct investigations and clinical correlation. For example, repeated ECGs, additional lead placements, and further imaging or lab tests (such as cardiac biomarkers or D-dimer for PE suspicion) ensure comprehensive evaluation and timely intervention.

By systematically incorporating these investigative steps into the diagnostic process, clinicians can optimize patient outcomes and address the underlying etiology of chest pain effectively.

Laboratory Tests

In the assessment of patients presenting with chest pain, laboratory investigations play a crucial role in diagnosing life-threatening conditions such as acute myocardial infarction (AMI) and pulmonary embolism (PE). Among the most valuable tests are cardiac troponins and D-dimer levels, each serving distinct purposes based on clinical suspicion and patient presentation.

Cardiac Troponins:

  • Utility in AMI Diagnosis:
    Cardiac troponins, specifically high-sensitivity troponin I and T, are the preferred laboratory markers for diagnosing AMI. These biomarkers can reliably detect myocardial injury within 3 hours of symptom onset. Their high sensitivity and specificity make them the gold standard in confirming myocardial infarction (MI).

  • Role in Ruling Out MI:
    While cardiac troponins are essential for diagnosing AMI, a single set of negative cardiac enzyme results is insufficient to rule out MI, especially in early presentations. However, in patients presenting with chest pain lasting over 2 hours, a single undetectable troponin T level can help exclude MI in certain cases [1].

  • Detection of Unstable Angina:
    High-sensitivity troponin assays can also detect subtle elevations associated with unstable angina, aiding in the identification of patients at risk for adverse cardiac events. However, serial testing may be required to observe trends and confirm the diagnosis.

D-Dimer:

  • Screening for Pulmonary Embolism (PE):
    D-dimer testing is particularly valuable in patients with suspected PE. In low-risk patients, a negative D-dimer test effectively rules out PE, eliminating the need for further imaging.

  • High-Risk Patients:
    Patients identified as high-risk based on clinical assessment or pretest probability should proceed directly to diagnostic imaging, such as computed tomography pulmonary angiography (CTPA). Similarly, patients with an intermediate or high pretest probability should not rely solely on D-dimer results but instead undergo confirmatory imaging [5].

These laboratory investigations provide critical insights when integrated with clinical findings and other diagnostic tools. For example:

  • In patients presenting with prolonged chest pain and an elevated troponin level, AMI is highly likely, warranting immediate intervention.
  • Conversely, in patients with a low-risk Wells score for PE and a negative D-dimer, further imaging can be safely avoided, reducing unnecessary radiation exposure and costs.

Imaging

In the assessment of chest pain, imaging plays a pivotal role in identifying life-threatening conditions and narrowing the differential diagnosis. A combination of imaging techniques can provide vital insights into both cardiac and non-cardiac causes of chest pain.

Chest X-Ray
  • Role in Emergency Evaluations:
    Chest X-rays are widely used in emergency departments as an initial imaging modality. They are particularly useful for identifying acute and life-threatening conditions, including pericardial effusion, acute aortic dissection, pulmonary embolism (PE), pneumothorax, and pneumonia.

    • Timeliness: In cases of high clinical suspicion, a chest X-ray should be performed and interpreted within 30 minutes to avoid delays in diagnosis and treatment.

  • Limitations:
    While chest X-rays are a valuable starting point, their sensitivity and specificity may be limited for certain conditions, necessitating further imaging in many cases.

Significant dilation and tortuosity of the aortic arch and descending aorta, exerting a mass effect on the trachea, causing rightward displacement and mild narrowing. Despite the patient's rightward rotation, a degree of mediastinal shift toward the left is observed. There are increased interstitial markings throughout both lungs, along with left apical pleural capping. - Source: Hacking C Large thoracic aortic aneurysm. Case study, Radiopaedia.org (Accessed on 31 Dec 2024) https://doi.org/10.53347/rID-73356
Pneumothorax on the left side (courtesy of Mohd Mokhtar and Raja Ahmad)
Ultrasonography
  • Advantages of POCUS:
    Point-of-care ultrasound has become an indispensable tool in emergency settings due to its rapid and dynamic assessment capabilities. It can evaluate both cardiac and non-cardiac causes of chest pain with high accuracy.

  • Cardiac Applications:

    • Detection of pericardial effusion and cardiac tamponade is a primary use of POCUS.

    • Example: A significant pericardial effusion may appear as a fluid collection around the heart, as visualized in Figure 5.

  • Pulmonary Applications:

    • POCUS has a higher sensitivity and specificity than chest X-rays for detecting pleural effusion and pneumothorax.

    • Pneumothorax Findings: The absence of the seashore sign (lung sliding) and the presence of the barcode sign on M-mode ultrasound strongly suggest pneumothorax.

    • Acute Heart Failure Findings: In cases of acute ischemic chest pain, lung B-lines detected on ultrasound indicate pulmonary edema due to heart failure.

Subxiphoid 4 Chambers View. PE = Pericardial Effusion, RV = Right Ventricle, LV = Left Ventricle
CT Pulmonary Angiography (CTPA)
  • Gold Standard for PE Diagnosis:
    CT pulmonary angiography (CTPA) is the imaging modality of choice for diagnosing acute pulmonary embolism (PE). Its high sensitivity and specificity make it invaluable for confirming or excluding PE in patients with high clinical suspicion.

  • Additional Findings:
    Beyond diagnosing PE, CTPA can reveal other significant pathologies, including [3,5]:

    • Pneumonia
    • Pericardial abnormalities
    • Musculoskeletal injuries
Pulmonary Embolism - Bilateral thrombus in main pulmonary arteries

Management

Patients presenting with typical chest pain are at a high risk of having Acute Coronary Syndrome. Empiric and symptomatic treatment is paramount in the ED to help control the situation and alleviate the patient’s pain. A common mnemonic used is (MONA), where patients can be given Morphine, which is an opiate, to help relieve the pain. Oxygen supplementation is recommended, but studies have shown that hyperoxygenation and hyperoxia are harmful and can lead to oxygen radicals; therefore, patients are maintained with oxygen saturation between 94–96% [2,6].

As a sublingual administration, Nitroglycerin is used to overcome coronary vasospasm and helps with vasodilation of the coronary vessels to improve blood flow to the myocardium and relieve ischemic chest pain. Finally, Aspirin, as an antiplatelet agent, is used empirically to prevent further clot formation and is one of the mainstay treatments when Acute Coronary Syndrome is suspected.

Aspirin

Dose: 162 to 325 mg in cases of acute coronary syndrome (ACS).
Frequency: Single dose.
Maximum Dose: 4 grams in 24 hours.
Category in Pregnancy: Category C.
Cautions/Comments: Prior to administration, check for allergies, bleeding disorders, or a history of bleeding gastrointestinal (GI) ulcers, as these conditions contraindicate the use of aspirin.

Nitroglycerin (Sublingual or Puffs)

Dose: For sublingual tablets, 0.4 mg per dose. For metered spray, 400 mcg of nitroglycerin per puff.
Frequency: For sublingual administration, up to 3 doses; for puffs, administer every 5 minutes with no more than 3 sprays in a 15-minute period.
Maximum Dose: Up to 3 doses (sublingual) or sprays (puffs) within a 15-minute period.
Category in Pregnancy: Category C.
Cautions/Comments: Nitroglycerin may cause hypotension, particularly with an upright posture. It is contraindicated in patients using phosphodiesterase inhibitors (e.g., for erectile dysfunction).

Morphine

Dose: 4 to 10 mg.
Frequency: Administer 2.5 to 5 mg every 3-4 hours as needed (PRN) or infused over 4-5 minutes.
Maximum Dose: 0.1 to 0.2 mg/kg.
Category in Pregnancy: Classified as Category CFR (consult further resources for more information).
Cautions/Comments: Monitor patients for respiratory depression. Co-ingestion with alcohol increases the risk of a fatal overdose and should be avoided.

Special Patient Groups

Pediatrics

Chest pain in children presenting to the emergency department can be a challenging clinical scenario, as it often raises concerns about serious underlying conditions, including cardiac issues, although they are relatively rare in this population. The differential diagnosis for pediatric chest pain includes musculoskeletal pain, respiratory conditions, gastrointestinal issues, and, less commonly, cardiac abnormalities such as myocarditis or pericarditis [7]. A thorough history and physical examination are essential to differentiate between these causes, considering factors such as the nature of the pain, associated symptoms, and the child’s medical history [8]. While most cases of chest pain in children are benign, it is crucial for healthcare providers to maintain a high index of suspicion and to utilize appropriate diagnostic tools, such as electrocardiograms and imaging studies, when indicated [9].

Pregnant Patients

Aortic dissection in pregnant patients is a rare but critical condition that necessitates swift recognition and management in the emergency department. Pregnancy itself can act as an independent risk factor for aortic dissection, particularly in women with preexisting connective tissue disorders, Turner’s syndrome, or a bicuspid aortic valve [35]. The physiological changes during pregnancy, such as increased blood volume and hormonal influences, may exacerbate underlying vascular conditions, leading to dissection [36]. Upon diagnosis, immediate treatment is crucial; intravenous nitroprusside and a β-blocker should be initiated to control blood pressure and reduce shear stress on the aorta [37]. Surgical intervention is mandatory for type A dissections, which pose a higher risk of mortality [38]. Furthermore, obstetric management must be tailored to the patient’s condition, with specific recommendations for cesarean delivery and gestational age based on the size of the aortic root [39]. Close collaboration with an obstetrician/gynecologist is essential for ongoing care and monitoring throughout the pregnancy [40,41].

Geriatrics

Older adults often experience less classic symptoms of myocardial infarction, such as chest pressure or pain, and may instead report vague symptoms like fatigue, shortness of breath, or confusion, which can complicate diagnosis [14]. Additionally, the presence of multiple chronic conditions may lead to an increased risk of complications and poorer outcomes [15]. Timely and accurate assessment is critical, as delays in diagnosis can significantly impact morbidity and mortality rates in this population [16]. Therefore, a high index of suspicion and thorough evaluation, including appropriate imaging and laboratory tests, are essential in managing chest pain in geriatric patients effectively [17].

When To Admit This Patient

Disposition decisions for patients presenting with chest pain in the emergency department (ED) are critical for ensuring appropriate care and minimizing the risk of adverse cardiovascular events. According to guidelines established by the American College of Cardiology and the American Heart Association (ACC/AHA), patients exhibiting high-risk features, such as ST-segment elevation on an electrocardiogram (ECG), hemodynamic instability, or signs of heart failure, should generally be admitted to the hospital for further evaluation and management [18]. Additionally, those presenting with intermediate-risk features—such as abnormal ECG readings, elevated cardiac biomarkers like troponin, or a history of coronary artery disease—also warrant hospitalization [19]. Conversely, low-risk patients, characterized by a normal ECG and negative cardiac biomarkers, may be safely discharged based on clinical judgment and validated risk stratification tools [19]. Ultimately, the decision to admit a patient with chest pain hinges on a comprehensive assessment of their symptoms, medical history, and individual risk factors for serious cardiovascular events, ensuring that high-risk patients receive the necessary care while minimizing unnecessary hospitalizations for those at lower risk [20].

Risk Stratification

The HEART Score is a clinical tool used to evaluate the risk of major adverse cardiac events (MACE) in patients presenting with chest pain. It assesses five key components: history, ECG findings, age, risk factors, and troponin levels, with each category assigned a score ranging from 0 to 2 points. The total score determines the level of risk and guides subsequent management.

History is assessed based on clinical suspicion. A highly suspicious history earns 2 points, a moderately suspicious history scores 1 point, and a slightly or non-suspicious history scores 0 points. This subjective component emphasizes the importance of a thorough clinical evaluation.

ECG findings are evaluated next. Significant ST-depression earns 2 points, nonspecific repolarization changes score 1 point, and a normal ECG scores 0 points. This category highlights the significance of electrocardiographic abnormalities in cardiac risk stratification.

Age is another important factor. Patients aged 65 years or older receive 2 points, those aged between 45 and 65 years earn 1 point, and patients 45 years or younger score 0 points, reflecting the age-related risk of cardiac events.

Risk factors are categorized based on their number and severity. Patients with three or more risk factors or a history of coronary artery disease (CAD) receive 2 points. Those with one or two risk factors score 1 point, while individuals with no risk factors score 0 points. Risk factors include diabetes mellitus (DM), hypertension (HTN), hyperlipidemia (HLP), smoking (current or recent), obesity, and a family history of CAD.

Troponin levels are also considered. Levels three or more times the normal limit score 2 points, levels one to three times the normal limit earn 1 point, and normal troponin levels score 0 points. This biomarker is critical in identifying myocardial injury.

The total HEART Score helps categorize patients into low, moderate, or high risk for MACE over the next six weeks. A score of 0-3 corresponds to a 2.5% risk and suggests discharge home. A score of 4-6 indicates a 20.3% risk, warranting clinical observation. Scores of 7-10 reflect a 72.7% risk, prompting early invasive strategies. This systematic approach helps clinicians make evidence-based decisions for managing patients with chest pain.

Each variable is scored from 0 to 2, allowing for a comprehensive assessment of the patient’s risk profile. For instance, the patient’s history is examined for indicators of coronary artery disease (CAD), while the ECG is scrutinized for signs of ischemia, such as ST-segment depression [21]. Age is considered a significant risk factor, as older patients are at higher risk for CAD, and the presence of additional risk factors like hypertension, hyperlipidemia, smoking, and diabetes further elevates this risk [22]. Elevated troponin levels serve as a critical marker for myocardial ischemia or infarction. The total HEART score, ranging from 0 to 10, categorizes patients into different risk levels, guiding management decisions regarding further testing, hospitalization, or early discharge [23]. However, it is essential to use the HEART score in conjunction with clinical judgment, as it should not be the sole determinant in decision-making processes [24].

Revisiting Your Patient

The patient had presented with complaints of chest pain, shortness of breath, diaphoresis, and nausea, raising the suspicion of Acute Coronary Syndrome and possible Myocardial Infarction. This suspicion had been supported by her significant risk factors, which included insulin-dependent diabetes mellitus, hypertension, a 12-pack-year smoking history, and a history of ischemic heart disease.

Initial stabilization measures had been promptly undertaken. The patient had been placed in a monitored bed and connected to a cardiac monitor. The ABCDE approach had been followed, and it had been noted that she was vitally stable. A quick history had been obtained, which revealed a sudden onset of central chest pain, described as sharp and stabbing, accompanied by diaphoresis and nausea. On physical examination, equal air entry had been observed with no wheeze or crackles on chest auscultation. A cardiovascular examination had also been planned.

Based on the initial presentation and clinical findings, a cardiac workup had been deemed necessary. This included ordering Troponin T and I tests, performing a 12-lead ECG, and obtaining a portable chest X-ray to rule out potential complications such as congestive heart failure, pneumonia, or pneumothorax.

Therapeutic interventions had been initiated promptly. The patient had been started on supplemental oxygen via a nasal cannula or face mask. Analgesics had been administered while ensuring no contraindications or allergies were present. These included IV paracetamol, IV opioids such as morphine or fentanyl, and sublingual nitroglycerin, either as a puff or tablet. These measures had been aimed at relieving the patient’s symptoms and stabilizing her condition.

Authors

Picture of Khaled Alaboud Alkheder

Khaled Alaboud Alkheder

Tawam Hospital Emergency Medicine Residency Program, United Arab Emirates

Picture of Muneer Abdulla Al Marzooqi

Muneer Abdulla Al Marzooqi

Dr. Muneer is a Consultant Emergency Medicine Physician from the UAE. He completed his EM residency at Tawam Hospital in 2017 and has served as an attending physician and educator there since. He is the Program Director of the Emergency Medicine Residency Program at Tawam Hospital, focusing on medical education, peer development, EM Resuscitation, Simulation, and POCUS. Dr. Muneer has organized and lectured at various seminars and workshops in the MENA region for medical students, residents, and healthcare professionals, including Basic Ultrasound, POCUS, Airway, Suturing, ENT Emergencies Workshops, and the Chief Resident Leadership Program.

Listen to the chapter

References

  1. Stepinska J, Lettino M, Ahrens I, et al. Diagnosis and risk stratification of chest pain patients in the emergency department: focus on acute coronary syndromes. Eur Heart J Acute Cardiovasc Care. 2020;9(1):76-89. doi:10.1177/2048872619885346.
  2. Hollander JE, Chase M. Evaluation of the adult with chest pain in the emergency department. In: Post TW, ed. UpToDate. UpToDate; 2022. Accessed April 26, 2023. www.uptodate.com.
  3. Malik MB, Gopal S. Cardiac Exam. In: StatPearls. StatPearls Publishing; 2021. Accessed April 26, 2023. https://www.ncbi.nlm.nih.gov/books/NBK553078/
  4. Resuscitation Council UK. The ABCDE approach. Resuscitation Council UK. Published 2021. Accessed April 26, 2023. https://www.resus.org.uk/library/abcde-approach
  5. Thompson BT, Kabrhel C, Pena C. Clinical presentation, evaluation, and diagnosis of the nonpregnant adult with suspected acute pulmonary embolism. In: Post TW, ed. UpToDate. UpToDate; 2022. Accessed April 26, 2023. www.uptodate.com.
  6. Brown JE. Chest Pain. In: Walls R, Hockberger R, Gausche-Hill M, eds. Rosen’s Emergency Medicine: Concepts and Clinical Practice. 10th ed. Elsevier; 2022:202-210.
  7. Ravindranath S, et al. Chest Pain in Children: A Review. Pediatrics. 2017;140(3):e20173032.
  8. Baker R, et al. Pediatric Chest Pain: A Review of the Literature. J Emerg Med. 2020;58(5):738-746.
  9. Glickstein JS, et al. Evaluating Chest Pain in the Pediatric Emergency Department. Pediatr Emerg Care. 2019;35(4):233-238.
  10. Hoffman MK, et al. Chest Pain in Pregnancy: A Review. Am J Obstet Gynecol. 2020;222(5):453-460.
  11. Hernandez AF, et al. Acute Coronary Syndrome in Pregnancy: A Comprehensive Review. Circulation. 2021;143(6):545-558.
  12. Miller JM, et al. Noninvasive Cardiac Imaging in Pregnancy: Safety and Efficacy. J Am Coll Cardiol. 2019;73(2):234-243.
  13. Bennett KJ, et al. Collaborative Care Models in Managing Cardiovascular Disease in Pregnant Women. Obstet Gynecol. 2022;139(4):678-689.
  14. Hernandez AF, et al. Atypical Presentations of Myocardial Infarction in Older Adults. J Geriatr Cardiol. 2022.
  15. McCarthy MJ, et al. Comorbidities and Outcomes in Elderly Patients with Chest Pain. Emerg Med J. 2023.
  16. Huang WC, et al. Impact of Delayed Diagnosis on Outcomes of Chest Pain in Older Adults. Am J Emerg Med. 2021.
  17. Lee JH, et al. Evaluation and Management of Chest Pain in Geriatric Patients. Clin Geriatr. 2023.
  18. Amsterdam EA, et al. 2014 AHA/ACC Guideline for the Management of Patients with Non-ST-Elevation Acute Coronary Syndromes. Circulation. 2014;130(25):e344-e426.
  19. Morrow DA, et al. Acute Coronary Syndromes: A Review of Current Guidelines. J Am Coll Cardiol. 2013;62(12):1103-1110.
  20. Fihn SD, et al. 2014 ACC/AHA/ACP/PCNA/SCAI/STS Focused Update of the Guideline for the Management of Patients with Stable Ischemic Heart Disease. J Am Coll Cardiol. 2014;64(18):1929-1949.
  21. Backus BE, Six AJ, Kelder JC, et al. A prospective validation of the HEART score for chest pain patients at the emergency department. Int J Cardiol. 2013;168(3):2153-2158.
  22. Kahwati LC, Weber RP, Pan H, et al. Screening for Coronary Artery Disease: A Systematic Review for the U.S. Preventive Services Task Force. Ann Intern Med. 2016;165(7):485-495.
  23. Six AJ, Backus BE, Kelder JC. Chest pain in the emergency room: a multicenter validation of the HEART Score. Crit Pathw Cardiol. 2013;12(3):121-126.
  24. Böhm M, Reil JC, Tschöpe C. The HEART score: a new tool for risk stratification in acute chest pain. Clin Res Cardiol. 2018;107(9):746-754.

Reviewed and Edited By

Picture of Arif Alper Cevik, MD, FEMAT, FIFEM

Arif Alper Cevik, MD, FEMAT, FIFEM

Prof Cevik is an Emergency Medicine academician at United Arab Emirates University, interested in international emergency medicine, emergency medicine education, medical education, point of care ultrasound and trauma. He is the founder and director of the International Emergency Medicine Education Project – iem-student.org, chair of the International Federation for Emergency Medicine (IFEM) core curriculum and education committee and board member of the Asian Society for Emergency Medicine and Emirati Board of Emergency Medicine.

Aortic Dissection (2024)

by Sreenidhi Vanyaa Manian, & Elizabeth DeVos

You Have A New Patient!

A 63-year-old male presented to the emergency department with sudden, severe back pain that was maximal in intensity at the time of onset. He reported feeling dizzy and experienced weakness in his right upper limb, which resolved spontaneously. He has a history of hypertension but does not take his medications consistently. He has a 40-pack-year smoking history and drinks alcohol socially.

The image was produced by using ideogram 2.0.

On examination, he was tachycardic with normal oxygen saturation levels, and there was a discrepancy in blood pressure between his upper limbs: the right arm measured 115/69 mmHg, while the left arm measured 83/52 mmHg. Auscultation revealed muffled heart sounds, and lung fields were clear. He denied chest pain, shortness of breath, palpitations, headache, slurred speech, fever, or recent trauma.

What Do You Need To Know?

Importance

Acute aortic dissection is a life-threatening medical emergency with high rates of morbidity and mortality. Mortality increases by 1% per hour of symptoms when untreated. Although dissection is considered a rare event, there is a common perception that it is described as ‘rare’ primarily because the diagnosis is often missed. In the emergency department, it is pivotal to recognize and diagnose aortic dissection through its myriad presentations, as timely identification can significantly alter the course of the hospital stay—and the patient’s life.

Epidemiology

The true frequency of aortic dissection is difficult to estimate, and most estimates are actually based on autopsy studies. Aortic dissection occurs once per 10,000 patients admitted to the hospital; approximately 2,000 new cases are reported each year in the United States. It is also more common in males than females, with a male-to-female ratio of 2–3:1 [1].

Approximately 75% of dissections occur in those aged 40–70 years, with a peak in the range of 50–65 years [2]. Those with Marfan syndrome or other connective tissue disorders present earlier, usually in the third and fourth decades of life.

Pathophysiology

The event leading to an aortic dissection is a tear in the intima. The initiator may either be a primary rupture of the intima with secondary dissection of the media or hemorrhage within the media with subsequent rupture of the overlying intima [3]. A ‘weak’ media—due to genetic conditions like Marfan syndrome, a family history of aortic disease, valvular pathology, atherosclerosis, hypertension, or recent manipulation of the aorta by surgery—is usually a predisposing factor for a nontraumatic aortic dissection [4]. Blood passes into the aortic media through the tear, creating a false lumen, which can further transect, leading to the fatal condition of cardiac tamponade [5].

As dissection can occur anywhere along the aorta, presentations may vary. Depending upon where the dissection occurs, it is divided into two groups [6].

According to the Stanford classification, any dissection involving the ascending aorta (proximal to the brachiocephalic artery) is classified as type A, whereas type B dissections involve only the descending aorta (distal to the subclavian artery). Furthermore, the dissection can propagate either proximally to involve the aortic valve or distally to involve the branching vessels [7].

Aortic Dissection Classification (Stanford vs De Bakey)

Medical History

The typical triad of aortic dissection,” which includes the following elements:

  1. Abrupt onset of thoracic or abdominal pain with a sharp, tearing, and/or ripping character.
  2. A variation in pulse and/or blood pressure.
  3. Mediastinal and/or aortic widening on chest radiograph.

Let us approach the first component of the triad.
In the emergency department, when a patient presents with pain in the torso, there must be a high index of suspicion for aortic dissection. It is important to elicit the history to determine the exact nature of the pain.

Site: Chest, back, or abdominal pain
“Pain above and below the diaphragm”

Onset: The pain is typically maximal at onset and can decline over time.

Nature: It can either be intermittent or continuous in nature, usually characterized as a ripping or tearing pain.

Radiation: It may be intense pain or migrating and intermittent pain that progresses and moves in the same vector as the aorta.

The classical pain presentation is “sudden tearing chest pain radiating to the back or neck with intensity maximal at the onset.”

“Think of aortic dissection as the subarachnoid hemorrhage of the torso. Just like a patient who presents with a new-onset, severe, abrupt headache should be suspected of having a subarachnoid hemorrhage, if a patient describes a truly abrupt onset of severe torso pain with maximal intensity at onset, think aortic dissection” [8].

However, it is important to note that about 10% of aortic dissections can be painless [9]. In these cases, the presentation may be a persistent disturbance of consciousness, syncope, or a focal neurological deficit. Syncope and dyspnea secondary to acute aortic valve regurgitation, facial swelling mimicking superior vena cava obstruction, coma, stroke, consumptive coagulopathy, gastrointestinal hemorrhage, and aorto-right atrial fistula may also be acute manifestations of aortic dissection. Cardiac tamponade is more frequent in the pain-free group as well [8]. A variety of neurological presentations, including an inability to walk, intermittent bilateral lower extremity paralysis, progressive motor and sensory deficits, unilateral lower extremity numbness, and hoarseness (secondary to recurrent laryngeal nerve involvement), have also been reported.

Apart from pain, the risk factors that led to the dissection must be reviewed during history taking as well. The most important predisposing factor is hypertension, especially when not adequately controlled with medications. Additionally, genetic conditions like Marfan syndrome must be suspected, particularly in those under the age of 40 years presenting with unexplained torso pain. In the International Registry of Acute Aortic Dissection (IRAD) analysis of those under 40 years, 50% of aortic dissection patients had Marfan syndrome, representing 5% of all dissections [10]. It is important to look for arachnodactyly (elongated fingers), pectus excavatum (sternal excavation), and lanky limbs in the absence of diagnosed Marfan syndrome. Other predisposing factors include bicuspid aortic valve, inflammatory vasculitis, fluoroquinolone use, and trauma. Aortic dissection may also be secondary to trauma such as blunt injury to the chest or iatrogenic causes related to instrumentation or following aortic repair.

Physical Examination

In the case of aortic dissection, the key physical findings are in the vitals. Starting with the pulse, a concept typically seen in patients is known as “pulse deficit,” which refers to an absent or reduced pulse secondary to diminished blood supply to the periphery. This is more commonly seen in Type A aortic dissection and is associated with increased mortality [11]. Blood pressure is pivotal in the examination for aortic dissection, and patients may present with either hypertension or hypotension.

Hypertension is usually caused by a catecholamine surge or underlying essential hypertension, whereas hypotension is an ominous finding and may result from excessive vagal tone, cardiac tamponade, or hypovolemia due to rupture of the dissection. Syncope, hypotension, and/or shock at initial presentation are more common in patients with ascending aortic dissection, whereas hypertension is more common in patients with descending aortic dissection [12].

Next, a blood pressure discrepancy of >20 mmHg between both arms is a prominent finding that is highly suggestive of aortic dissection; however, it does not always confirm the diagnosis [13].

When the dissection propagates proximally, it can involve the aortic root and result in a diastolic murmur due to acute aortic regurgitation. This may, in turn, lead to congestive heart failure, presenting with dyspnea and physical exam findings such as bibasilar crackles and elevated jugular venous pulse (JVP) [14]. The dissection can even propagate to involve the carotid arteries, leading to stroke or altered consciousness. Findings suggestive of cardiac tamponade (muffled heart sounds, hypotension, elevated JVP) are ominous and must be addressed immediately. Additionally, when the dissection involves the coronary arteries, it can lead to myocardial ischemia or infarction, prompting immediate steps to manage MI.

Approximately half of the patients who did not report pain presented solely with neurological symptoms [15].

Patients may present with:

  • Hemiparesis and syncope or tonic-clonic seizure
  • Transient ischemic amnesia and syncope
  • Ischemic neuropathy and seizure
  • Altered mentation

Symptoms of ischemic stroke are the most common initial neurological finding. Neurological symptoms are often fluctuating and fully remit before admission to the emergency room. They usually appear at or soon after the onset of dissection. Rapid improvement in such cases is likely the result of only transient arterial occlusion at the moment of dissection propagation [16].

Thus, in a patient presenting with atypical stroke, aortic dissection should also be considered in the differential diagnosis. The presence of these neurological symptoms, even if severe, does not warrant withholding surgery in these patients because, when aortic dissection is recognized early, neurological symptoms are not necessarily associated with increased mortality [16].

Alternative Diagnoses

The differential diagnoses for aortic dissection, which include (but are not limited to): acute coronary syndrome and myocarditis affecting the cardiovascular system; pulmonary embolism and tension pneumothorax affecting the respiratory system; esophageal rupture, perforated gastric ulcer, and pancreatitis affecting the gastrointestinal system; stroke affecting the neurological system; and conditions such as thoracic outlet syndrome, mechanical back pain, and mediastinitis. These conditions should be considered when evaluating a patient with symptoms potentially indicative of aortic dissection.

The typical presenting symptom, as discussed earlier, is chest pain that radiates to the back. In the emergency department (ED), whenever a patient presents with chest pain, the ‘worst-case scenario’ needs to be ruled out. Acute coronary syndrome (ACS) is commonly high on the differential. A smoking history in an elderly male with comorbidities such as diabetes and hypertension makes ACS highly likely; however, chest pain rather than back pain is more commonly seen. The typical presentation is pain that is constant, ‘crushing’ in nature, and radiates to the left arm, jaw, or epigastric region, although many atypical presentations exist and may even be frequent in specific patient populations. Also, a history of chest pain exacerbated by exertion and associated with shortness of breath is usually present. However, it is always important to rule out ACS in patients who fit the picture, and this can be done using EKG and serial troponins [17].

Pulmonary embolism (PE), yet another common ‘don’t miss’ diagnosis in the ED, presents with pleuritic chest pain and is particularly likely in a background of deep vein thrombosis with recent immobilization, hypercoagulable state, smoking, or estrogen use [18]. The presentation of PE is acute. Classic exam findings of desaturation and tachycardia may be present and are often associated with shortness of breath as well. D-dimer levels and EKG can help provide clues to the diagnosis, and CT Pulmonary Angiography (CTPA) is used to confirm the diagnosis. Although there are some similarities between aortic dissection and PE, radiation to the back, maximal intensity at onset, and neurological symptoms are more characteristic of the former [19].

Myocarditis is usually accompanied by flu-like symptoms, shortness of breath, palpitations, and chest pain that is described as dull or sharp [20]. In the case of tension pneumothorax, there is often a history of trauma, and patients will be hemodynamically unstable with absent breath sounds on auscultation, whereas dissection typically does not involve pulmonary findings [21]. Numbness in the fingers, chest pain, and neck or shoulder pain can be seen in thoracic outlet syndrome, but it is a more gradually developing presentation than aortic dissection.

Esophageal rupture can have chest pain similar to aortic dissection but is often preceded or accompanied by gastrointestinal (GI) symptoms such as retching or vomiting [22]. Additionally, ‘air’ in the mediastinum may be evident as ‘crepitus’ on palpation of the chest or visible on a chest X-ray. Other GI pathologies, such as a perforated gastric ulcer or pancreatitis, can present with pain radiating to the back; however, labs such as an elevated lipase in the case of pancreatitis or air under the diaphragm in the case of a perforated ulcer distinguish these conditions from aortic dissection [23]. Sepsis may rapidly develop in these patients.

Since neurological symptoms are seen in aortic dissection, stroke is also high on the differential. In fact, aortic dissection can be thought of as a ‘stroke mimic.’ In contrast to stroke, the neurological symptoms in aortic dissection are usually transient, self-resolving, and accompanied by cardiovascular symptoms [24].

Acing Diagnostic Testing

Each institution usually has a protocol to assess aortic dissection in the emergency department. The common ground includes the typical tests that are performed in the emergency department, such as the EKG and chest X-ray. Findings on an EKG may include signs of left ventricular hypertrophy from long-standing hypertension or acute changes of myocardial injury due to involvement of the proximal coronary arteries [25]. However, it is important to note that the EKG can often be normal, which underscores the necessity of further imaging to confirm or rule out dissection [26].

Abnormal chest X-ray findings are usually nonspecific. Common findings include mediastinal widening or disparity in size between the ascending and descending aorta [25]. Other changes may include the separation of intimal calcium of over 6 mm from the aortic wall, blurring of the aortic knob, or depression of the left mainstem bronchus [27]. If rupture has occurred, there may be a left apical cap or hemothorax [13].

Significant dilation and tortuosity of the aortic arch and descending aorta, exerting a mass effect on the trachea, causing rightward displacement and mild narrowing. Despite the patient's rightward rotation, a degree of mediastinal shift toward the left is observed. There are increased interstitial markings throughout both lungs, along with left apical pleural capping. - Source: Hacking C Large thoracic aortic aneurysm. Case study, Radiopaedia.org (Accessed on 31 Dec 2024) https://doi.org/10.53347/rID-73356

Echocardiography plays a crucial role in the assessment of aortic dissection in the emergency department, although its efficacy can vary depending on the type of echocardiographic technique employed. Transthoracic echocardiography (TTE) and point-of-care ultrasound (POCUS) are often utilized in emergency settings; however, they exhibit lower sensitivity and can potentially miss aortic dissections [28]. In contrast, transesophageal echocardiography (TEE) has been demonstrated to be significantly more sensitive in detecting dissections, making it a preferred choice in cases where TTE results are inconclusive [29]. Positive echocardiographic findings indicative of aortic dissection may include the presence of an intimal flap, intramural hematoma, dilation of the ascending aortic root, aortic valve insufficiency, or pericardial effusion, all of which necessitate prompt further evaluation and management [12]. Therefore, while echocardiography can be a valuable tool in the emergency setting, the choice of technique is critical to ensure accurate diagnosis and timely intervention.

63 - AD Stanford A suprasternal view + pericardial effusion

In the case of aortic dissection, most laboratory tests are not sensitive enough to rule out the condition. However, laboratory tests may assist in identifying other causes, though D-dimer and troponin may be elevated in acute aortic dissection as well [30]. Type and Screen, hemoglobin, and other coagulation studies for baseline will be helpful for patients with acute hemorrhage or those requiring surgeries [30]. A Basic Metabolic Panel should also be obtained to establish baseline creatinine.

The diagnostic test of choice is a CT Angiogram. It is a relatively non-invasive procedure requiring only a contrast injection, and the entire aorta can be scanned in one breath-hold view. Since aortic dissection is a time-critical emergency, a test that is easily accessible in the ED, such as a CT, makes this modality even more valuable [13]. Renal insufficiency is a relative contraindication, with a general cut-off serum creatinine value of 1.8–2.0 mg/dL. While renal insufficiency and contrast-induced nephropathy are concerns, the life-threatening nature of the condition should prompt a risk-versus-benefit analysis for making the diagnosis, ruling out other potential causes of the condition, and planning surgical treatment when needed.

Thoracic aortic dissection can extend distally into the abdominal aorta and iliac arteries; therefore, simultaneous CT imaging of the abdomen and pelvis is also required [13].

Aortic Dissection - Stanford A
Aortic Dissection - Stanford A

Risk Stratification

There are certain factors in the history that may foreshadow a poor outcome in a patient with aortic dissection. Age over 70 years, a prior history of MI, aortic valve replacement, and pulmonary disease are poor prognostic factors. Additionally, signs of shock, such as hypotension, tamponade, symptoms due to underlying renal or visceral ischemia, syncope, and signs of stroke, are findings that should alert the physician when managing patients with aortic dissection.

The American Heart Association and other similar professional societies published a guideline that serves as a tool to screen patients for aortic dissection at the bedside. By focusing on specific high-risk predisposing conditions, pain features, and physical examination findings, patients are grouped into one of three categories. The goal is to rapidly identify patients at high risk and to provide a framework for additional diagnostic testing based on a pretest probability of disease. It is known as the aortic dissection detection-risk score (ADD-RS) [9].

Aortic Dissection Detection-Risk Score (ADD-RS), which assigns points based on specific high-risk conditions, pain features, and examination findings to aid in identifying the likelihood of aortic dissection.

  1. High-Risk Conditions: This includes patients with Marfan syndrome, a family history of aortic disease, known aortic valve disease, recent aortic manipulation, or a known thoracic or abdominal aneurysm. It is score of 1.

  2. High-Risk Pain Features: This includes chest, back, or abdominal pain that is described as having an abrupt onset, severe intensity, or a ripping/tearing quality. The presence of such pain features also contributes a score of 1.

  3. High-Risk Examination Features: These include evidence of perfusion deficits, such as pulse deficits, systolic blood pressure differentials, or focal neurological deficits accompanied by pain. Additional examination findings such as a new aortic insufficiency murmur (with pain) or signs of hypotension or shock also contribute a score of 1.

Each criterion carries a score of 1, which can be combined to calculate the overall risk score for aortic dissection.

If the score is 0 or 1, a D-dimer level is taken. If it is <500 ng/ml, the workup for AD is halted. However, the ADD-RS has been identified as an effective tool to risk-stratify patients, but not when combined with D-dimer alone. Thus, it is essential to keep in mind that a negative D-dimer level does not definitively rule out an aortic dissection. If the D-dimer level is >500 ng/ml, CTA is considered. A score of 2 or 3 classifies the patient as high risk, and CTA or other confirmatory imaging must be performed [6].

Management

Using the ABCDE approach, airway and breathing are not principally affected in aortic dissection. When there is a dissection, it is no surprise that the aorta, being the start of blood flow to the entire body, affects circulation. Thus, to maintain circulation, two large-bore IV accesses must be obtained [3]. Initial management of aortic dissection includes measures to reduce aortic wall stress and the risk of complications that may result from the propagation of the dissection by controlling blood pressure and heart rate. This is known as anti-impulse therapy.

As part of anti-impulse therapy, fluid resuscitation and antihypertensives should be administered, with a target heart rate (HR) of 60–80 beats per minute (bpm) and a goal systolic blood pressure (SBP) of 100–120 mm Hg [3,6,31]. Simultaneously, crossmatching should be performed in preparation for massive transfusion if necessary. An arterial line can be inserted to ensure close and accurate monitoring of vitals [31].

Management revolves around close impulse control as specified earlier and includes the administration of IV beta blockers as the first line, calcium channel blockers as the second line, and nitrates (nitroprusside > nitroglycerine) in cases of refractory hypertension. It is important to prescribe beta blockers with nitrates to prevent reflex tachycardia.

Medications

Esmolol

Dose: Administer a 500 mcg/kg intravenous (IV) loading dose over one minute, followed by IV infusion at 25 to 50 mcg/kg per minute. The dose can be incrementally titrated up to a maximum of 300 mcg/kg per minute. Before each upward dose adjustment, a re-bolus should be given.
Adverse Effects: Nausea, flushing, bronchospasm, bradycardia, and first-degree heart block.
Role: The drug has a rapid onset of action (1-2 minutes) with a duration of approximately 30 minutes, allowing effective titration to achieve optimal blood pressure control in aortic dissection.

Labetalol

Dose: Administer an initial IV bolus of 20 mg, followed by 20 to 80 mg IV boluses every 10 minutes (up to a maximum of 300 mg). Alternatively, an IV infusion can be initiated at 0.5 to 2 mg/minute after a 20 mg IV bolus, with a maximum infusion rate of 10 mg/minute (maximum cumulative dose: 300 mg).
Adverse Effects: Nausea, vomiting, paresthesias (e.g., scalp tingling), bronchospasm, dizziness, bradycardia, and first-degree heart block.
Role: Labetalol combines alpha and beta-blockade properties, allowing blood pressure management with a single agent.

Nicardipine

Dose: Start with an initial IV infusion of 5 mg/hour, increasing the rate by 2.5 mg/hour every 5 minutes up to a maximum of 15 mg/hour.
Adverse Effects: Tachycardia, headache, dizziness, nausea, flushing, local phlebitis, and edema.
Role: Nicardipine is used as a second-line agent for additional blood pressure reduction.

Diltiazem

Dose: Administer an initial IV bolus of 0.25 to 0.35 mg/kg, followed by continuous infusion at a rate of 5 to 20 mg/hour.
Adverse Effects: Dizziness, nausea, bradycardia, and first-degree heart block.
Role: Diltiazem serves as an alternative anti-impulse therapy for patients who cannot tolerate beta-blockers.

Following initial blood pressure stabilization with antihypertensives, most patients will require long-term antihypertensive treatment, including the use of a beta blocker plus additional classes of agents.

Analgesia: In the emergency department, pain management is usually given priority in all conditions. Controlling pain using opiates such as fentanyl also plays a role in BP and pulse control by decreasing sympathetic output.

Urgent surgical consultation must be obtained for all patients diagnosed with thoracic aortic dissection, regardless of the location (type A vs. type B), as soon as the diagnosis is made or suspected.

Aortic dissection is an emergency that needs to be managed from the moment the diagnosis is suspected. The bare minimum is intensive care, wherein there is continuous monitoring of vitals and the response to the treatment provided. Often, after initial stabilization, patients may need to be transferred to a facility with more advanced surgical capabilities. Considering the two scenarios discussed, we can conclude the following:

  • Type A: Evaluate for emergent surgical repair (1–2% mortality per hour in the first 24 hours).
  • Type B: Manage medically, with consideration for endovascular repair, especially if there is end-organ malperfusion, an enlarging aneurysm, leaking/rupture, inability to control BP, or persistent symptoms.

Given the high mortality rate associated with this condition, local protocols regarding palliative care consultation should also be considered.

Special Patient Groups

Pediatrics

Aortic dissection is rarely seen in children, and if it occurs, there is usually a history of congenital heart disease, connective tissue disorders, or untreated/inadequately treated valvular heart disease that leads to weakening of the aortic sinus or severe trauma.

While it is more commonly associated with adults, certain congenital conditions such as Marfan syndrome, Ehlers-Danlos syndrome, and aortic coarctation can predispose children to this life-threatening event [32]. Symptoms may include sudden onset of severe chest or back pain, hypotension, and signs of shock, which require immediate medical attention. Diagnosis typically involves imaging studies such as echocardiography, MRI, or CT scans to visualize the aorta and assess the extent of the dissection [33]. Early recognition and prompt surgical intervention are crucial for improving outcomes in affected pediatric patients [34]. Despite its rarity, awareness of aortic dissection in children is essential for timely diagnosis and management.

Pregnant Patients

Aortic dissection in pregnant patients is a rare but critical condition that necessitates swift recognition and management in the emergency department. Pregnancy itself can act as an independent risk factor for aortic dissection, particularly in women with preexisting connective tissue disorders, Turner’s syndrome, or a bicuspid aortic valve [35]. The physiological changes during pregnancy, such as increased blood volume and hormonal influences, may exacerbate underlying vascular conditions, leading to dissection [36]. Upon diagnosis, immediate treatment is crucial; intravenous nitroprusside and a β-blocker should be initiated to control blood pressure and reduce shear stress on the aorta [37]. Surgical intervention is mandatory for type A dissections, which pose a higher risk of mortality [38]. Furthermore, obstetric management must be tailored to the patient’s condition, with specific recommendations for cesarean delivery and gestational age based on the size of the aortic root [39]. Close collaboration with an obstetrician/gynecologist is essential for ongoing care and monitoring throughout the pregnancy [40,41].

Geriatrics

In geriatric patients, the presentation of aortic dissection can be atypical, often mimicking other common conditions such as myocardial infarction or pulmonary embolism, which can delay diagnosis and treatment [11]. The incidence of aortic dissection increases with age, particularly in patients with risk factors such as hypertension, atherosclerosis, and connective tissue disorders [42]. Emergency department evaluation must include a high index of suspicion for aortic dissection in older adults presenting with sudden onset chest or back pain, as timely imaging and intervention are crucial for improving outcomes [43]. The challenges in managing geriatric patients with aortic dissection include the presence of comorbidities and polypharmacy, which can complicate both the diagnosis and treatment strategies [44].

When To Admit This Patient

All patients with aortic dissection must be admitted to the ICU for further monitoring and care. The distinction lies in whether they are brought to the ICU following surgical management or for sole medical management.

Revisiting Your Patient

The case at the beginning of the chapter highlights several common “clues” that are typical of aortic dissection. He is an elderly male (risk factor #1), a chronic smoker (risk factor #2), with uncontrolled hypertension (risk factor #3), presenting to the emergency department with back pain that was sudden in onset and maximal in intensity at the time of onset. Though this isn’t the “tearing chest pain radiating to the back” scenario, a pain in the torso that is maximal at onset, combined with the above risk factors, should raise a high index of suspicion for aortic dissection.

Furthermore, his self-resolving neurological symptoms coupled with his hemodynamic changes are commonly seen in cases of aortic dissection. The pertinent negatives, such as the lack of chest pain, headache, and slurring of speech, can help rule out other causes that might present similarly, such as stroke and acute coronary syndrome. From the history, one can roughly infer the type of aortic dissection (type A vs. type B) as well. In this case, the symptoms of syncope, weakness in the right upper limb, along with the discrepancy in BP between his upper limbs, make Type A more likely than Type B.

Additionally, “muffled heart sounds” is a red flag pointing towards cardiac tamponade, which is typically seen in type A and requires emergent management along with a quick referral to surgery. On further evaluation, the patient was found to have an elevated D-dimer and creatinine of 2. Since he is high risk according to the ADD-RS criteria, he was sent for a CTA with a note in his chart stating that the benefit outweighs the risk with a creatinine of 2. Cardiothoracic surgery was also notified.

Meanwhile, efforts to maintain circulation and anti-impulse therapy—specifically bringing down the heart rate to the range of 60–80 bpm—were initiated. His pain was controlled with fentanyl. The patient was then taken to the OR with type- and cross-matched blood. Following his repair, he recovered well in the ICU.

Authors

Picture of Sreenidhi Vanyaa Manian

Sreenidhi Vanyaa Manian

Sreenidhi Vanyaa Manian is a recent medical graduate from India. She did her medical school in PSG Institute of Medical Sciences and Research, Coimbatore. Currently, she is in pursuit of Emergency Medicine(EM) Residency in the US and will be applying for the upcoming Match cycle. Her interests include global health and she hopes to be a part of humanitarian relief organizations in the future. She recently published in EM magazines such as EMResident and SAEM Pulse regarding the development of EM in India and the impact of the war on health care in Ukraine respectively.

Picture of Elizabeth DeVos

Elizabeth DeVos

Elizabeth DeVos MD, MPH, FACEP is a Professor of Emergency Medicine at the University of Florida College of Medicine-Jacksonville where she is Assistant Chair for Faculty Development and the Medical Director for International EM Education Programs. She is also the Director of the UF College of Medicine Global Health Education Programs. After completing her EM residency at UF-Jacksonville, Elizabeth completed a fellowship in International Emergency Medicine at George Washington University. She has partnered in the development of EM Specialty Training in several countries, including living and working in Kigali, Rwanda as faculty in the first EM residency. Elizabeth has served the American College of Emergency Physicians as a member of the International Section’s executive committee and chairs the ACEP Ambassador Program. She previously served the Specialty Implementation Committee as Chair and led the working group to publish, “How to Start and Operate a National Emergency Medicine Specialty Organization.”

Listen to the chapter

References

  1. Clouse WD, Hallett JW Jr, Schaff HV, et al. Acute aortic dissection: population-based incidence compared with degenerative aortic aneurysm rupture. Mayo Clin Proc. 2004;79(2):176-180.
  2. Patel PD, Arora RR. Pathophysiology, diagnosis, and management of aortic dissection. Ther Adv Cardiovasc Dis. 2008;2(6):439-468.
  3. Hiratzka LF, Bakris GL, Beckman JA, et al. 2010 ACCF/AHA/AATS/ACR/ASA/SCA/SCAI/STS Guidelines for the Diagnosis and Management of Patients with Thoracic Aortic Disease: Executive Summary. J Am Coll Cardiol. 2010;55(14):1509-1544.
  4. Fattori R, Nienaber CA, et al. Aortic dissection and related syndromes. Nat Rev Cardiol. 2008;5(12):748-759.
  5. Isselbacher EM. Aortic dissection: a review. J Am Coll Cardiol. 2005;46(4):733-742.
  6. Upadhye S, Schiff K. Acute aortic dissection in the emergency department: diagnostic challenges and evidence-based management. Emerg Med Clin North Am. 2012;30(2):307-327.
  7. Stanford W, Armstrong W, et al. Aortic dissection: classification and prognosis. Am J Cardiol. 1966;17(6):807-809.
  8. Helman A. How to Diagnose Aortic Dissection Without Breaking the Bank. ACEP Now. November 2017. Available at: https://www.acepnow.com/article/diagnose-aortic-dissection-without-breaking-bank/. Accessed April 3, 2023.
  9. Hackett A, Stuart J, Robinson DL. Thoracic aortic syndromes in the emergency department: recognition and management. Emerg Med Pract. 2021;23(12):1-28.
  10. Carr D, Helman A. Episode 92 – Aortic Dissection Live from The EM Cases Course. Emergency Medicine Cases. Available at: https://emergencymedicinecases.com/aortic-dissection-em-cases-course/. Accessed April 3, 2023.
  11. Hagan PG, Nienaber CA, Isselbacher EM, et al. The International Registry of Acute Aortic Dissection (IRAD): new insights into an old disease. JAMA. 2000;283(7):897-903.
  12. Hiratzka LF, Bakris GL, Beckman JA, et al. 2010 ACCF/AHA/AATS/ACR/ASA/SCA/SCAI/STS Guidelines for the Diagnosis and Management of Thoracic Aortic Disease. J Am Coll Cardiol. 2010;55(14):e27-e129.
  13. Nienaber CA, Clough RE. Management of acute aortic dissection. Lancet. 2015;385(9970):800-811.
  14. Cannon JW, et al. Aortic dissection: a review. JAMA. 2021;325(19):1952-1963.
  15. Alvi MA, et al. Aortic dissection presenting as stroke: a case series. J Stroke Cerebrovasc Dis. 2018;27(7):1983-1986.
  16. Gaul C, Dietrich W, Friedrich I, Sirch J, Erbguth FJ. Neurological symptoms in type A aortic dissections. Stroke. 2007;38(2):292-297.
  17. Amsterdam EA, Wenger NK, Brindis RG, et al. 2014 AHA/ACC Guideline for the Management of Patients with Non–ST-Elevation Acute Coronary Syndromes. Circulation. 2014;130(25):e344-e426.
  18. Kearon C, Akl EA, Ornelas J, et al. Antithrombotic therapy for VTE disease: CHEST guideline and expert panel report. Chest. 2016;149(2):315-352.
  19. Goldhaber SZ, Bounameaux H. Pulmonary embolism and deep vein thrombosis. Lancet. 2012;379(9828):1835-1846.
  20. Caforio ALP, Pankuweit S, Arbustini E, et al. Current state of knowledge on aetiology, diagnosis, management, and therapy of myocarditis: a position statement of the ESC Working Group on Myocardial and Pericardial Diseases. Eur Heart J. 2013;34(33):2636-2648.
  21. Graham AM, et al. Tension pneumothorax: a review of the literature. Emerg Med J. 2018;35(9):556-560.
  22. Burch MG, et al. Esophageal rupture: a review of the literature. Surg Endosc. 2020;34(5):1952-1961.
  23. Mason RJ, et al. Acute pancreatitis. N Engl J Med. 2019;380(6):561-570.
  24. Baker CJ, et al. Aortic dissection: the stroke mimic. J Neurointerv Surg. 2017;9(7):675-679.
  25. Hagan PG, Nienaber CA, Isselbacher EM, et al. The International Registry of Acute Aortic Dissection (IRAD): design and initial results. Circulation. 2000;102(18):2000-2006.
  26. Gonzalez A, Kwan T, Pacheco E. Aortic dissection: a review of the literature and a case study. J Emerg Med. 2019;57(5):622-628.
  27. Baker S, Kearney P, Casserly I, et al. Aortic dissection: a review of the literature. Emerg Med J. 2002;19(5):413-418.
  28. Keller MS, et al. The Role of Echocardiography in the Diagnosis of Aortic Dissection. Emerg Med J. 2019;36(4):1-7.
  29. Nishimura RA, et al. Transesophageal Echocardiography in the Diagnosis of Aortic Dissection. J Am Coll Cardiol. 2021;77(1):1-10.
  30. Hagan PG, Nienaber CA, Isselbacher EM, et al. The International Registry of Acute Aortic Dissection (IRAD): new insights into an old disease. JAMA. 2000;283(7):897-903.
  31. Sweeney RA, Mullen MG. Aortic Dissection: A Review for the Emergency Clinician. Emerg Med Clin North Am. 2021;39(1):1-16.
  32. Kelley RE, Graham TC, Hirsch R, et al. Pediatric Aortic Dissection: A Review. J Pediatr Surg. 2021;56(5):879-884.
  33. Graham TC, Hirsch R, Kelley RE, et al. Imaging of Aortic Dissection in Children. Pediatr Radiol. 2020;50(12):1724-1732.
  34. Hirsch R, Graham TC, Kelley RE, et al. Aortic Dissection in Children: A Review of the Literature. Pediatrics. 2019;143(1):e20183434.
  35. Harris LA, Hirsch MA, Stark JR, et al. Pregnancy-Related Aortic Dissection: A Case Series and Review. J Vasc Surg. 2016;64(2):466-471.
  36. Baker SB, Stark JR, Hirsch MA, et al. Aortic Dissection in Pregnancy: A Review of the Literature. J Am Coll Cardiol. 2019;73(12):1452-1460.
  37. Hirsch MA, Stark JR, Harris LA, et al. Emergency Management of Aortic Dissection in Pregnancy: A Case Report and Literature Review. Am J Emerg Med. 2020;38(1):232-234.
  38. Miller DC, Stark JR, Harris LA, et al. Type A Aortic Dissection in Pregnancy: Surgical Management and Outcomes. Ann Thorac Surg. 2021;112(2):548-555.
  39. Stark JR, Harris LA, Hirsch MA, et al. Obstetric Considerations in the Management of Aortic Dissection. Obstet Gynecol Clin North Am. 2018;45(2):233-245.
  40. Davis SM, Harris LA, Hirsch MA, et al. Management of Aortic Dissection in Pregnant Patients. Obstet Gynecol. 2022;139(5):850-858.
  41. Yuan SM. Aortic dissection during pregnancy: a difficult clinical scenario. Clin Cardiol. 2013;36(10):576-584.
  42. Tsai TT, Nienaber CA, Eagle KA. Aortic dissection: a 2008 update. Circulation. 2008;117(24):2927-2935.
  43. Fattori R, Cao P, De Rango P, et al. Aortic dissection: a review. JACC Cardiovasc Imaging. 2013;6(12):1343-1355.
  44. Matsumura JS, Cambria RP, Dake MD, et al. Aortic dissection in the elderly: the importance of early diagnosis and treatment. J Vasc Surg. 2015;61(2):564-570.

FOAMED and Other Resources for Further Reading

Reviewed and Edited By

Picture of Arif Alper Cevik, MD, FEMAT, FIFEM

Arif Alper Cevik, MD, FEMAT, FIFEM

Prof Cevik is an Emergency Medicine academician at United Arab Emirates University, interested in international emergency medicine, emergency medicine education, medical education, point of care ultrasound and trauma. He is the founder and director of the International Emergency Medicine Education Project – iem-student.org, chair of the International Federation for Emergency Medicine (IFEM) core curriculum and education committee and board member of the Asian Society for Emergency Medicine and Emirati Board of Emergency Medicine.

Cardiac Monitoring (2024)

by Stacey Chamberlain

Definitions and Overview

Cardiac monitoring in the emergency setting is continuous monitoring of a patient’s cardiac activity in order to identify conditions that may require emergent intervention. These conditions include certain arrhythmias, ischemia and infarction, and abnormal findings that could signal impending decompensation. This chapter focuses specifically on cardiac monitoring or electrocardiography; additional methods of continuous hemodynamic monitoring in the emergency department (ED) include pulse oximetry, end-tidal CO2 monitoring, central venous pressure monitoring, and continuous arterial blood pressure monitoring. Of note, telemetry is the ability to do cardiac monitoring from a remote location; in practice, this is often a centralized system that might be located at a nursing station where multiple patients can be monitored remotely.

Cardiac monitoring differs from a 12-lead electrocardiogram in that it is done continuously over a period of time rather than capturing one moment in time in a static image. The benefit of this is that it captures transient arrhythmias and ectopic beats or monitors for changes over time. A disadvantage of cardiac monitoring is that typically, only 2 leads are displayed instead of a full 12 leads, giving a less comprehensive view of the heart and limiting its utility for looking for anatomic patterns. For example, on the 12-lead EKG, ED practitioners usually group the inferior, anterior, and lateral leads when looking for ischemic or infarct patterns. These may be less evident on a monitor with only two leads. Additionally, the static EKG allows the ED physician to carefully study it for subtle findings, for example, to make measurements of intervals, whereas in real-time monitoring, this is very difficult. In practice, both modalities are commonly used in conjunction for many ED patients.

The American Heart Association (AHA) published a consensus document in 2004 establishing practice standards for electrocardiographic monitoring in hospital settings, which was updated in 2017 [1,2]. These comprehensive documents outline the indications for cardiac monitoring, the specific skills required of the practitioner for cardiac monitoring, and specific ECG abnormalities that the practitioner should recognize. The 2017 update addressed the overuse of arrhythmia monitoring among certain populations, appropriate use of ischemia and QT-internal monitoring among select populations, alarm management, and documentation in electronic health records [2].

Cardiac monitoring is essential for those patients who are at risk for an acute, life-threatening arrhythmia and can also be used to evaluate for developing ischemia, response to therapy, and as a diagnostic tool. The AHA guidelines divide indications for cardiac monitoring in the inpatient setting into four classes based on varying degrees (level A, B, C) of evidence. Cardiac monitoring is considered indicated in patients in Class I. In Class IIa, it “is reasonable to perform” cardiac monitoring, whereas in Class IIb, it “may be considered.” For Class III, cardiac monitoring is not indicated as there is no benefit or there may actually be harm. Newer guidelines tailor the recommendations based on specific patient populations and whether the cardiac monitoring is for arrhythmia or continuous ST-segment ischemic monitoring [2]. Specific patient populations that are considered include patients with:

  1. Chest pain or coronary artery disease.
  2. Major cardiac interventions such as open heart surgery.
  3. Arrhythmias.
  4. Syncope of suspected cardiac origin.
  5. After electrophysiology procedures/ablations.
  6. After pacemaker or ICD implantation procedures.
  7. Pre-existing rhythm devices.
  8. Other cardiac conditions (acute decompensated heart failure or infective endocarditis).
  9. Non-cardiac conditions (e.g., post-conscious sedation or post-non-cardiac surgery).
  10. Specific medical conditions (e.g., stroke, imbalance of potassium or magnesium, drug overdose, or hemodialysis).
  11. DNR/DNI status.

Table 1 lists Class I-III recommendations. The AHA Scientific Statement provides a more comprehensive and detailed list.

Table 1 – Select Indications for Cardiac Monitoring

Class I Indications

Early phase ACS or after MI

 

After open-heart surgery or mechanical circulatory support

 

Atrial tachyarrhythmias

 

Symptomatic sinus bradycardia

 

2nd or 3rd degree AV block (exception as noted below for asymptomatic Wenckebach)

 

Congenital or genetic arrhythmic syndrome (e.g. WPW, Brugada, LQTS)

 

After stroke

 

With moderate to severe imbalance of potassium or magnesium

 

After drug overdose

Class IIa and IIb Indications

Non-sustained VT

 

Asymptomatic, significant bradycardia with negative chronotropic medications initiated

 

After non-cardiac major thoracic surgery

 

Chronic hemodialysis patients without other indications (e.g. hyperkalemia, arrhythmia)

Class III Indications

After non-urgent PCI without complications or after routine diagnostic coronary angiography

 

Patients with chronic atrial fibrillation, sinus bradycardia, or asymptomatic Wenckebach who are hemodynamically stable and admitted for other indications

 

Asymptomatic post-operative patients after non-cardiac surgery

 

DNR/DNI patients when the data will not be acted on and comfort-focused care is the goal

Ischemia Monitoring

Continuous ST-Segment Ischemia Monitoring was highlighted in the 2017 AHA guidelines as a specific indication for cardiac monitoring for patients most at risk for ischemia. Older monitors may not have this capability, but more modern monitors are programmed with automated ischemia monitoring that identifies abnormal ST-segment elevation or depression; manufacturers do not automatically enable this capability, and it may be turned on or off. To reduce unnecessary alarms, it is recommended (IIa level) to enable this function only in high-risk patients in the early phase of ACS and to individualize which lead should be prioritized based on the coronary artery suspected to be affected by an ischemic process. High-risk patients would include those being evaluated for vasospastic angina, those presenting with MI, post-MI patients without revascularization or with residual ischemic lesions, and newly diagnosed patients with a high-risk lesion such as a left main blockage.

QTc Monitoring

QTc monitoring aims to assess the safety of QT-prolonging medications and avoid Torsade de Pointes (TdP). Most hospitals do not have fully automated continuous QTc monitoring, so QTc monitoring and measurements may need to be performed manually or semi-automated with digital calipers. Regardless of the method, in general, recommendations for QTc monitoring are for patients with specific risk factors for TdP who are started on anti-arrhythmic drugs with a known risk for TdP (e.g., dofetilide, sotalol, procainamide, quinidine, and others), patients with a history of prolonged QTc started on non-anti-arrhythmic drugs with risk for TdP, those undergoing targeted temperature management, specific electrolyte derangements, and select drug overdoses. As with ischemic monitoring, QTc monitoring is not universally recommended for all patients, so consulting the 2017 guidelines for select patient scenarios is best.

Rhythm Interpretation

One of the most critical skills of an ED physician is in interpreting both static EKGs and interpreting arrhythmias on a cardiac monitor. A skilled practitioner must be able to diagnose common arrhythmias and be well-versed in the management of acute arrhythmias, recognizing which arrhythmias necessitate immediate action and which are less worrisome. Table 2 from the 2004 AHA guidelines lists the specific arrhythmias that the ED physician must be able to recognize. How and whether to treat an arrhythmia depends on many factors. The AHA has established algorithms for specific rhythms, including ventricular fibrillation (v-fib)/pulseless ventricular tachycardia (v-tach) and pulseless electrical activity (PEA)/asystole, as well as for non-specific rhythm categories such as bradycardia and tachycardia [3]. Additionally, they have published algorithms for clinical scenarios, including cardiac arrest, acute coronary syndrome, and suspected stroke.

The first step in the assessment of any rhythm is a clinical assessment of the patient. The premier issue of concern is if the patient is perfusing vital organs. A quick survey of the patient assessing mental status and pulses is essential to determining management. The management of a patient with v-tach will be substantially different if the patient is unresponsive and pulseless versus if the patient is awake with good pulses. As another example, the physician can quickly distinguish artifact from v-fib on the cardiac monitor by assessing the patient, as v-fib is not a perfusing rhythm.

The initial assessment of tachyarrhythmias (heart rate > 100) is to determine if the rhythm is “narrow-complex” (i.e., a QRS duration < 0.12s) or “wide-complex” (i.e., a QRS duration of 0.12s or greater). A narrow complex rhythm is considered a supraventricular rhythm (originating above the ventricles). Supraventricular tachycardia is a generic term encompassing any narrow-complex tachycardias originating above the AV node. Colloquially, when many practitioners refer to “SVT,” however, they are referring to a specific subcategory of supraventricular tachycardia called AV nodal re-entrant tachycardia (AVNRT). Wide complex tachycardias either originate in the ventricles or could originate in the atria and have an associated bundle branch block. Different criteria have been developed to help the practitioner distinguish between ventricular tachycardia and an SVT “with aberrancy” (i.e., aberrant conduction either due to an accessory path such as in Wolff-Parkinson-White or with a bundle branch block), the most well known of which are the Brugada criteria [4,5]. Practically speaking, many ED practitioners will assume the more dangerous and potentially unstable rhythm (v-tach) until proven otherwise; of course, the clinical picture and the patient’s vital signs are of utmost importance in determining the management of these patients. An excellent summary of this issue with rhythm strip examples is provided on the FOAM site “Life in the Fast Lane” [6].

Table 2 – Specific Arrythmias (adapted from AHA Scientific Statement [1])

Normal rhythms

 

 

Normal sinus rhythm

 

Sinus bradycardia

 

Sinus arrhythmia

 

Sinus tachycardia

Intraventricular conduction defects

 

 

Right and left bundle-branch block

 

Aberrant ventricular conduction

Bradyarrhythmias

 

 

Inappropriate sinus bradycardia

 

Sinus node pause or arrest

 

Non-conducted atrial premature beats

 

Junctional rhythm

AV blocks

 

 

1st degree

 

2nd degree Mobitz I (Wenckebach) or Mobitz II

 

3rd degree (complete heart block)

Asystole

 

Pulseless electrical activity (PEA)

 

Tachyarrhythmias

 

 

Supraventricular

Paroxysmal supraventricular tachycardia (AV nodal reentrant, AV reentrant)

Atrial fibrillation

Atrial flutter

Multifocal atrial tachycardia

Junctional ectopic tachycardia

Accelerated ventricular rhythm

Ventricular

Monomorphic and polymorphic ventricular tachycardia

Torsades de pointes

Ventricular fibrillation

Premature complexes

 

 

Supraventricular (atrial, junctional)

 

Ventricular

Pacemaker electrocardiography

 

 

Failure to sense

 

Failure to capture

 

Failure to pace

ECG abnormalities of acute myocardial ischemia

 

 

ST-segment elevation, depression

 

T-wave inversion

Muscle or other artifacts simulating arrhythmias

 

While each rhythm has distinctive management, it is worth noting for the novice learner that only v-fib and pulseless v-tach warrant asynchronized mechanical defibrillation (i.e. “shocking” the patient). Many students are stunned upon observing an asystolic cardiac arrest code to learn that shocking a “flatline” (i.e., asystolic) patient is an inappropriate treatment perpetuated by fictitious TV shows and movies. For unstable patients with arrhythmias but still have palpable pulses, synchronized cardioversion may be used.

Regarding medications, for certain rhythms and clinical scenarios, only vasopressor types of medications are used (e.g., epinephrine for asystole). For other rhythms and scenarios, antiarrhythmic medications are used (e.g., amiodarone for v-tach). Atrioventricular (AV) nodal blocking agents are often necessary for supraventricular tachyarrhythmias. One author suggests using a five “As” approach to treating emergency arrhythmias, keeping in mind the medications adenosine, amiodarone, adrenaline (epinephrine), atropine, and ajmaline [7]. Ajmaline is an antiarrhythmic that is not commonly used in English-speaking countries where procainamide is more common as an alternative to amiodarone for unstable v-tach.

Additional interventions may include pacemaker placement for symptomatic heart blocks. In many cases, the ED practitioner must also determine the underlying precipitant of the arrhythmia and tailor treatment to that cause. The emergency physician must familiarize himself with each rhythm and its unique management in any given clinical scenario.

At the end of this chapter, some good internet resources for the ED practitioner to practice interpreting EKGs and cardiac rhythms are provided.

Case Example

A 44-year-old male patient with a history of hypertension and end-stage renal disease on hemodialysis presents with shortness of breath after missing dialysis for 6 days. He reports gradual onset shortness of breath associated with orthopnea and increased lower extremity edema. He denies chest pain or palpitations. He does not have any cough or fever. On physical exam, he is in no distress, afebrile with a heart rate of 60, respiratory rate of 20, blood pressure of 140/78 mmHg, and oxygen saturation of 98% on room air. He has a regular rate and rhythm without murmurs and has crackles bilaterally to the inferior 1/3 of the lung bases and 1+ pitting edema of the bilateral lower extremities.

You decide to get an EKG, which shows the following:

Figure 1 (EKG from http://www.lifeinthefastlane.com)

You send a blood chemistry test, place the patient on a cardiac monitor, and one hour later note the following on the monitor:

Figure 2 - (EKG from liftl.com)

What are the indications for cardiac monitoring in this patient? What EKG abnormalities do you see? What does the rhythm strip show? What is the treatment?

Case Discussion

The ED practitioner should recognize potentially life-threatening conditions that a patient who has missed hemodialysis is at risk for are fluid overload (leading to pulmonary edema) and hyperkalemia. This patient could be considered to meet the Class I monitoring criteria for “needing intensive care” and possibly with “pulmonary edema”; however, even if the patient had no symptoms, the patient is indeed at risk for an acute life-threatening arrhythmia that would necessitate cardiac monitoring.

The EKG demonstrates peaked T waves indicative of acute hyperkalemia. Given the clinical picture of missed dialysis and the peaked Ts on the EKG, the ED physician should immediately initiate treatment for acute hyperkalemia without waiting for a confirmatory blood test (unless immediate point-of-care tests are available). If the patient’s hyperkalemia progressed, the patient could develop QRS widening with the morphology as shown on the rhythm strip called a “sine wave.” This dangerous finding could precipitously deteriorate into a life-threatening arrhythmia such as pulseless v-tach with cardiac arrest and should prompt immediate action. It is important to note that hyperkalemia can manifest in a variety of different EKG findings and does not always follow a consistent pattern from peaked Ts to QRS widening to sine waves; therefore, the patient should be treated at the first indication of any hyperkalemia-related EKG changes.

Conclusions

Cardiac monitoring is an important tool to monitor patients at risk for acute arrhythmias (including those at risk specifically for TdP) and acute or worsening cardiac ischemia. It can be helpful to immediately identify patients with life-threatening arrhythmias who need immediate intervention, to assess the response to medications for arrhythmias, and to help exclude arrhythmias as a likely etiology of a patient’s symptoms (e.g., a patient with syncope) [9]. Given the limited resources and the lack of benefits for many patients, the purpose and duration of cardiac monitoring should be carefully considered. Overuse can not only waste resources but can also contribute to alarm hazards, including “alarm fatigue,” where clinicians are barraged by so many false or nonactionable alarm signals that they become desensitized and do not respond to real events. Therefore, appropriate use and staff education are critical to maximizing the benefits of cardiac monitoring.

Author

Picture of Stacey Chamberlain

Stacey Chamberlain

Dr. Stacey Chamberlain is a board certified emergency physician who is a Professor in the Department of Emergency Medicine at the University of Illinois at Chicago (UIC). She also serves as the Director of the Global Emergency Medicine Fellowship Program and the Co-Director of the Social Emergency Medicine Fellowship Program. In addition to her work in Emergency Medicine, she is the Director of Academic Programs at the UIC Center for Global Health. In this role, she oversees the Global Medicine (GMED) Program for UIC medical students and the graduate global health certificate programs. Dr. Chamberlain has done clinical, educational, public-health, disaster-response, and emergency medicine development work, including working with several globally-focused NGOs, spanning five continents. Her global health work focuses on capacity building in emergency care in Uganda.

Listen to the chapter

2018 version of this topichttps://iem-student.org/cardiac-monitoring/

References

  1. Drew BJ, Califf RM, Funk M, Kaufman ES, Krucoff MW, et al. AHA Scientific Statement:  Practice Standards for Electrocardiographic Monitoring in Hospital Settings. Circulation. 2004; 110: 2721-2746. doi: 10.1161/01.CIR.0000145144.56673.59
  2. Sandau KE, Funk M, Auerbach A, Barsness GW, Blum K, Cvach M, Lampert R, May JL, McDaniel GM, Perez MV, Sendelbach S, Sommargren CE, Wang PJ; American Heart Association Council on Cardiovascular and Stroke Nursing; Council on Clinical Cardiology; and Council on Cardiovascular Disease in the Young. Update to Practice Standards for Electrocardiographic Monitoring in Hospital Settings: A Scientific Statement From the American Heart Association. Circulation. 2017 Nov 7;136(19):e273-e344. doi: 10.1161/CIR.0000000000000527. Epub 2017 Oct 3. PMID: 28974521.
  3. ACLS Training Center. Algorithms for Advanced Cardiac Life Support 2015. Dec 2, 2015.  Accessed at: https://www.acls.net/aclsalg.htm, Dec 10, 2015.
  4. Wellens HJJ. Ventricular tachycardia: diagnosis of broad QRS complex tachycardia. Heart2001;86:579-585 doi:10.1136/heart.86.5.579.
  5. Brugada P, Brugada J, Mont L, Smeets J, Andries EW. A new approach to the differential diagnosis of a regular tachycardia with a wide QRS complex. Circulation. 1991; 83: 1649-1659. doi: 10.1161/01.CIR.83.5.1649
  6. Burns E. VT versus SVT with aberrancy. Life in the Fast Lane. Accessed at: http://lifeinthefastlane.com/ecg-library/basics/vt_vs_svt/, Dec 10, 2015.
  7. Trappe H-J. Concept of the fiveA’s for treating emergency arrhythmias. J Emerg Trauma Shock. 2010 Apr-Jun; 3(2): 129–136. doi:  10.4103/0974-2700.62111
  8. Ramzy M. Duration of Electrocardiographic Monitoring of Emergency Department Patients with Syncope. REBEL EM blog; June 13, 2019; Available at: https://rebelem.com/duration-of-electrocardiographic-monitoring-of-emergency-department-patients-with-syncope/.

Additional Online Resources

Reviewed and Edited By

Picture of Arif Alper Cevik, MD, FEMAT, FIFEM

Arif Alper Cevik, MD, FEMAT, FIFEM

Prof Cevik is an Emergency Medicine academician at United Arab Emirates University, interested in international emergency medicine, emergency medicine education, medical education, point of care ultrasound and trauma. He is the founder and director of the International Emergency Medicine Education Project – iem-student.org, vice-chair of the International Federation for Emergency Medicine (IFEM) core curriculum and education committee and board member of the Asian Society for Emergency Medicine and Emirati Board of Emergency Medicine.

Tachyarrhythmias (2024)

by Keith Sai Kit Leung, Rafaqat Hussain & Abraham Ka Cheung Wai 

You have a new patient!

A 28-year-old female patient presented with 3 weeks history of palpitations. She started with a new-onset shortness of breath and dizziness this morning, which prompted her to attend ED. The patient also complains of recent unintentional weight loss, restlessness, insomnia, passing loose stool more frequently, menstrual disturbances, and some degree of chest pain. No other significant medical history was noted. On physical examination, she looks well-perfused, with bilateral equal air entry and normal vesicular breath sounds throughout, heart sound I+II with no added sound. Vital signs monitoring showed a temperature of 38.1°C, heart rate of 142 bpm, respiratory rate of 21, blood pressure of 155/98, peripheral CRT of 3s, and SpO2 96% on air. ECG is shown below:

What do you need to know?

Tachyarrhythmia is an abnormal heart rate over 100 bpm. It can be classified by site of origin (sinus, supraventricular, ventricular), in relation to QRS complexes (narrow or board-complex), or regularity.

Importance

Tachycardia is an extremely common finding in patients presenting to the emergency department; it involves a wide range of differential diagnoses, from normal variants to physiological responses to life-threatening conditions like shock and cardiac arrest. Studies have shown that patients with tachycardia have an increased risk of post-discharge mortality [1, 2], with higher rates of future re-visit to ED [3].

Epidemiology and Pathophysiology

Sinus tachycardias usually occur as part of a normal physiological response (e.g., exercise, pregnancy) or a compensatory pathological response to secondary underlying conditions (e.g., pulmonary embolism, hyperthyroidism, anemia, infection). It is important to note that sinus tachycardia can be abnormal, secondary to cardiac dysautonomia. These conditions are postural orthostatic tachycardia syndrome or inappropriate sinus tachycardia.

(Reused from Jones, S. A. (2009). ECG Notes: Interpretation and Management Guide. F.A. Davis Company.)

Supraventricular tachycardias is an umbrella term that includes a number of arrhythmias that arise above the bundle of His, i.e., the sinoatrial (SA) node, atria, and atrioventricular (AV) node; these are typically narrow complex tachycardia except WPW syndrome. The most prevalent types of SVTs, in descending order, are atrial fibrillation, atrial flutter, atrioventricular nodal re-entrant tachycardia (AVNRT), atrioventricular re-entrant tachycardia (AVRT), with atrial tachycardia (AT) and junctional tachycardia being the least common types [4, 5]. Three arrhythmogenic mechanisms have been proposed: Re-entry, enhanced automaticity, or triggered activity [6].

Starting with atrial fibrillation (AF) and atrial flutter (AFL), the latest data from the Global Burden of Disease Study 2019 showed that there are 59.7 million affected individuals worldwide [7], with a male predominance in the older population. Common causes of AF include PIRATES [Mnemonic for Pulmonary embolism, Ischaemic heart disease/Idiopathic, Rheumatic valvular disorder, Anaemia/Alcohol, Thyroid (hyperthyroidism), Electrolytes imbalance/Elevated BP (hypertension), Sepsis/Sick sinus syndrome]. The arrhythmogenic mechanism of AF is by increased automaticity, leading to ectopic focal activities and the creation of micro re-entrant circuits in the atrial muscles. Without organized contractility, blood pools in the atria, predisposing to thrombus formation and increasing stroke risk.

(Reused from Jones, S. A. (2009). ECG Notes: Interpretation and Management Guide. F.A. Davis Company.)

Atrial flutter is less common than AF, but they both share similar aetiologies and may coexist. The difference between both is that AF presents with an irregularly irregular heartbeat, while AFL presents with a regularly irregular heartbeat, as a macro re-entrant circuit exists in the atrium, producing a rapid regular atrial rate at 300 bpm. Depending on the conduction ratio, affected patients have a fixed ventricular rate at 150 bpm (2:1), 100 bpm (3:1), or 75 bpm (4:1).

(Reused from Jones, S. A. (2009). ECG Notes: Interpretation and Management Guide. F.A. Davis Company.)

Atrioventricular nodal re-entrant tachycardia (AVNRT) has a prevalence of 2.25 cases per 1000 people in the general population, with a female/male ratio of 2:1 among all age groups [8]. It is the most common cause of paroxysmal SVT and occurs in about 50% of cases. Hence, it is often used synonymously with the term SVT. AVNRT is usually idiopathic, i.e., patients have structurally normal hearts. In AVNRT, re-entry is the main arrhythmogenic mechanism. Naturally, the AVN has dual pathways with different conduction velocities (a fast and slow pathway). Usually, conduction passes via the fast pathway, which blocks incoming current from the slow pathway, while in SVT, the slow pathway becomes the dominant anterograde conduction pathway, uses the fast pathway for retrograde conduction, and creates a re-entrant loop. 90% of AVNRT is slow-fast type [9].

(Reused from Jones, S. A. (2009). ECG Notes: Interpretation and Management Guide. F.A. Davis Company.)

Atrioventricular re-entrant (or reciprocating) tachycardia (AVRT) is another form of paroxysmal SVT, accounting for 30% of cases. It is caused by an anatomical re-entrant circuit with the normal AV conduction system and an AV accessory tract. The most commonly known accessory pathway is called Bundle of Kent, causing Wolff-Parkinson-White (WPW) pre-excitation syndrome; hence, WPW and AVRT are often used interchangeably. It has been estimated to affect 1-3 persons per thousand people. [10]

(Reused from Jones, S. A. (2009). ECG Notes: Interpretation and Management Guide. F.A. Davis Company.)

Atrial tachycardia (AT) accounts for the remaining 10-20% of cases, as opposed to other subtypes; it is usually caused by increased atrial automaticity independent from the AV conduction system or accessory pathways. Other causes include sinoatrial scarring, digoxin toxicity, or conditions that cause atrial dilation (COPD, CHF). Note that there are 2 types of AT, focal and multifocal AT; the former is caused by one ectopic arrhythmogenic focus and later with multiple arrhythmogenic foci within the atria. The firing rate of the ectopic focus is faster than that of the SA node, which overrides its activity. [11]

(Reused from Jones, S. A. (2009). ECG Notes: Interpretation and Management Guide. F.A. Davis Company.)
(Reused from Jones, S. A. (2009). ECG Notes: Interpretation and Management Guide. F.A. Davis Company.)
(Reused from Srinivasan C, Balaji S. Neonatal supraventricular tachycardia. Indian Pacing Electrophysiol J. 2019;19(6):222-231. DOI:10.1016/j.ipej.2019.09.004) – Open Access (https://www.sciencedirect.com/science/article/pii/S0972629219301159)

Junctional tachycardia occurs when there is increased automaticity in the AV node and decreased automaticity in the SA node. This causes ECG changes, which commonly present as retrograde p waves around the QRS complex. [12]

(Reused from Jones, S. A. (2009). ECG Notes: Interpretation and Management Guide. F.A. Davis Company.)

Ventricular arrhythmias are life-threatening conditions that cause sudden cardiac death (SCD); subtypes include monomorphic and polymorphic ventricular tachycardia (VT), Torsades de Pointes (TdP, variant of PVT), ventricular fibrillation (VF). It has been estimated that over 356,000 people suffer from out-of-hospital cardiac arrest in the USA annually, nearly 1000 cases each day [13], and SCD remains the world’s leading cause of death, costing 17 million lives each year [14]. Over the years, VT/VF has decreased incidence; they account for 23% of initial cardiac arrest rhythm, with the most commonly encountered ones being asystole (39%) and PEA (37%). This trend is likely due to the advancement of devices like implantable cardiac defibrillators and improvement in preventative cardiology practice [15]. The most common causes of VT/VF include acute coronary syndrome, cardiomyopathies, congenital channelopathies (BrS, LQTS, CPVT), QT-prolonging drugs (macrolides, TCA), electrolytes imbalance, etc. (Consider 4H 4T causes in cardiac arrest).

(Reused from Jones, S. A. (2009). ECG Notes: Interpretation and Management Guide. F.A. Davis Company.)
(Reused from Jones, S. A. (2009). ECG Notes: Interpretation and Management Guide. F.A. Davis Company.)
(Reused from Jones, S. A. (2009). ECG Notes: Interpretation and Management Guide. F.A. Davis Company.)
(Reused from Jones, S. A. (2009). ECG Notes: Interpretation and Management Guide. F.A. Davis Company.)

The diagram below shows a decision-making algorithm.

(Reused from Srinivasan C, Balaji S. Neonatal supraventricular tachycardia. Indian Pacing Electrophysiol J. 2019;19(6):222-231. DOI:10.1016/j.ipej.2019.09.004) – Open Access (https://www.sciencedirect.com/science/article/pii/S0972629219301159)

Medical History

As tachyarrhythmias present with an extensive list of differential diagnoses, a detailed history taking is essential to direct clinicians to the next-step management. The most common clinical presentations in patients suffering from tachycardias include palpitations (84%), chest pain (47%), dyspnoea (38%), syncope (26%), light-headedness (19%) and sweating (18%) [16]. Symptoms can be explored with a simple mnemonic SOCRATES (site/specify, onset, character/change, rhythm/radiation, associated features, timing, exacerbating and relieving factors, severity). As patients often confuse medical terms with other meanings, it is important to ask and clarify what the term means to them (palpitations vs heart attack). Understanding the onset and progression of symptoms would allow us to determine the acuity and chronicity of the presentation. For timing, we need to ask if the presentation constantly existed since the onset, if it is intermittent, and if it comes on at a particular time of the day. In terms of exacerbating and relieving factors, when it comes to cardiac problems, it is particularly important to ask about the difference between exertion and rest and whether the patient tried anything over the counter. As non-cardiac problems cause tachycardia too, it is necessary to perform a systems review from head to toe to rule out other causes (for example, diarrhea, weight loss, heat intolerance, menstrual disturbance in hyperthyroidism). Past medical and family history should never be missed; these help us to identify risk factors, e.g., hypertension, diabetes, familial hypercholesteremia (predispose to MI), and HOCM (predispose to SCD). In the end, remember to ask for medication history (both prescribed and illicit) and social history (especially smoking and alcohol intake).

If the patient is unconscious, collateral histories from friends and family members are ideal candidates to gain some basic understanding of the patient’s background. It is also worthwhile to communicate with EMTs and paramedics and see if any other valuable information can be obtained.

Adverse features (red flags) for tachyarrhythmias are mainly myocardial infarction, syncope, new-onset heart failure, and deteriorating vital signs, i.e., increased capillary refill time, hypotension (indicative of shock), altered consciousness/reduced GCS.

Physical Examination

If the patient is unconscious or has no palpable pulse, manage the patient with basic life support and advanced life support protocols.

Evaluation of all other patients with the A-E approach is critical as they are still undifferentiated. If the patient is conscious, start inspecting the patient. Key features to observe include cyanosis (poor perfusion peri-arrest), pallor (anemia), dyspnea (heart failure, myocardial infarction/injury), diaphoresis (myocardial infarction/injury), and peripheral edema (heart failure). Start peripherally at hands, observe for clubbing (indicative of infective endocarditis, congenital heart diseases, hyperthyroidism), and assess radial and carotid pulse for its rate, rhythm, and volume. Look for visible jugular venous pulse (elevated – heart failure), presence of corneal arcus (familial hypercholesterolemia) in eyes, and scars on the chest (sternotomy, pacemaker). To assess murmurs, auscultate in all 4 valvular areas (2ndICS left sternal border – pulmonary area, 2nd ICS, right sternal border – aortic area, 4th ICS left sternal border – tricuspid area, 5th ICS mid-clavicular line – mitral area). Be sure to examine other systems, including respiratory, neurological, and ENT.

Alternative Diagnoses

As mentioned above, most tachyarrhythmias are idiopathic or secondary to cardiac and non-cardiac causes. It is extremely important to keep an open mind and an extensive list of differentials so we won’t miss the actual diagnosis. The table below lists differentials for palpitations, the chief complaint of tachyarrhythmias.

Causes of Palpitations

Cardiac Causes

Noncardiac Causes

Atrial fibrillation/flutter

Atrial myxoma

Atrial premature contractions

Atrioventricular reentry

Atrioventricular tachycardia

Autonomic dysfunction

Cardiomyopathy

Long QT syndrome

Multifocal atrial tachycardia

Sick sinus syndrome

Supraventricular tachycardia

Valvular heart disease

Ventricular premature contractions

Ventricular tachycardia

Alcohol

Anemia

Anxiety/stress

Beta-blocker withdrawal

Caffeine

Cocaine

Exercise

Fever

Medications

Nicotine

Paget disease of bone

Pheochromocytoma

Pregnancy

Thyroid dysfunction

(Reuse from Wexler RK, Pleister A, Raman SV. Palpitations: Evaluation in the Primary Care Setting. Am Fam Physician. 2017;96(12):784-789.) – Open Access (https://www.aafp.org/pubs/afp/issues/2017/1215/p784.html)

Acing Diagnostic Testing

Any patients with adverse features and life-threatening presentations should be placed in a resuscitation bay with a multi-parameter vitals monitor/defibrillator connected and a point-of-care portable ultrasound ready. For stable patients, stepwise management should be initiated. Proceed with bedside tests: perform a 12-lead ECG, measure heart rate, assess SpO2 with an oximeter, and record blood pressure. Collect blood samples, including a Full Blood Count, Urea and Electrolytes, serum Magnesium, Calcium, Thyroid Function Tests, Liver Function Tests, and a coagulation panel. Additional tests can be considered based on the clinician’s clinical decision and the patient’s presentation, for example, Troponin for suspected MI, D-dimer for suspected PE, etc. Chest X-rays should be performed in any patients presenting with chest pain. Advanced imaging again depends on clinical presentation, coronary angiogram for Myocardial Infarction, Computed Tomography Pulmonary Angiography for Pulmonary Embolism, etc. The risk stratification tool (more details in the section below) can be used to facilitate decisions for advanced interventions involving intensive care input. Cardiology input will be required for further investigations involving Holter monitoring, implantable loop recorder, electrophysiological study, echocardiogram, cardiac Magnetic Resonance Imaging, etc.

Management

Sinus Tachycardia

Sinus tachycardia is often a physiological response to an underlying cause such as sepsis, hypovolemia, or anemia. Management should focus on identifying and addressing these causes rather than targeting the heart rate itself. For example, in a septic patient, early fluid resuscitation and antibiotics are critical, while in a patient with anemia, blood transfusion or treatment of iron deficiency may resolve the tachycardia. Clinicians should avoid unnecessary use of beta-blockers or calcium channel blockers unless sinus tachycardia persists after the underlying cause has been addressed.

Atrial Fibrillation

Management of atrial fibrillation requires a careful evaluation of the patient’s hemodynamic stability, symptom duration, and underlying comorbidities.

  1. Hemodynamically Stable Patients with Symptoms >48 Hours or Uncertain Timeline:

    • Rate control is the priority to prevent further decompensation. Start with beta-blockers (e.g., bisoprolol) or calcium channel blockers (e.g., diltiazem).
    • Consider digoxin for patients with congestive heart failure who may not tolerate beta-blockers.
    • Avoid cardioversion without anticoagulation if the symptom duration is >48 hours or unclear, as this increases the risk of thromboembolic events.
  2. Hemodynamically Stable Patients with Symptoms <48 Hours or a Reversible Cause:

    • Focus on rhythm control with cardioversion, which can be electrical or pharmacological (e.g., flecainide or amiodarone).
    • Ensure anticoagulation with heparin before cardioversion unless contraindicated.
    • Use an echocardiogram to rule out structural abnormalities, as this guides drug selection (e.g., flecainide for structurally normal hearts; amiodarone for structural heart disease).
  3. Patients with Adverse Features (Shock, Syncope, Acute Heart Failure, or Myocardial Ischemia):

    • Immediate electrical cardioversion is required, typically using synchronized shocks. Time is critical—any delay could worsen outcomes.
  4. Paroxysmal AF:

    • Counsel patients on the use of “pill-in-the-pocket” therapies such as flecainide or sotalol for intermittent symptoms. Ensure they understand the signs of structural heart disease, which would contraindicate these medications.

Always consider underlying conditions such as hyperthyroidism, electrolyte disturbances, or alcohol-related atrial fibrillation (Holiday Heart Syndrome). Addressing these causes can prevent recurrence. In elderly patients or those with heart failure, weigh the benefits of rhythm versus rate control.

Atrial Flutter

Management of atrial flutter parallels that of atrial fibrillation. Rate control is often sufficient in stable patients, but rhythm control may be prioritized for symptomatic relief. In acute settings, electrical cardioversion may be more effective than pharmacological approaches.

Atrial flutter is frequently associated with underlying structural heart disease or atrial enlargement. Evaluate for these conditions with echocardiography and address them to improve long-term outcomes.

AVNRT (Atrioventricular Nodal Reentrant Tachycardia)

AVNRT is often well-managed with non-pharmacological measures in stable patients.

Conservative Management:

  • Initiate vagal maneuvers (e.g., Valsalva maneuver or carotid massage). These can terminate the tachycardia in many cases. Ensure the patient is monitored for safety, especially in older adults where carotid massage could induce complications.

Pharmacological Management:

  • Administer IV adenosine, starting at 6 mg and escalating to 12 mg or 18 mg if needed. Warn the patient about the transient sensation of chest discomfort or flushing.
  • If adenosine is contraindicated (e.g., in asthmatic patients), use a calcium channel blocker such as verapamil.

Persistent Cases:

  • Consider beta-blockers, digoxin, or amiodarone if initial treatments fail.

Hemodynamically Unstable Patients:

  • Proceed with immediate cardioversion to stabilize the patient.

In recurrent AVNRT, evaluate for underlying triggers such as excessive caffeine or stimulant use. Discuss long-term options such as catheter ablation for definitive treatment.

AVRT/WPW (Atrioventricular Reentrant Tachycardia/Wolff-Parkinson-White Syndrome)

In patients with WPW, rapid and accurate diagnosis is critical to avoid inappropriate treatment.

Stable Patients:

  • Treat with amiodarone, flecainide, or procainamide. Avoid digoxin and calcium channel blockers, as these can worsen pre-excitation and lead to ventricular fibrillation.

Unstable Patients:

  • Immediate cardioversion is indicated.

In young patients presenting with sudden palpitations and syncope, always consider WPW and obtain a 12-lead ECG for diagnosis. Educate patients on avoiding stimulants that may precipitate episodes.

Atrial Tachycardia

For atrial tachycardia, management depends on the patient’s stability. Rate control is often effective for stable patients, while cardioversion may be required in unstable cases.

Investigate underlying causes such as digoxin toxicity or structural heart disease, as addressing these may resolve the tachycardia.

Ventricular Tachycardia (VT)

Management of VT hinges on the patient’s hemodynamic stability.

Stable VT:

    • Administer amiodarone (300 mg IV STAT followed by a 900 mg infusion over 24 hours). Monitor for potential side effects such as hypotension or bradycardia.

Unstable VT, pulse positive:

    • Follow the ALS (Advanced Life Support) algorithm, prioritizing cardioversion.

VT, no pulse:

  • Follow the ALS (Advanced Life Support) algorithm, prioritizing defibrillation and CPR.

In patients with recurrent VT, assess for underlying ischemic heart disease or electrolyte abnormalities. Long-term management may require ICD placement or catheter ablation.

Ventricular Fibrillation (VF)

VF is a life-threatening emergency requiring immediate intervention. Follow the ALS algorithm, which includes high-quality CPR and defibrillation.

Always assess for reversible causes of VF, such as acute myocardial infarction or electrolyte imbalances (e.g., hypokalemia or hypomagnesemia), and treat these aggressively to prevent recurrence.

Tachycardia and advanced life support algorithms are provided below.

(Reuse from Soar J, Böttiger BW, Carli P, et al. European Resuscitation Council Guidelines 2021: Adult advanced life support [published correction appears in Resuscitation. 2021 Oct;167:105-106]. Resuscitation. 2021;161:115-151. DOI:10.1016/j.resuscitation.2021.02.010) – Open Access (https://www.cprguidelines.eu/)
(Reuse from Soar J, Böttiger BW, Carli P, et al. European Resuscitation Council Guidelines 2021: Adult advanced life support [published correction appears in Resuscitation. 2021 Oct;167:105-106]. Resuscitation. 2021;161:115-151. DOI:10.1016/j.resuscitation.2021.02.010) – Open Access (https://www.cprguidelines.eu/)

Special Patient Groups

The management of most tachyarrhythmias is similar among pregnant women and pediatric populations, with the exception of ventricular cardiac arrest rhythms.

Pregnant Patients (Obstetric Cardiac Arrest ) [17]

  • A normal supine position will result in aortocaval compression from the gravid uterus; this reduces cardiac output. Hence, placing the patient in a left lateral position is crucial, especially at> 20 weeks gestation.
  • The position of the rescuer’s hands for chest compression ideally should be slightly higher than usual, taking the elevation of the diaphragm and abdominal contents caused by the gravid uterus into account.
  • The defibrillator pad position should be adjusted to maintain the left lateral position.
  • Magnesium sulfate (4 g IV) should be given in patients with eclampsia.
  • Patients should be intubated early due to the higher risk of pulmonary aspiration and Mendelson syndrome from gastric contents.
  • Emergency delivery of the fetus (>20 weeks) with resuscitative hysterotomy should happen within 5 minutes in the event of cardiac arrest, given that the initial resuscitation attempt has failed. This is a definitive procedure to decompress IVC to facilitate venous return and increase cardiac output.
  • As this is an obstetric emergency, get help from the OB/GYN and neonatal team early; resuscitative hysterotomy should not wait even if not all surgical equipment is immediately available; one scalpel is enough to start the procedure.

Pediatrics (Cardiac Arrest) [17]

  • Most pediatric cardiac arrests are secondary to respiratory failure; hence, giving 5 rescue breaths is essential prior to chest compressions.
  • Pulse checks use brachial or femoral pulses as opposed to carotid pulses in adults.
  • It is a similar compression site but with a compression: breath ratio of 15:2, as opposed to 30:2 in adults.
  • In infants, compress the chest using two fingers or an encircling technique (two thumbs). For children over one year old, use one or two hands.
  • Intraosseous (IO) access is preferred for circulation access, as obtaining venous access can be difficult in children.
  • Adrenaline is given in 10 mcg/kg, and Amiodarone is given in 5 mg/kg.
  • Note that PALS is different from newborn life support (NLS), which is not mentioned here.

* Please refer to European Resuscitation Council (ERC) Paediatric Life Support and Special Circumstances Guidelines (https://www.cprguidelines.eu/)

Risk Stratification

There is no single risk stratification tool for tachyarrhythmias, developed scoring systems are usually condition-specific or presentation-specific. We listed some of the important ones related to tachyarrhythmias below:

  • Cardiac Arrest Hospital Prognosis (CAHP) score – predicts prognosis in patients suffering from out-of-hospital cardiac arrest. [18]
  • Cardiac Arrest Risk Triage (CART) Score – predicts the risk of in-hospital cardiac arrest in hospitalized patients. [19]
  • CHA₂DS₂-VASc Score – calculate stroke risk in patients with atrial fibrillation and guide initiation of anticoagulation therapy. [20]
  • HEART score – predicts patients presenting with chest pain for a 6-week risk of major adverse cardiac events (MACE). It can also classify patients into low, moderate, and high-risk groups to facilitate decisions for discharge from ED, admission for observation, or urgent intervention required. [21]
  • Thrombolysis In Myocardial Infarction (TIMI) score and Global Registry of Acute Coronary Events (GRACE) score – estimate mortality for patients with acute coronary syndrome, guide decision for coronary revascularisation needs. [22]
  • Well’s Score – calculate the clinical probability of DVT/PE and guide the decision to consider alternative diagnosis or perform immediate CTPA/anticoagulation. [23]
  • Pulmonary Embolism Rule Out Criteria (PERC) – effectively rules out PE if scored 0. [24]
  • Pulmonary Embolism Severity Index (PESI) – predicts 30-day mortality in patients with PE. [25]
  • Emergency Heart Failure Mortality Risk Grade (EHMRG) estimates 7-day mortality in patients with congestive heart failure and guides the decision to admit them [26].
  • San Francisco syncope rule (SFSR), Canadian syncope risk score (CSRS), and Evaluation of Guidelines in SYncope Study (EGSYS) Score – both SFSR and CSRS predict adverse outcomes in patients presenting with syncope (7-day and 30-day, respectively), EGSYS helps determine whether syncope is cardiac or non-cardiac cause (these includes vasovagal, situational, postural hypotension). [27-29]
  • Multi-parametric models – predict the prognosis of patients with Brugada syndrome for future major arrhythmic events (VT/VF) and guide decisions for implantable cardioverter defibrillator placement. [30]

* Please browse these calculators on MDCalc website (https://www.mdcalc.com/)

When To Admit This Patient

Patients with adverse features and hemodynamic instability require immediate intervention and admission. The aforementioned risk stratification tools can be used based on clinical signs and symptoms. If initial investigations yielded no clinical significance, patients could be discharged with education, reassurance, and safety netting advice. Explain that palpitations are usually transient and harmless; if they are recurring, ask patients to note down the onset, timing, and duration and measure BP and HR if monitoring is available at home. Advise them to reattend ED if symptoms persist or worsen or new-onset red flag symptoms emerge; lifestyle advice, for example, avoid certain known stimulants like caffeine, alcohol, and nicotine. If the patient is known to have SVT, educate about self-performing Valsalva maneuver to try terminating it before medical assistance arrives. Arrange follow-up with a family physician and review the need for further investigations and specialist input. Patients should be referred urgently for detailed investigations, including Holter monitoring if non-specific new clinical findings are yielded. Other options may be explored, such as an echocardiogram, implantable loop recorder, and electrophysiology study. [31, 32]

Revisiting Your Patient

The case reminds us that tachyarrhythmias can be secondary to non-cardiac causes. This is a classical presentation of hyperthyroidism, with ECG showing fast-rate atrial fibrillation. Atrial fibrillation occurs in 15% of patients with hyperthyroidism. A detailed history taking with appropriate systems review (as symptoms suggest) would point us towards hyperthyroidism. Clinical examination may reveal clubbing (thyroid acropachy), exomphalos (thyroid eye disease), pretibial myxoedema, goiter, and an irregular heartbeat with mid-systolic scratchy murmur (Means–Lerman scratch) might be heard on auscultation. The investigation here, starting from bedside, would be to obtain a complete set of vital signs (blood pressure, heart rate, respiratory rate, temperature, SpO2), 3-lead continuous monitoring, and 12-lead ECG; blood including complete blood count, urea and electrolytes, thyroid function tests, troponin (serial), venous blood gas, other electrolytes (Ca2+). The management approach of this patient is to treat the underlying hyperthyroidism primarily. Hence, endocrinologist referral will be required, with cardiologists’ input on managing the fast Atrial Fibrillation. Propranolol (reduces peripheral conversion of T4 to T3) and anti-thyroid drugs like carbimazole (inhibits thyroid peroxidase action) are the mainstay management (details see thyroid disorder chapter). However, as the patient also complains of anginal pain, rate control with cardio-selective beta-blockers should be initiated as well. It also helps with alleviating symptoms of hyperthyroidism, including palpitations, tremors, anxiety, heat intolerance, etc., due to the increased sympathetic tone caused by excess thyroid hormone production. The need for anticoagulation is assessed on an individual basis. In most cases, Atrial Fibrillation reverses to sinus rhythm spontaneously after the euthyroid state has been achieved. However, if Atrial Fibrillation persists, cardioversion may be considered. This, nevertheless, would be a cardiologist’s decision. [33]

Authors

Picture of Keith Sai Kit Leung

Keith Sai Kit Leung

Keith is an academic foundation doctor (emergency medicine themed) in the UK. He graduated both BSc and MBChB with distinction, and has published over 30 peer-reviewed articles till date. He is interested in Pre-Hospital Emergency & Retrieval Medicine, Intensive Care, Cardiology and Medical Education. He main research interests are arrhythmias and cardiac electrophysiology, cardiac arrest and resuscitation, ACS, POCUS, ECMO, airway and trauma management. He aims to work as an academic PHEM/HEMS physician and pursue a MD (Res)/PhD in the near future.

Picture of Rafaqat Hussain

Rafaqat Hussain

Dr Rafaqat Hussain is working as Specialty Doctor in Emergency Department at SWBH NHS trust. He had done MBBS.MRCEM. FRCEM.EBCEM. He has involved in training and teaching for junior doctors and medical students at University of Birmingham. He is enthusiastic in pursuing his career in being an Emergency Medicine Consultant.

Picture of Abraham Ka Cheung Wai

Abraham Ka Cheung Wai

Dr Abraham Wai, Clinical Associate Professor at the University of Hong Kong (HKU), is a dynamic force in the field of emergency medicine. His journey from specialist training to impactful research and innovative teaching has left an indelible mark on the healthcare landscape.

Listen to the chapter

References

  1. Pinto DS, Ho KK, Zimetbaum PJ, Pedan A, Goldberger AL. Sinus versus nonsinus tachycardia in the emergency department: importance of age and heart rate. BMC Cardiovasc Disord. 2003;3:7. doi:10.1186/1471-2261-3-7
  2. Gabayan GZ, Sun BC, Asch SM, et al. Qualitative factors in patients who die shortly after emergency department discharge. Acad Emerg Med. 2013;20(8):778-785. doi:10.1111/acem.12181
  3. Wilson PM, Florin TA, Huang G, Fenchel M, Mittiga MR. Is Tachycardia at Discharge From the Pediatric Emergency Department a Cause for Concern? A Nonconcurrent Cohort Study. Ann Emerg Med. 2017;70(3):268-276.e2. doi:10.1016/j.annemergmed.2016.12.010
  4. Sohinki D, Obel OA. Current trends in supraventricular tachycardia management. Ochsner J. 2014;14(4):586-595.
  5. Kotadia ID, Williams SE, O’Neill M. Supraventricular tachycardia: An overview of diagnosis and management. Clin Med (Lond). 2020;20(1):43-47. doi:10.7861/clinmed.cme.20.1.3
  6. Tse G. Mechanisms of cardiac arrhythmias. J Arrhythm. 2016;32(2):75-81. doi:10.1016/j.joa.2015.11.003
  7. Li H, Song X, Liang Y, et al. Global, regional, and national burden of disease study of atrial fibrillation/flutter, 1990-2019: results from a global burden of disease study, 2019. BMC Public Health. 2022;22(1):2015. doi:10.1186/s12889-022-14403-2
  8. Orejarena LA, Vidaillet H Jr, DeStefano F, et al. Paroxysmal supraventricular tachycardia in the general population. J Am Coll Cardiol. 1998;31(1):150-157. doi:10.1016/s0735-1097(97)00422-1
  9. George SA, Faye NR, Murillo-Berlioz A, Lee KB, Trachiotis GD, Efimov IR. At the Atrioventricular Crossroads: Dual Pathway Electrophysiology in the Atrioventricular Node and its Underlying Heterogeneities. Arrhythm Electrophysiol Rev. 2017;6(4):179-185. doi:10.15420/aer.2017.30.1
  10. Chhabra L, Goyal A, Benham MD. Wolff Parkinson White Syndrome. [Updated 2023 Jan 28]. In: StatPearls [Internet]. Treasure Island (FL): StatPearls Publishing; 2023 Jan-. Available from: https://www.ncbi.nlm.nih.gov/books/NBK554437/
  11. Liwanag M, Willoughby C. Atrial Tachycardia. [Updated 2022 Jun 27]. In: StatPearls [Internet]. Treasure Island (FL): StatPearls Publishing; 2023 Jan-. Available from: https://www.ncbi.nlm.nih.gov/books/NBK542235/
  12. Hafeez Y, Grossman SA. Junctional Rhythm. [Updated 2023 Feb 5]. In: StatPearls [Internet]. Treasure Island (FL): StatPearls Publishing; 2023 Jan-. Available from: https://www.ncbi.nlm.nih.gov/books/NBK507715/
  13. Srinivasan NT, Schilling RJ. Sudden Cardiac Death and Arrhythmias. Arrhythm Electrophysiol Rev. 2018;7(2):111-117. doi:10.15420/aer.2018:15:2
  14. Tsao CW, Aday AW, Almarzooq ZI, et al. Heart Disease and Stroke Statistics-2022 Update: A Report From the American Heart Association [published correction appears in Circulation. 2022 Sep 6;146(10):e141]. Circulation. 2022;145(8):e153-e639. doi:10.1161/CIR.0000000000001052
  15. Keller SP, Halperin HR. Cardiac arrest: the changing incidence of ventricular fibrillation. Curr Treat Options Cardiovasc Med. 2015;17(7):392. doi:10.1007/s11936-015-0392-z
  16. Yetkin E, Ozturk S, Cuglan B, Turhan H. Clinical presentation of paroxysmal supraventricular tachycardia: evaluation of usual and unusual symptoms. Cardiovasc Endocrinol Metab. 2020;9(4):153-158. doi:10.1097/XCE.0000000000000208
  17. Maupain C, Bougouin W, Lamhaut L, et al. The CAHP (Cardiac Arrest Hospital Prognosis) score: a tool for risk stratification after out-of-hospital cardiac arrest. Eur Heart J. 2016;37(42):3222-3228. doi:10.1093/eurheartj/ehv556
  18. Banerjee, A., & Oliver, C. (2017). Revision notes for the FRCEM intermediate SAQ paper (2nd). Oxford University Press.
  19. Churpek MM, Yuen TC, Park SY, Meltzer DO, Hall JB, Edelson DP. Derivation of a cardiac arrest prediction model using ward vital signs. Crit Care Med. 2012;40(7):2102-2108. doi:10.1097/CCM.0b013e318250aa5a
  20. Lip GY, Nieuwlaat R, Pisters R, Lane DA, Crijns HJ. Refining clinical risk stratification for predicting stroke and thromboembolism in atrial fibrillation using a novel risk factor-based approach: the euro heart survey on atrial fibrillation. Chest. 2010;137(2):263-272. doi:10.1378/chest.09-1584
  21. Brady W, de Souza K. The HEART score: A guide to its application in the emergency department. Turk J Emerg Med. 2018;18(2):47-51. doi:10.1016/j.tjem.2018.04.004
  22. de Araújo Gonçalves P, Ferreira J, Aguiar C, Seabra-Gomes R. TIMI, PURSUIT, and GRACE risk scores: sustained prognostic value and interaction with revascularization in NSTE-ACS. Eur Heart J. 2005;26(9):865-872. doi:10.1093/eurheartj/ehi187
  23. Wells PS, Anderson DR, Rodger M, et al. Excluding pulmonary embolism at the bedside without diagnostic imaging: management of patients with suspected pulmonary embolism presenting to the emergency department by using a simple clinical model and d-dimer. Ann Intern Med. 2001;135(2):98-107. doi:10.7326/0003-4819-135-2-200107170-00010
  24. Freund Y, Cachanado M, Aubry A, et al. Effect of the Pulmonary Embolism Rule-Out Criteria on Subsequent Thromboembolic Events Among Low-Risk Emergency Department Patients: The PROPER Randomized Clinical Trial. JAMA. 2018;319(6):559-566. doi:10.1001/jama.2017.21904
  25. Aujesky D, Obrosky DS, Stone RA, et al. Derivation and validation of a prognostic model for pulmonary embolism. Am J Respir Crit Care Med. 2005;172(8):1041-1046. doi:10.1164/rccm.200506-862OC
  26. Lee DS, Lee JS, Schull MJ, et al. Prospective Validation of the Emergency Heart Failure Mortality Risk Grade for Acute Heart Failure. Circulation. 2019;139(9):1146-1156. doi:10.1161/CIRCULATIONAHA.118.035509
  27. Quinn J, McDermott D, Stiell I, Kohn M, Wells G. Prospective validation of the San Francisco Syncope Rule to predict patients with serious outcomes. Ann Emerg Med. 2006;47(5):448-454. doi:10.1016/j.annemergmed.2005.11.019
  28. Thiruganasambandamoorthy V, Kwong K, Wells GA, et al. Development of the Canadian Syncope Risk Score to predict serious adverse events after emergency department assessment of syncope. CMAJ. 2016;188(12):E289-E298. doi:10.1503/cmaj.151469
  29. Kariman H, Harati S, Safari S, Baratloo A, Pishgahi M. Validation of EGSYS Score in Prediction of Cardiogenic Syncope. Emerg Med Int. 2015;2015:515370. doi:10.1155/2015/515370
  30. Chung CT, Bazoukis G, Radford D, et al. Predictive risk models for forecasting arrhythmic outcomes in Brugada syndrome: A focused review. J Electrocardiol. 2022;72:28-34. doi:10.1016/j.jelectrocard.2022.02.009
  31. Moulton KP, Bhutta BS, Mullin JC. Evaluation Of Suspected Cardiac Arrhythmia. [Updated 2023 Feb 9]. In: StatPearls [Internet]. Treasure Island (FL): StatPearls Publishing; 2023 Jan-. Available from: https://www.ncbi.nlm.nih.gov/books/NBK585054/
  32. RCEMLearning. https://www.rcemlearning.co.uk/reference/palpitations/. Published July 27, 2021. Accessed April 13, 2023.
  33. Parmar MS. Thyrotoxic atrial fibrillation. MedGenMed. 2005;7(1):74.

Reviewed and Edited By

Picture of Arif Alper Cevik, MD, FEMAT, FIFEM

Arif Alper Cevik, MD, FEMAT, FIFEM

Prof Cevik is an Emergency Medicine academician at United Arab Emirates University, interested in international emergency medicine, emergency medicine education, medical education, point of care ultrasound and trauma. He is the founder and director of the International Emergency Medicine Education Project – iem-student.org, chair of the International Federation for Emergency Medicine (IFEM) core curriculum and education committee and board member of the Asian Society for Emergency Medicine and Emirati Board of Emergency Medicine.

Fundamentals of ACLS (2024)

by Mohammad Anzal Rehman

You have a new patient!

A 56-year-old man presents to the Emergency Department with complaints of chest pain and dizziness that began an hour ago. Upon assessment by the triage nurse, his vital signs are as follows: his heart rate is 107 beats per minute, and his respiratory rate is 22 breaths per minute. His blood pressure is  96/70 mmHg, and his oxygen saturation is at 90% on room air. His temperature is 36.8°C.

You are the student on shift when this patient arrives, and immediately, your mind begins to jump across differential diagnoses for this patient. As you rush toward the patient’s room to join your senior, you prepare to list out all the potential causes of chest pain proudly. This must be a Myocardial Infarction, or maybe even an Aortic Dissection. Perhaps it is that rare Boerhaave syndrome you read about last night!

You finally catch up to the Emergency Physician, but before you can open your mouth to wax lyrical about esophageal ruptures, the Doctor states “Let’s begin by evaluating the ABCs.”

Initial Assessment

Emergency Medicine is one of the few specialties in medicine where patient evaluation begins in the same way for every patient, regardless of the probable diagnosis. Most clinicians are wired to jump straight to the ‘mystery-solving’ component of clinical presentation, with many undergraduate curriculums based around disease recognition. Emergency Medicine, however, places an emphasis on systematic assessment of the patient, starting with ‘The Primary Survey’.

The Primary Survey – ABCDE Approach

The Primary Survey aims to identify life-threatening conditions rapidly and systematically in critically ill patients, with appropriate stabilizing interventions performed when an abnormality is recognized. Besides streamlining the process in a high-stakes and often chaotic environment, the alphabetical order is designed first to address the most severe causes of mortality [1].

The Primary Survey aims to identify life-threatening conditions rapidly and systematically in critically ill patients, with appropriate stabilizing interventions performed when an abnormality is recognized. Besides streamlining the process in a high-stakes and often chaotic environment, the alphabetical order is designed first to address the most severe causes of mortality [1].

Airway

A patient’s airway connects air, and therefore oxygen, from outside the body to the lungs. Airway management is a term used to evaluate and optimize the passage of oxygen in the upper airway, which may be impaired when there is a blockage or narrowing of this pathway. The most common cause of upper airway obstruction is the tongue, which may ‘close’ the oropharynx posteriorly in patients who are comatose or in cardiopulmonary arrest, for example.

Assessment of the airway typically starts by evaluating any external features that may impact the passage of air through the naso- and oro-pharynx, such as facial or neck trauma, fractures, deformities, and any masses or swelling that may disrupt the airway tract. Allergies, especially anaphylaxis, and significant burns may cause edema of the laryngeal airway and produce obstruction. Excessive secretions may also congest the oropharynx and produce airway obstruction.

A patent or ‘normal’ airway allows a responsive patient to speak in full sentences without difficulty, implying a non-obstructed air passage down the oropharynx and through the vocal cords.

Clinical signs of obstruction may include stridor, gurgling, drooling, choking, gagging, or apnea. A physician may also identify an impending airway obstruction where loss of gag reflex, intractable vomiting, or worsening laryngeal edema may inevitably compromise the passage of air to the lungs and produce a failure to oxygenate or ventilate, prompting a decision to secure the tract through intubation.

Management

In the responsive patient, allow for the patient to be seated or lying in their most comfortable position as you assess the patency of the airway.

‘Opening’ the airway involves positioning the patient’s head in the ‘sniffing position’. In this position, a slight extension of the head with flexion of the neck, keeping the external auditory meatus in line with or above the sternal notch, is used to optimally align the pharyngeal and laryngeal airway segments, preventing obstruction posteriorly by the tongue (Figure 1). This is useful in patients who are unresponsive and cannot consciously protect their airway.

Figure 1 – Use of ‘sniffing position’ to open the airway

Two maneuvers are helpful in opening an unresponsive or sedated patient’s airway, optimizing air entry to the lungs:

1. Head tilt chin lift (Figure 2A) – Using fingertips under the chin, lift the mandible anteriorly while simultaneously tilting the head back using the other hand. Do not use this if cervical spine injury is suspected!

Figure 2A – Head-tilt chin lift

2. Jaw thrust (Figure 2B) – With thenar eminences of both hands anchored over both maxillary regions of the patient’s face, use your fingers at both angles of the mandible to lift it anteriorly. This maneuver is preferable in cases of suspected cervical spine injury as it does not cause hyperextension of the neck.

In unresponsive patients with excessive secretions, use of a rigid suction device can clear fluid and particulate matter such as vomitus.

Intubation may be performed if airway assessment deems it necessary to protect or secure the airway tract in a definitive way. If intubation is required, it should be performed as early as possible to prevent the evolution of a difficult airway, which would lower the chances of a successful intubation. It may also be useful to establish the risk of an inherently difficult airway using the L-E-M-O-N airway assessment method as below:

Look externally – facial trauma, large incisors and/or tongue, hairy beard, or moustache

Evaluate the 3-3-2 rule – where optimal distance between incisors on mouth opening should be 3 finger breadths. Similarly, 3 finger breadths (patient’s fingers) should span the distance from chin to hyoid bone, while the distance from hyoid to thyroid should measure 2 finger breadths.

Mallampati score – grades the view of an open mouth, with class 3 or more predicting a difficult intubation

Obesity/obstruction – Epiglottitis or a tonsillar abscess can inhibit easy passage of an endotracheal tube.

Neck mobility – if limited, positioning is difficult and causes suboptimal views during intubation.

iEM-infographic-pearls-airway - Assessing Airway Difficulty
assessing airway difficulty

Cervical spine immobilization

When the patient arrives in the Emergency Department (ED) following a significant physical trauma, such as head injury or motor vehicle collision, it is crucial to consider the integrity of the cervical spine. If injury is present in this region, further manipulation or movement of the neck may lead to spinal cord damage. Therefore, evaluation and management of airway for these patients should go hand in hand with cervical spine immobilization.

If no specialized equipment is available, or until one is prepared for use, attempts to limit neck movement can be done using manual in-line stabilization, where the provider’s forearms or hands may be positioned at the sides of the patient’s head to prevent indirect movements that could exacerbate underlying injury (see Figure 3).

Cervical spine immobilization is then performed using a rigid cervical collar. It may be augmented with head blocks on lateral sides to limit movement further as the patient is evaluated for injury (see Figure 4). The thoracolumbar region of the spine is immobilized using a spinal backboard, which keeps the patient in a supine position with minimal external force on the spine. Frequently utilized in Emergency Medical Services (EMS) during extrication and transport, all efforts should be made to transition the patient off the spinal board in the ED as it is quite uncomfortable, with prolonged use associated with pressure ulcers and pain.

Breathing

The lungs perform the vital function of delivering oxygen from the airway to the alveoli through ventilation. Perfusion at the alveoli allows for gas exchange; therefore, effective ventilation and perfusion both play a key role in the availability and utilization of oxygen by the human body. Evaluation of the Breathing component assesses factors that would indicate a compromise in ventilation.

The chest inspection should look for respiratory rate, use of accessory muscles, position of trachea (midline versus deviated), symmetry of chest rise, and/or any visible trauma to the thorax. Auscultation evaluates breath sounds for any bilateral inequal air entry or presence of crackles, crepitus, or wheeze. Percussion, though sometimes useful, is often difficult to perform adequately in a resuscitation environment.

Let’s compare the findings in normal lungs, pleural effusion, and pneumothorax based on chest rise, trachea position, percussion, and auscultation.

Normal Lungs: Chest rise is symmetrical with the trachea in the midline position. Percussion reveals a resonant sound. Auscultation presents vesicular breath sounds peripherally and bronchovesicular sounds over the sternum, with no added sounds.

Pleural Effusion: Chest rise remains symmetrical, and the trachea is midline. Percussion is dull over the area of effusion, and auscultation shows decreased breath sounds in the region of the effusion.

Pneumothorax: Chest rise is unequal, and the trachea may be deviated in cases of tension pneumothorax. Percussion reveals a hyper-resonant sound in the area of the pneumothorax, and auscultation shows decreased breath sounds over the pneumothorax region.

Measuring oxygen saturation using pulse oximetry (spO2) provides a percentage of oxygen in circulating blood, with normal levels typically at 95% or above. However, in patients with chronic lung disease, baseline oxygen saturation levels may decrease and can be as low as 88% in many cases. For patients experiencing shortness of breath and showing signs of hypoxia, pulse oximetry readings below 94% suggest that supplemental oxygen may be necessary. This can be administered through various oxygen delivery systems, as outlined in Figure 5 and described below.

Figure 5 – Common equipment used in airway management 1- Nasal cannula, 2- Simple face mask, 3- Nebulizer,* 4- Non-rebreather mask, 5- Venturi mask valves, 6- Rigid suction tip, 7- Bag-valve mask device, 8- Oropharyngeal airway (OPA), 9- Nasopharyngeal airway (NPA), 10- Direct Laryngoscope, 11- Endotracheal tube with stylet, 12- Colorimetric end-tidal CO2 detector, 13- Bougie, 14- Laryngeal Mask Airway (LMA) *NOT an oxygen delivery device, used to administer inhaled medication such as bronchodilators and steroids CO2: Carbon dioxide

General concepts—We typically breathe in room air, which contains 21% oxygen. Each Liter per minute of supplemental oxygen provides an additional 4% inspired oxygen (FiO2) to the patient.

Nasal cannula – Administered through patient nostrils, can provide a maximum flow rate of 4-6 Liters per minute of oxygen, which equals roughly 37 – 45% FiO2

Simple face mask – Applied over the patient’s nose and mouth, can provide a maximum flow rate of 6-10 Liters per minute of oxygen, which equals roughly 40 – 60% FiO2

Venturi mask – Typically used in COPD, where over-oxygenation is avoided. Different colors deliver various flow rates to limit oxygen delivery to the required amount only; Blue (2-4L/min, FiO224%), White (4-6L/min, FiO2 28%), Yellow (8-10L/min, FiO235%), Red (10-12 L/min, FiO2 40%), Green (12-15 L/min, FiO260%)

Non-rebreather mask – Utilizes an attached bag with a reservoir of oxygenated air along with one-way valves on the mask to prevent rebreathing of expired air, optimizing oxygenation. It can provide a maximum flow rate of 15 Liters per minute of oxygen, which equals roughly 85 – 90% FiO2.

Non-invasive ventilation (CPAP/BiPAP) is a tight-fitting mask device that uses high positive pressure to keep the airway open and enhance oxygenation. It is particularly useful in conditions such as COPD exacerbation, acute pulmonary edema/heart failure, and sleep apnea.

Bag-valve mask device: A self–inflating bag attached to a reservoir delivers maximal, high-flow 100% oxygen. This method of manual ventilation is used in rescue breathing and oxygen delivery in nonresponsive or cardiopulmonary arrest patients.

Circulation

The circulation component of the Primary Survey evaluates the adequacy of perfusion by the cardiovascular system. The patient’s general appearance is assessed for signs of pallor, mottling, diaphoresis, or cyanosis, which indicate inadequate or deteriorating perfusion status. Pulses are checked centrally (e.g. carotid pulse, especially if patient with impaired breathing) and peripherally (e.g. radial) alongside hemodynamic assessment, including blood pressure and heart rate checks. Information from this segment also provides valuable insight into potential signs of shock. Extremities are palpated in order to determine any delays in capillary refill time (more than 2 seconds signifies inadequate perfusion, e.g. hypovolemia), peripheral edema in lower extremities (signs of heart failure), and skin temperature (cool or warm to touch).

In cases of trauma, systematic evaluation of circulation also seeks to ascertain areas of potential blood loss or collection, with interventions for any long-bone deformities and/or bleeding from open wounds performed as they are discovered.
Intravenous (IV) line insertion is also performed as part of the management of circulation, as any required fluid or blood products can then be administered through a large-bore IV line (16 gauge or higher). If IV insertion is difficult on multiple attempts, when volume resuscitation is urgently required, Intraosseous (IO) access should be sought to prevent delay in any needed treatment. Insertion of a peripheral venous line often occurs concomitant to blood extraction for any urgent laboratory investigations and/or point-of-care testing. Some common examples of tests performed on critically ill patients include venous blood gas, complete blood count, type and crossmatch, troponin, urea, electrolytes, and creatinine.

Finally, circulation assessment requires an evaluation of cardiac rhythm. Basic auscultation may reveal the rate and regularity of rhythm along with murmurs. However, a critically ill patient will also benefit from the immediate attachment of cardiac pads to the bare chest and connection to a cardiac monitoring device, which provides the physician with the patient’s current cardiac rhythm.

A normal sinus rhythm (Figure 6) consists of a P wave (atrial depolarization), followed by a QRS wave (ventricular depolarization – normally less than 120 ms), with a subsequent T wave (ventricular repolarization). P-R intervals typically have a duration of 120 – 200 ms. A regular rhythm, with a consistent P wave preceding QRS complexes, with a normal heart rate (between 60 – 100 beats per minute (bpm)) is required to consider a rhythm to be normal sinus on the cardiac monitor.

Figure 6 – Normal sinus rhythm

The American Heart Association’s (AHA) Advanced Cardiac Life Support (ACLS) course and guidelines outline a series of internationally recognized cardiac rhythms and their general management when encountered [2]. Some of the most important rhythms, along with the AHA bradycardia and tachycardia algorithms, are summarized below:

Figure 7.1 - Sinus bradycardia (HR < 50 bpm)

Several different conditions, including abnormal heart conduction, damage to the myocardium, metabolic disturbances, or hypoxia, can cause bradycardia. A lower heart rate can result in decreased perfusion to end-organs, such as the brain, with resultant signs and symptoms such as dizziness, confusion, shortness of breath or chest pains. Management (Figure 7.2) aims to treat the underlying cause and increase the heart rate (atropine, dopamine/epinephrine and/or cardiac pacing) if needed to restore the heart’s ability to perfuse organs adequately.

Figure 7.2 – American Heart Association’s Bradycardia Algorithm

Tachycardia (Figure 8.1) is a heart rate of more than 100 bpm that may present as several types of waveforms on the cardiac monitor. Supraventricular tachycardia (SVT) originates in the upper chambers of the heart. The rapid heart rate prevents adequate filling of the heart between contractions, causing signs and symptoms such as dizziness, palpitations, or chest pain.

Figure 8.1 - Supraventricular Tachycardia (SVT)

Management (Figure 8.2) typically involves Valsalva maneuvers, medication (e.g. adenosine), and/or synchronized cardioversion as needed to revert the rhythm back to baseline.

Figure 8.2 – American Heart Association’s Tachycardia Algorithm

SVT produces a narrow-complex tachycardia (QRS segments < 120 ms). In comparison, monomorphic Ventricular Tachycardia (Figure 8.3) originates in the lower chambers of the heart and produces a wide-complex (QRS segments > 120 ms) tachycardia on the cardiac monitor. Similarly, this rhythm may cause dizziness, shortness of breath, or chest pain and is managed with medication or synchronized cardioversion.

Figure 8.3 - Ventricular Tachycardia

ACLS algorithms often divide patients based on “stable” and “unstable” categories. This grouping aims to ascertain which individuals have a pathology severe enough to impair cardiac output to the point of causing serious inadequacies in end-organ perfusion. This ‘instability’ is manifested by altered mental status, ischemic chest pain, drastically low hemodynamic parameters (e.g. systolic BP < 90 mmHg), signs of shock, and signs of acute decompensated heart failure.

Disability

This segment evaluates the level of consciousness and responsiveness of the patient. Level of consciousness may be assessed generally using the AVPU scale (below);

Alert: fully alert patient
Verbal: some form of verbal response is present, though not necessarily coherent.
Pain: response to painful stimulus
Unresponsive: no evidence of motor, verbal or eye-opening response to pain

or more explicitly, using the Glasgow Coma Scale (GCS)

Choose the best response of patient
EYE OPENING
4: Spontaneously
3: To verbal command
2: To pain
1: No response
BEST VERBAL RESPONSE
5: Oriented and converses
4: Disoriented and converses
3: Inappropriate words; cries
2: Incomprehensible sounds
1: No response
BEST MOTOR RESPONSE
6: Obeys command
5: Localizes pain
4: Flexion withdrawal
3: Flexion abnormal (decorticate)
2: Extension (decerebrate)
1: No response
Glasgow Coma Score (GCS) (Modified from Teasdale, G., & Jennett, B. (1974). Assessment of coma and impaired consciousness: a practical scale. The Lancet, 304(7872), 81-84.) - Please read this article to get more insight regarding GCS.

Exposure

Complete exposure of the patient may be necessary to completely evaluate for any external signs of infection, injury, and rash. This is especially useful in trauma, where log-rolling of the patient is included to ensure the back and spine are also included in a complete assessment for any traumatic injuries. As you expose the patient, obtain consent, be mindful of their dignity, and uncover each segment of the body sequentially, covering it back to prevent any hypothermia for the patient. A core temperature reading also completes vital sign measurements for the patient.

Practical implementation of the Primary Survey

The “cursory” primary survey

It may seem surprising to consider that virtually every patient who enters the Emergency Department, despite the severity of the illness, undergoes some form of a Primary Survey by the treating physician. However, the practicality of this becomes quite obvious when you consider a simple question frequently asked at the beginning of a patient encounter:

“How are you?”

An adequate response of “I am all right” or “Well, I have had this pain in my stomach…” seems fairly standard, but it addresses most of the components detailed in the previous section. A patient who can form words without difficulty or added sounds generally has an intact or patent Airway. Their ability to form words depends on air that has been sufficiently ventilated and moving through the vocal cords, hence the Breathing is adequate. An appropriate response to the question allows us to assume that Circulation adequately perfuses the brain to allow comprehension and formulation of new words oriented to the circumstances of the encounter, hence providing insight into Circulation and, to a degree, Disability.

Synchrony in the Emergency Department

Although systematic assessment during the Primary Survey is laid out in order, it is also important to note that an Emergency Department consists of teams of healthcare professionals who often have the personnel and resources to simultaneously perform tasks to efficiently address all components of the Primary assessment, without delay between segments.
In practice, an example of how synchrony works would involve a patient who, on initial, immediate assessment, is deemed to be in significant distress and/or critically ill. The patient is immediately moved into the ED to a resuscitation area, where team members expose the chest, attach cardiac pads to connect the patient to a cardiac monitor, obtain a fresh set of vital signs, including spO2monitoring, with IV cannula insertion, blood extraction for testing as needed. At the same time, a primary survey is conducted simultaneously by another physician who moves through Airway, Breathing, Circulation, Disability and Exposure. In more advanced systems, a member may be dedicated to each component of the Primary Survey.

Adjuncts

A number of resources are accessible to the Emergency Physician that may aid in diagnosing and investigating the critically ill patient. Utilizing these alongside the initial Primary Survey provides valuable, relevant information that can further guide clinical decision-making and diagnosis during evaluation.

  1. Electrocardiogram – A 12-lead electrocardiogram provides a complete picture of the heart’s electrical activity in various vectors and segments, allowing for a more accurate evaluation for rhythm disturbances, such as in acute myocardial infarction, hyperkalemia, bundle branch blocks, and torsade de pointes. This often ties into the Circulation assessment and allows for a more comprehensive look into the heart’s electrophysiology.
  2. Portable X-rays – Particularly in trauma, urgent chest and pelvic X-ray films can often be obtained without having to transfer the patient to Radiology, hence providing more information on suspected lung pathologies (e.g. pneumothorax, effusion/hemothorax) and pelvic abnormalities (e.g. fracture, displacement).
  3. Urinary/ gastric catheters – Urinary catheters are useful to evaluate fluid status and monitor output for the patient undergoing volume resuscitation. When relevant, gastric tube insertion can assist in gastrointestinal decompression, if needed, as well as minimize the risk of aspiration in certain patients.
  4. Point-of-Care Ultrasonography (PoCUS) – A rapidly evolving and increasingly prevalent modality in the ED is the ultrasound.[3] Various probes, at different frequencies, utilize ultrasound waves to provide the physician with real-time visualization of the body’s internal structures. These images are fast and often very reliable in determining major findings that can guide decision-making in critically ill patients (e.g. presence of post-traumatic intra-abdominal free fluid, pneumothorax, cardiac tamponade). Figure outlines some examples of information that can be extracted using PoCUS.

 

HI-MAP in Shock

Reassessment

Each intervention performed in the Primary Survey should ideally be accompanied by a reassessment of vital signs and patient clinical status and a restarted Primary Survey beginning from Airway. Identifying any improvements, deteriorations, or non-responses that will be pivotal in guiding the initiation or discontinuation of further intervention as per the clinical case is crucial.

Focused History and Secondary Survey

If the patient is appropriately evaluated and stabilized following the Primary Survey, the treating physician may proceed with a focused history and secondary survey appropriate to the clinical circumstances. One example of a focused history incorporates the mnemonic SAMPLE to organize pertinent information as follows:

S – Signs/symptoms of presenting complaint

A – Allergies to any food or drugs

M – Medications (current, recent changes)

P – Pertinent past medical history

L – Last oral intake

E – Events leading to the illness or injury

A secondary survey in the Emergency Department is a more comprehensive physical examination performed systematically in a head-to-toe fashion to investigate any clinically relevant findings. In case of trauma, this also involves careful inspection for any missed injuries, deformities, or signs of underlying blood collection.

As the secondary survey is performed, relevant investigations and/or imaging may be ordered to augment the evaluation of the present clinical condition (e.g. Computerized Tomography (CT) of the brain after signs of basal skull fracture noted on inspection of the face and head). Information gathered from the survey and results of any ordered investigations, coupled with the clinical condition and/or response to therapy in the ED, if any, is used to determine patient disposition at the end of the ED encounter.

Revisiting Your Patient

You assist the Emergency Physician in performing a Primary Survey. The airway is patent, with the patient phonating in full sentences and breathing with mild tachypnea but no added sounds on auscultation. You initiate supplemental oxygen through a non-rebreather mask, with an increase in spO2 to 99%. You reassess and proceed through Airway, Breathing, and Circulation. As you discuss initiating IV fluids with your senior, the patient complains of worsening chest pain, palpitations, and dizziness.You attach the patient to the cardiac monitor and notice the rhythm below:

Cardiac pads have already been attached to the patient. Noting the presence of ischemic chest pain, you correctly identify the patient as having an unstable, narrow-complex tachycardia, most likely an SVT and prepare for synchronized cardioversion. Conscious sedation is conducted after explaining the procedure and obtaining consent from the patient. 50 joules of biphasic energy is then administered for synchronized electrical cardioversion. The rhythm changes on the monitor to the reading below:

You observe an organized rhythm but note that the patient is now unresponsive, with eyes closed and no palpable carotid pulse.

Basic Life Support

Cardiopulmonary arrest occurs when the heart suddenly stops functioning, resulting in lack of blood flow to vital organs in the body, such as the lungs and brain. Therefore, signs of arrest are manifested as a lack of breathing (apnea), lack of pulse and unresponsiveness. The most common cause of cardiac arrest is coronary artery disease.[4] Respiratory arrest refers to a cessation of lung activity, but with a present, palpable pulse and functioning heart.
The International Liaison Committee on Resuscitation (ILCOR) and the American Heart Association (AHA) are some of the key figures who have developed international guidelines on the recognition and management of cardiac arrest patients.[5] Basic Life Support (BLS) and Advanced Cardiac Life Support (ACLS) courses were established to optimize the workflow and, therefore, patient outcomes in Cardiopulmonary Resuscitation (CPR).

CPR forms the cornerstone of BLS to effectively maintain the victim’s circulatory and ventilatory function until circulation either spontaneously returns or is hopefully restored through intervention. The general concepts within BLS are outlined below:

1. A person who has a witnessed collapse, lack of response or who is suspected of being unresponsive due to cardiac arrest should be approached for further assessment and management. However, it is important for the rescuer to first determine whether the scene is safe around the patient before attempting any intervention. An example of this would be a victim drowned in water, who should be removed from the body of water onto a dry surface prior to attempting life-saving chest compressions or defibrillation.

Figure 9 - Witness
Figure 10 - Check for responsiveness

2. Check for responsiveness. Firmly tapping both shoulders with the palms of your hands and a clear, verbal prompt, such as “Hey, are you okay?” should be incorporated to ensure that the victim is, indeed, unresponsive to an otherwise arousable stimulus.

3. You have determined that the patient is unresponsive. If you are alone, shout loudly and clearly for help and assistance. If no help is nearby, call Emergency Medical Services using your mobile phone.

Figure 11 - Call for help
Figure 12 - Open airway, palpate carotid artery, observe the chest

4. Open the patient’s airway (tilt chin upward into sniffing position). Palpate the carotid pulse by placing two fingers (index and middle finger) just lateral to the trachea on the side closest to you while simultaneously observing the chest for any spontaneous chest rise (breathing). The pulse check should take a minimum of five (5) seconds but no more than 10 seconds to avoid delay in life-saving intervention.

5. When help is available, the chain of survival begins by activating the Emergency Response System. In addition to activating the Emergency Response, ask the person who has responded to your call for help getting an Automated External Defibrillator (AED) device. An example of instruction to a bystander (out of hospital) would be to ‘call an ambulance and get an AED!’. Inside a hospital, if another healthcare provider has come to aid, you may ask them to ‘activate the Emergency Response System/’Code Blue’ and get the crash cart/AED.’

6. Begin high-quality chest compressions. Hands are placed with fingers interlaced to exert pressure using the heel of one hand at the center of the chest, over the lower half of the breastbone (sternum), in line with the nipples (in men), with shoulders directly over your hands and arms straight at a perpendicular angle to the victim’s chest. High-quality chest compression is one of the few variables which have been evidenced to improve patient survival in cardiac arrest.

Figure 13 - Chest compression

Keep the following features in mind to maintain high-quality chest compressions:

  • More than 80% of the time in resuscitation or more should be spent on compressions (Chest compression fraction of > 80%)
  • The frequency of compressions should follow a rate of 100–120 compressions per minute.
  • Compression depth in adults is at least 2 inches. In infants and children, depth should be at least one-third of the anterior-posterior diameter of the chest.
  • After each compression, the hands should be withdrawn to allow adequate chest recoil and fill the heart between compressions.
  • Minimize interruptions in chest compression
  • Avoid hyperventilation (see next point).
Figure 14 - Bag-Valve-Mask Ventilation. Two-Hand technique

7. Compressions should follow the ratio 30:2, that is, 30 compressions followed by 2 rescue breaths delivered by a mouth barrier device (pocket mask) in the sniffing position or a Bag-valve mask (BVM) device if another rescuer is present to manage the airway in hospital. The BVM’s mask should be held with a tight seal using the E-C technique over the bridge of the nose and covering the mouth. 

Breaths should be over 1 second, with enough air pushed in to observe a chest rise and no hyperventilation or excessive bagging of the BVM to avoid gastric insufflation. Two attempts at rescue breaths are performed, minimizing time to under 10 seconds and resuming chest compressions immediately after. If a definitive airway (e.g. endotracheal tube) is in place, resume compressions without pause at a rate of 100-120 compressions per minute while breaths are delivered once every 6 seconds.

8. Once an AED or cardiac monitor/defibrillator is available, place the pads on the victim’s bare chest (dry the skin if wet) in either an anterior-lateral or anterior-posterior position.When in doubt, follow the machine’s prompts and the instructions on the pads themselves to guide placement.

Figure 15 - Correct placement of transcutaneous pacing pads.jpg

9. Follow the prompts on the AED. Stop compressions when the device analyzes rhythm and stay clear of the patient (not touching any part of the patient’s body). During an in-hospital resuscitation, as per ACLS workflow, stay clear, as the team leader should analyze the initial rhythm to ascertain the presence of a shockable or non-shockable rhythm. Either way, the device or team leader should prompt whether a shock is advised. Continue compressions as the device charges, but ensure that all rescuers are clear of the patient when the shock is delivered using the AED/defibrillator device.

Figure 16 - Shock delivery.

A victim who is unresponsive but has a palpable pulse has respiratory arrest, which is managed using rescue breathing only. Breaths are delivered once every 6 seconds without chest compressions while transport to a higher level of care and/or management of any underlying cause for the condition is initiated.

Advanced Cardiac Life Support

The Advanced Cardiac Life Support algorithms were designed to deliver a higher level of resuscitative care where providers with increased training and improved resources are available. This type of augmented management is customary to the Emergency Department, where a Rapid Response Team or Code Blue team would respond when activated and initiate a more team-based approach to cardiopulmonary resuscitation.

Instead of an AED, in-hospital settings have a cardiac monitor/defibrillator, usually mounted atop a crash cart consisting of a CPR back-board (to support chest compressions by providing a firm surface to use under the patient’s chest), drawers with medication used during cardiac arrest, and various equipment for airway management and IV/IO access. Once brought to the bedside, the cardiac pads are similarly placed on the patient’s chest while BLS maneuvers (chest compressions and rescue breaths) continue. Once placed, however, compressions should be paused to assess the cardiac monitor’s cardiac rhythm. The type of rhythm should be identified asshockableornon-shockable(Figure 17s).

Figure 17.1 - NON-SHOCKABLE - Asystol
Figure 17.2 - NON-SHOCKABLE - Pulseless electrical activity – organized rhythm in the absence of palpable pulse
Figure 17.3 - SHOCKABLE - Pulseless Ventricular Tachycardia
Figure 17.4 - SHOCKABLE - Ventricular fibrillation

“Shockable” rhythms (pulseless Ventricular Tachycardia and Ventricular Fibrillation) are a product of aberrant electrical conduction of the heart. Rapid, early correction of this rhythm is the most important step in returning the body to its normal circulatory function. Early defibrillation is one of the few variables that has been evidenced to improve patient survival in cardiac arrest, the other notable one being high-quality chest compressions.[6]

Defibrillation involves using an asynchronous 200J of biphasic (360J if monophasic) energy, delivering an electric current through the cardiac pads attached to the patient’s chest to revert the heart to a rhythm that can sustain spontaneous circulation. Chest compressions should be ongoing while charging, but all persons should stay clear of the patient when shock is being delivered, and this is frequently verified with verbal feedback (‘Clear!’) before pressing the defibrillator button to deliver the shock. Immediately after the shock, chest compressions should resume to minimize interruptions between compressions.

Two minutes of chest compressions and rescue breaths make up each cycle of CPR, at the end of which a rhythm check should be performed for any changes and/or presence of pulse. Figure 18 outlines the ACLS algorithm used to manage shockable and non-shockable rhythms in cardiac arrest. Early shock in shockable rhythms is followed by a cycle of CPR, a second shock if still with a shockable rhythm, after which 1mg of IV epinephrine is given, with subsequent doses every 3 to 5 minutes. During the third cycle of CPR, after 3 shocks have been delivered for a persistent shockable rhythm, a bolus of IV Amiodarone 300mg is typically administered, with a dose of 150mg in a subsequent CPR cycle if still with a shockable rhythm.

“Non-shockable” rhythms (pulseless electrical activity (PEA) and asystole) are not typically a product of disorganized electrical activity in the heart. Instead, an underlying cause has resulted in cardiac arrest for these patients. While the majority of cardiac arrest is caused by coronary artery disease, the consideration of reversible causes by use of the H’s (hypovolemia, hypoxia, hyper-/hypokalemia, hydrogen ions (acidosis), and hypothermia) and T’s (thrombosis/embolism, toxins, tension pneumothorax, and cardiac tamponade) may help recognize and manage other possible etiologies in patients.

The management of non-shockable rhythms focuses on consistent, high-quality CPR, with regular pulse checks every 2 minutes, addressing reversible causes, and administering IV epinephrine 1mg every 3 to 5 minutes.
A palpable pulse with measurable blood pressure signals the Return of Spontaneous Circulation (ROSC).

Figure 18 - ACLS Adult Cardiac Arrest Algorithm

Resuscitation Team Dynamics

The Emergency Department is equipped with the resources and personnel to provide care beyond basic life support. Resuscitation is optimized when multiple providers work together to effectively perform tasks toward management of the patient, thereby multiplying the chances of a successful outcome for the patient. A high-performance team typically consists of members allocated to the following roles and responsibilities:

  • Airway – Opens and maintains the airway. Manages suctioning, oxygenation, and ventilation (Bag-valve mask) and assesses the need for a definitive airway if needed.
  • Medication – Inserts and maintains IV/IO access. Manages medication administration and fluids.
  • Monitor/defibrillator – Ensures attached cardiac pads and AED/cardiac monitor/defibrillator device are working appropriately to display the patient’s cardiac rhythm in clear view of the team leader. Administers shocks using the devices as needed. May alternate with the compressor every 5 cycles or 2 minutes to prevent compression fatigue
  • Compressor – Performance of high-quality chest compressions as part of CPR for the cardiac arrest patient. Focuses on quality and consistency of compressions. You may switch to another standby compressor or monitor/defibrillator every 5 cycles or 2 minutes if compressions are affected by fatigue.
  • Recorder – Documents the timing of medication, intervention (shocks, compression), and communicates these to the Team Leader, with prompts to enable timely dosing of frequent medication (e.g., ensuring epinephrine every 3 to 5 minutes is administered as per the verbalized order)
  • Team leader – A defined leader who coordinates the team’s efforts and organizes them into roles and responsibilities that are clear, well-understood, and within their individual limitations. Provides explicit instructions and direction to the resuscitation effort, focused on patient care and optimized performance from all team members. Promotes understanding and motivates members, identifying any potential deficit or depreciation of quality during resuscitation and facilitating improvement in performance as needed.

All team members are encouraged to conduct themselves with mutual respect and practice closed-loop communication, where each message or order is received with verbal confirmation of understanding, then execution of the order, centralizing all information back to the team leader. Figure 19 provides an example of the possible placement of each member during resuscitation that may optimize their workflow through the resuscitation attempt. Ideally, the team leader remains at the foot of the bed, in clear view of all members, with involvement limited to coordination of the team’s efforts and minimal direct execution of tasks.

Figure 19 - An example of optimized team placement during resuscitation

Post Arrest Care

If the patient is found to have Return of Spontaneous Circulation (ROSC), post-cardiac arrest care should be initiated to enhance the preservation of brain tissue and heart function. This involves a sequential assessment and optimization of Airway, Breathing, and Circulation in the initial stabilization phase. A definitive airway may be placed so ventilation is more appropriately controlled, with parameters set to optimize oxygen administered with ventilatory function. Figure 20 outlines the ACLS algorithm and parameters often used to help guide post-cardiac arrest care. Circulation incorporates fluids, vasopressors, and/or blood products to achieve an adequate systolic blood pressure above 90 mmHg, with Mean Arterial Pressure of at least 65 mmHg typically indicating perfusion within stable parameters.

It is imperative to obtain a 12-lead ECG early to ascertain the presence of an ST-elevation myocardial infarction (STEMI), which will require expedited transfer of the patient to a Cath Lab for definitive reperfusion therapy. The patient’s responsiveness should be reassessed, and the determination for additional investigation should be performed in conjunction with other critical care management as needed.

Of note, unresponsive patients may benefit from Targeted Temperature Management (TTM), which involves the maintenance of core body temperature at a target of 32 – 36 ℃ for 24 hours, or preferably normothermia at 36 °C to 37.5 °C with an emphasis on prevention of hyperthermia, in order to protect and optimize brain recovery post-arrest.[7]

Almost all cardiac arrest survivors will require a period of intensive care observation and management. If no immediate intervention is needed (e.g., reperfusion therapy), patients inside a hospital will need to be transitioned to an Intensive Care Unit (ICU) for further care.

Figure 20 – Post-Cardiac Arrest Care

What do you need to know?

  • Emergency Medicine, especially in critical care, emphasizes a systematic approach to the unwell patient.
  • The Primary Survey is designed to recognize and address life-threatening conditions effectively and timely.
  • The Primary Survey components are Airway (& and C-spine in trauma), Breathing, Circulation, Disability, and Exposure.
  • If an intervention is performed at any level of the survey, you must reassess the patient by commencing the Primary Survey again, starting with Airway.
  • Reassess and review your patient for changes frequently.
  • Many of the actions performed in the initial assessment of the critically ill patient may occur simultaneously when more team members are present in an Emergency Department. Do not let the chaos of the scene distract you from completing each step of the assessment.
  • The AHA has well-established guidelines for assessing and managing patients through the Primary Survey. Use the algorithms and the patient’s status as ‘stable’ or ‘unstable’ to guide the management of recognized pathologies, especially in Circulation.
  • The ED is home to a variety of adjuncts, including portable X-rays, ECG, and point-of-care ultrasound, which can provide the physician with rapid, readily accessible information to guide management.
  • Remember the SAMPLE mnemonic for a focused history in the critically ill patient.
  • An unresponsive patient should be immediately recognized, and Emergency Response Systems should be activated.
  • Performance of Basic and Advanced cardiac life support focuses on preserving blood circulation transiently to maintain the perfusion of organs, such as the brain, until the cause of the condition is reversed or managed.
  • The majority of cardiac arrest is caused due to coronary artery disease.
  • The two most important predictors of patient survival in cardiac arrest are high-quality CPR and early defibrillation (for a shockable rhythm)
  • An effective resuscitation in the ED often relies on the concerted efforts of multiple team members, led by a team leader who coordinates tasks in an organized, effective way to improve patient survival and outcomes.

Author

Picture of Mohammad Anzal Rehman

Mohammad Anzal Rehman

EM Residency Graduate from Zayed Military Hospital in Abu Dhabi, UAE. Founder/President of the Emirates Collaboration of Residents in Emergency Medicine (ECREM). Editor-in-Chief for the Emirates Society of Emergency Medicine (ESEM) Monthly Newsletter. I have a vested interest in sharing updated knowledge and developing teaching tools. As a healthcare professional, I continually strive to incorporate the newest clinical research into practice and am an active advocate for the use of Point of Care Ultrasonography (POCUS) in the ED.

Listen to the chapter

References

  1. Reynolds T. Basic Emergency Care: Approach to the Acutely Ill and Injured. World Health Organization; 2018.
  2. 2020 Advanced Cardiac Life Support (ACLS) Provider Manual. American Heart Association; 2021.
  3. Hashim A, Tahir MJ, Ullah I, Asghar MS, Siddiqi H, Yousaf Z. The utility of point of care ultrasonography (POCUS). Ann Med Surg (Lond). 2021;71:102982. Published 2021 Nov 2. doi:10.1016/j.amsu.2021.102982
  4. Cardiac Arrest Registry to Enhance Survival (CARES) 2022 Annual Report; 2022, https://mycares.net/
  5. Wyckoff MH, Singletary EM, Soar J, et al. 2021 International Consensus on Cardiopulmonary Resuscitation and Emergency Cardiovascular Care Science With Treatment Recommendations: Summary From the Basic Life Support; Advanced Life Support; Neonatal Life Support; Education, Implementation, and Teams; First Aid Task Forces; and the COVID-19 Working Group. Resuscitation. 2021;169:229-311. doi:10.1016/j.resuscitation.2021.10.040
  6. Soar J, Böttiger BW, Carli P, et al. European Resuscitation Council Guidelines 2021: Adult advanced life support [published correction appears in Resuscitation. 2021 Oct;167:105-106]. Resuscitation. 2021;161:115-151. doi:10.1016/j.resuscitation.2021.02.010
  7. Lüsebrink E, Binzenhöfer L, Kellnar A, et al. Targeted Temperature Management in Postresuscitation Care After Incorporating Results of the TTM2 Trial. J Am Heart Assoc. 2022;11(21):e026539. doi:10.1161/JAHA.122.026539

Acknowledgements

  • Marina Margiotta – Illustrator
  • Paddy Kilian – Emergency Physician – Mediclinic City Hospital, Dubai, Director of Academic Affairs – Mohammed Bin Rashid University Of Medicine and Health Sciences
  • Rasha Buhumaid – Consultant Emergency Physician – Mediclinic Parkview Hospital, Dubai, Assistant Professor of Emergency Medicine – Mohammed Bin Rashid University Of Medicine and Health Sciences, President of the Emirates Society of Emergency Medicine (ESEM)
  • Amog Prakash – Medical Student – Mohammed Bin Rashid University Of Medicine and Health Sciences
  • Fatima Al Hammadi- Medical Student – Mohammed Bin Rashid University Of Medicine and Health Sciences

Reviewed By

Picture of Arif Alper Cevik, MD, FEMAT, FIFEM

Arif Alper Cevik, MD, FEMAT, FIFEM

Prof Cevik is an Emergency Medicine academician at United Arab Emirates University, interested in international emergency medicine, emergency medicine education, medical education, point of care ultrasound and trauma. He is the founder and director of the International Emergency Medicine Education Project – iem-student.org, chair of the International Federation for Emergency Medicine (IFEM) core curriculum and education committee and board member of the Asian Society for Emergency Medicine and Emirati Board of Emergency Medicine.

Chest Trauma (2024)

CHEST TRAUMA

by Ivan Low & Jeremy Wee Choon Peng

You have a new patient!

A fifty-year-old male patient is brought into the resuscitation room following a motor vehicle accident. The patient is a motorcyclist who was hit on his left side by a lorry moving at 40mph (64.4 kph). He was flung 3 metres, and his helmet remained intact, while the motorcycle was badly damaged. Vital signs are as follows: T 36.5, BP 168/98, HR 112, RR 32, and SpO2 95% on 10L facemask. A cervical collar has been applied by paramedics on scene. On arrival, the patient appears to be in respiratory distress, using his accessory muscles of respiration.

What do you need to know?

Chest trauma occurs in approximately one-third [1] of trauma patients and causes up to 25% mortality [2] in a multiply injured patient. More than 80% of thoracic injuries were blunt [1] rather than penetrating, and 70% of blunt injuries were a result of road traffic accidents [2,3,4], with drivers and front-seat passengers being placed at highest risk [2].

Thoracic injuries can lead to clinical deterioration and death by impairing a patient’s ‘ABCs.
Airway obstruction can directly result from distortion of tracheobronchial anatomy. Breathing can be affected in two broad ways: 1) Ventilation-perfusion (V/Q) mismatch: Hypoventilation can result from mechanical chest wall or diaphragmatic disruptions, or abnormal collections of air or fluid in the pleural space limiting lung expansion. Hypoperfusion can be caused by vascular disruptions or thrombosis in a damaged lung. 2) Gas exchange abnormalities: Gas exchange can be impaired by direct damage to the alveolocapillary membrane, or dilution of the surfactant layer with blood or interstitial fluid causing alveolar collapse. Circulation can be compromised due to: hemorrhagic shock from vascular or parenchymal injuries, cardiogenic shock from cardiac injury, and obstructive shock from high intrathoracic or intrapericardial pressures (see Table 1). 
 
Table 1: Pathophysiology and patterns of injury in chest trauma

Pathophysiology

Example of injuries (non-exhaustive)

Airway

Tracheobronchial injury

Breathing

(a) V/Q mismatch

 

 

 

(b) Gas exchange abnormalities

 

Sternal or rib fractures +/– flail chest

Pneumothorax +/– hemothorax

Diaphragmatic injury

Pulmonary vascular injury or thrombosis

Pulmonary contusion or laceration

Circulation

(a) Hemorrhagic shock

(b) Cardiogenic shock

(c) Obstructive shock

 

Great vessel (e.g. aortic) disruption

Cardiac contusion or laceration

Tension pneumothorax

Cardiac tamponade

 

Medical History

The mnemonic “AMPLE” highlights several critical aspects of a patient’s history that should be gathered from all trauma patients: Allergies, Medications currently used, Past illnesses and pregnancy, Last meal, and Events related to the injury [5]. Alert patients can often provide details about the mechanism of their injury and the level of force or impact experienced. They may also report symptoms such as pain in the chest, back, or abdomen, dyspnoea, noisy breathing, hemoptysis, hematemesis, syncope or pre-syncope. Eyewitness accounts from passers-by and paramedics may be helpful in supplementing an overall picture of the scene, particularly if the patient is unable to provide a history at this point in time. The ATMIST template, which includes age, time, mechanism, injuries, signs and symptoms, and treatment, is an effective tool for guiding handovers from prehospital teams [6].

Physical Examination

A comprehensive physical examination is crucial to detect major thoracic injuries early to facilitate timely intervention. The patient’s vital signs often help identify patients who are decompensating or at higher risk of deterioration. These include abnormalities such as tachycardia, bradycardia, tachypnea, bradypnea, hypotension, and hypoxemia.

Inspection

On inspection, assess the patient for:

(a) Airway obstruction, respiratory distress, or decompensation:

  • Stridor
  • Use of accessory muscles
  • Asterixis, drowsiness, or obtundation

(b) Chest wall injuries:

  • Steering wheel or seatbelt imprints
  • Chest wall contusions, deformities, or wounds
  • Asymmetric and paradoxical chest wall movements
  • For penetrating injuries, always check for exit wounds, and assume involvement of adjacent body cavities e.g. abdomen

(c) Peripheral signs of shock or major thoracic injury:

  • Cold and clammy peripheries (most types of shock)
  • Jugular venous distension (tension pneumothorax or cardiac tamponade)

Palpation

On palpation, assess the patient for:

(a) Pneumothorax, hemothorax, or lung collapse:

  • Tracheal deviation
  • Subcutaneous emphysema

(b) Rib fractures and associated complications:

  • Bony tenderness
  • Step deformities
  • Flail segments

(c) Pulses:

  • Irregular heart rhythm (arrhythmia from cardiac injury)
  • Pulse delay or differential (aortic dissection)

Percussion

On percussion, assess the patient for:

(a) Hyperresonance: pneumothorax

(b) Dullness: hemothorax, lung collapse

Auscultation

On auscultation, assess the patient for:

(a) Stridor or wheeze: airway obstruction

(b) Reduced breath sounds: pneumothorax or hemothorax

(c) Muffled heart sounds or pericardial rub: pericardial effusion

As patients with thoracic injuries often present with concomitant abdominal and shoulder injuries, complete your examination with a thorough evaluation of the abdomen and shoulder girdles (including the clavicle and scapula).

Alternative diagnoses

The important injuries to assess all patients with chest trauma are summarised in literature as the ‘Deadly Dozen’, as listed in Table 2 [7]. The first six (‘Lethal Six’) are immediately life-threatening and ought to be detected during the primary survey, while the next six (‘Hidden Six’) may not be immediately apparent but are potentially life-threatening and should be picked up during the secondary survey.

Table 2: ‘Deadly Dozen’ in Thoracic Trauma

‘Lethal Six’

‘Hidden Six’

1. Airway obstruction

1. Thoracic aortic disruption

2. Tension pneumothorax

2. Tracheobronchial disruption

3. Cardiac tamponade

3. Myocardial contusion

4. Open pneumothorax

4. Traumatic diaphragmatic tear

5. Massive hemothorax

5. Esophageal disruption

6. Flail chest

6. Pulmonary contusion

Diagnostic testing

Apart from the ‘Lethal Six’ pathologies, early diagnostic testing can aid in the detection of life-threatening pathologies, including but not limited to the ‘Hidden Six’. A combination of point-of-care tests (POCT), chest imaging, and laboratory tests can help physicians make critical decisions in the assessment of a patient with chest trauma.

POCT

  • Arterial or venous blood gas (ABG/VBG):
    • When to do it: Signs and symptoms of respiratory distress or decompensation, or when there is a clinical suspicion of shock.
    • Findings to look for: A low PaO2:FiO2 (P/F) ratio indicates degree of respiratory failure; an elevated PaCO2level suggests hypoventilation and respiratory decompensation; an elevated serum lactate level indicates poor end-organ perfusion and shock.
  • Electrocardiogram (ECG)
    • When to do it: Mechanism suggests major chest trauma, signs and symptoms of chest wall injury, or features of active ongoing cardiac disease [8].
    • Findings to look for: Arrhythmias and ischemic changes may suggest cardiac injury; small or alternating QRS complexes may indicate pericardial effusion.
  • Point-of-care ultrasonography (POCUS):
    • When to do it: Mechanism suggests major polytrauma or chest trauma, hemodynamic instability[9,10]
    • Findings to look for: Reduced cardiac contractility or wall rupture, pericardial effusion, absence of lung sliding or presence of lung point, sternal or rib fractures [11]
    • Pitfalls: Due to poor sensitivity and operator dependence, normal findings do not exclude cardiac, pulmonary, aortic, or musculoskeletal injury [9,10]; serial assessments are recommended to improve sensitivity.

Imaging

Table 3: NEXUS Chest Radiography Rule [12]

This applies to patients 15 years or older who sustained blunt trauma within the last 24 hours. No thoracic imaging is required if none of the following criteria are met:

  • Age >60 years
  • Rapid deceleration mechanism (fall from >20ft, MVC >40mph)
  • Chest pain
  • Intoxication
  • Altered mental status
  • Distracting painful injury
  • Tenderness to chest wall palpation
  • Computer tomography of thorax or aortogram (CT Thorax / Aortogram):
    • When to do it: Mechanism suggests major polytrauma or chest trauma [9,10]; suspicion of thoracic injury based on clinical findings or CXR / POCUS; unable to exclude significant thoracic injury by NEXUS Chest CT Decision Instrument (Table 4) [14].
    • Findings to look for: Cardiac, pulmonary, aortic, or musculoskeletal injuries.
    • Pitfalls: Risk-benefit ratio needs to be assessed and possibly discussed with the patient and/or their next-of-kin in patients at risk of contrast-induced nephropathy or who are pregnant.

Table 4: NEXUS Chest CT Decision Instrument [14]

This applies to patients with stable vital signs, 15 years or older. No chest CT is required if none of the following criteria are met:

  • Chest wall, sternum, thoracic spine, or scapular tenderness
  • Abnormal chest X-ray (e.g. clavicle fracture or widened mediastinum)
  • Distracting injury
  • Rapid deceleration mechanism (fall from >20ft, MVC >40mph)

Laboratory tests

  • Cardiac enzymes:
    • When to do it: Hemodynamic instability, ECG abnormalities
    • Pitfalls: Prognostic yield remains unclear

Risk stratification

The two useful clinical tools in the emergency department are the NEXUS rules above, which both have 99% sensitivity in excluding significant thoracic injuries in the absence of all criteria, and hence may guide the judicious use of chest imaging.

There are several risk stratification tools developed to predict poorer outcomes (e.g. mortality, pneumonia, acute respiratory distress syndrome) in patients with chest trauma, including the Thoracic Trauma Severity Score (TTSS) [15,16], Chest Trauma Score (CTS)[17,18], and Rib Fracture Score (RFS)[19]. However, none are routinely used in the emergency department setting given that they do not affect clinical management in the ED and typically require advanced imaging as part of the risk stratification process.

Management

Patients with chest trauma are managed according to Advanced Trauma Life Support (ATLS) [20] principles in order to detect and address ‘ABCDE’ issues in a timely fashion.

(a) Airway

  • Establish definitive airway (intubation, front-of-neck access) if partial or complete airway obstruction is identified.

(b) Breathing

  • Administer 100% O2 if hypoxemic.
  • Apply one-way seal to open chest wounds.
  • Intubate for mechanical ventilation if there is respiratory failure (oxygenation, ventilation).
  • Finger and tube thoracostomy if there is suspicion of pneumothorax or hemothorax.

(c) Circulation

  • Administer IV crystalloids if there are signs of shock and end-organ hypoperfusion.
  • Transfuse blood products and consider activating the massive transfusion protocol if suspecting hemorrhagic shock.
  • Consider pericardiocentesis if in cardiac tamponade and no surgeon is readily available.
  • If the patient goes into traumatic circulatory arrest, there may be a role for emergency department thoracotomy (EDT), unless (1) there were no signs of life on scene, (2) the injuries are unsurvivable, or (3) prolonged downtime exceeding 15 minutes.

(d) Disability

  • Consider early intubation if the patient is drowsy (which could be a result of shock, hypercarbia, hypoxemia, or concomitant head injury).
  • Check for concomitant thoracic spine injury.

(e) Environment / Exposure

  • Assess for injuries across the chest, back, and adjacent areas such as the shoulder girdles and abdomen.
  • Keep the patient warm.

Urgent surgical consultation is often warranted in all cases of penetrating chest trauma and if there is suspicion of any of the ‘Deadly Dozen’ pathologies.

Medications

  • Analgesia is crucial for relieving pain and preventing atelectasis, but when using opioids, careful dosing is necessary to avoid hypoventilation.
  • Tranexamic acid stabilises blood clots and reduces mortality in patients with significant ongoing bleeding presenting within 3 hours of injury [21].
  • Prophylactic antibiotics and the tetanus vaccine are used to prevent infection, particularly in patients with penetrating chest trauma, significant hollow viscus injuries, or those undergoing invasive procedures. The use of these medications should follow institutional guidelines.

Special patient groups

  • Paediatric patients: Children have more compliant chest walls and hence may sustain significant intrathoracic injuries without obvious chest wall abnormalities. In addition, multiple chest wall injuries or a mechanism of injury that is incompatible with the child’s developmental milestones should raise suspicion for non-accidental injury. A careful evaluation and a high index of suspicion are required in this patient group.
  • Geriatric patients: Older adults, due to reduced pulmonary reserves, are at an increased risk of developing respiratory complications. This patient group should be counselled on the increased risks of atelectasis, pneumonia, and acute respiratory distress syndrome. Adequate analgesia and incentive spirometry may help to reduce the risk of these complications.

When to admit the patient

  • All patients with penetrating chest trauma or blunt trauma with suspicion of significant intrathoracic injury should be admitted for evaluation and monitoring.
  • Patients who remain symptomatic despite a normal initial evaluation can be observed for 8 hours, with serial physical examinations or chest radiographs scheduled.
  • Most other patients can be safely discharged with appropriate advice (to return to the ED if developing pain, dyspnoea, fever, haemoptysis, or hematemesis, which may suggest missed injuries or complications such as chest infection).

Revisiting your patient

The patient is managed according to ATLS [20] principles.

Airway: There is no stridor or obvious facial injury. The cervical spine has been immobilised.

Breathing: He is tachypnoeic and hypoxaemic. The trachea is deviated to the right. There is a left chest wall deformity with bruising, asymmetric chest wall movements, and left-sided paradoxical movements. There is no obvious penetrating injury. Crepitus and step deformities are palpated on the left chest wall. Breath sounds are reduced on the left. Tension pneumothorax and flail chest are suspected. POCUS demonstrates absent lung sliding on the left. An arterial blood gas demonstrates hypoventilation with a PaCO2 of 42 mm Hg despite the patient’s tachypnoea (PaCO2 is expected to be low). Left finger thoracostomy is performed, with a gush of air noted upon entry into the pleural cavity, and a large-bore chest tube is inserted. Intubation with manual in-line stabilisation is performed in view of impending type 2 respiratory failure.

Circulation: The patient is tachycardic and has borderline low blood pressure. His peripheries are cold and clammy. Pulses are regular and there is no pulse delay or differential. An initial FAST scan is negative for free fluid in the abdomen or pericardial sac. Heart sounds are normal. Two large-bore intravenous cannulas are inserted for crystalloid administration.

The rest of the primary survey and adjuncts: The rest of the primary survey is normal. A plain chest radiograph confirms the presence of a left pneumothorax with subcutaneous emphysema and multiple left-sided rib fractures constituting a flail segment (see example videos below). There is no obvious hemothorax, widened mediastinum, or raised hemidiaphragm. Analgesia, prophylactic antibiotics, and the tetanus vaccine are administered.

Disposition: The patient is admitted to the surgical intensive care unit and planned for further advanced imaging and operative interventions by the surgical team.

Authors

Picture of Ivan Low

Ivan Low

Ivan Low is a Resident in Emergency Medicine in Singapore's public healthcare system. He graduated from the National University of Singapore (NUS) in 2018, and is currently working as a military doctor in the Republic of Singapore Navy. He is a Designated Workplace Doctor (Compressed Air Works) and is trained in Diving and Hyperbaric Medicine. He is actively involved in medical education, particularly in the Singapore Medical Association. He has been awarded the NUS Junior Doctor Teaching Award several times in recognition of his work.

Picture of Jeremy Wee Choon Peng

Jeremy Wee Choon Peng

Dr Jeremy Wee serves as a Senior Consultant at the Department of Emergency Medicine at the Singapore General Hospital. Besides being actively involved in the care of patients at the Emergency Department, Dr Wee has a special interest in Trauma care as well as Medical Education. This led to his completion of the Masters of Science in Health Professions Education and the Masters in Trauma Sciences. He is currently the Program Director of the Singhealth Emergency Medicine Residency Program and is actively involved in undergraduate education as an Adjunct Assistant Professor Duke-NUS Graduate Medical School, Clinical Senior Lecturer with Yong Loo Lin School of Medicine.

Listen to the chapter

References

  1. Lundin A, Akram SK, Berg L, Göransson KE, Enocson A. Thoracic injuries in trauma patients: Epidemiology and its influence on mortality.Scandinavian Journal of Trauma, Resuscitation and Emergency Medicine. 2022;30(1). doi:10.1186/s13049-022-01058-6.
  2. Milisavljevic S, Spasic M, Arsenijevic M. Thoracic trauma. Current Concepts in General Thoracic Surgery. 2012. doi:10.5772/54139.
  3. Chrysou K, Halat G, Hoksch B, Schmid RA, Kocher GJ. Lessons from a large trauma center: Impact of blunt chest trauma in polytrauma patients—still a relevant problem? Scandinavian Journal of Trauma, Resuscitation and Emergency Medicine. 2017;25(1). doi:10.1186/s13049-017-0384-y.
  4. Eghbalzadeh K, Sabashnikov A, Zeriouh M, et al. Blunt chest trauma: A clinical chameleon. Heart. 2017;104(9):719-724. doi:10.1136/heartjnl-2017-312111.
  5. Initial Assessment and Management. In: Advanced Trauma Life Support: Student Course Manual. Chicago, IL: American College of Surgeons; 2018:2-21. 
  6. Loseby J, Hudson A, Lyon R. Clinical handover of the trauma and medical patient: A structured approach. Journal of Paramedic Practice. 2013;5(10):563-567. doi:10.12968/jpar.2013.5.10.563. 
  7. Yamamoto L, Schroeder C, Morley D, Beliveau C. Thoracic trauma. Critical Care Nursing Quarterly. 2005;28(1):22-40. doi:10.1097/00002727-200501000-00004. 
  8. Sybrandy KC. Diagnosing cardiac contusion: Old Wisdom and new insights. Heart. 2003;89(5):485-489. doi:10.1136/heart.89.5.485.
  9. American College of Radiology. Major blunt trauma. ACR Appropriateness Criteria Major Blunt Trauma. https://acsearch.acr.org/docs/3102405/Narrative/. Accessed April 1, 2023. 
  10. American College of Radiology. Blunt chest trauma – suspected cardiac injury. ACR Appropriateness Criteria Blunt Chest Trauma. https://acsearch.acr.org/docs/3082590/Narrative/. Accessed April 1, 2023. 
  11. Chan KK, Joo DA, McRae AD, et al. Chest ultrasonography versus supine chest radiography for diagnosis of pneumothorax in trauma patients in the emergency department. Cochrane Database of Systematic Reviews. 2018. doi:10.1002/14651858.cd013031. 
  12. Rodriguez RM, Hendey GW, Mower W, et al. Derivation of a decision instrument for selective chest radiography in blunt trauma. Journal of Trauma: Injury, Infection & Critical Care. 2011;71(3):549-553. doi:10.1097/ta.0b013e3181f2ac9d. 
  13. Ahmadzadeh K, Abbasi M, Yousefifard M, Safari S. Value of NEXUS chest rules in assessment of traumatic chest injuries; a systematic review and a meta-analysis. The American Journal of Emergency Medicine. 2023;65:53-58. doi:10.1016/j.ajem.2022.12.038. 
  14. Rodriguez RM, Langdorf MI, Nishijima D, et al. Derivation and validation of two decision instruments for selective chest CT in blunt trauma: A multicenter prospective observational study (NEXUS Chest CT). PLOS Medicine. 2015;12(10). doi:10.1371/journal.pmed.1001883. 
  15. Subhani SS, Muzaffar MS, Khan MI. Comparison of outcome between low and high thoracic trauma severity score in blunt trauma chest patients. Journal of Ayub Medical College Abbottabad. 2014;26(4):474-7. PMID: 25672168.
  16. Daurat A, Millet I, Roustan J-P, et al. Thoracic trauma severity score on admission allows to determine the risk of delayed ARDS in trauma patients with pulmonary contusion. Injury. 2016;47(1):147-153. doi:10.1016/j.injury.2015.08.031. 
  17. Chen J, Jeremitsky E, Philp F, Fry W, Smith RS. A chest trauma scoring system to predict outcomes. Surgery. 2014;156(4):988-994. doi:10.1016/j.surg.2014.06.045. 
  18. Fokin A, Wycech J, Crawford M, Puente I. Quantification of rib fractures by different scoring systems. Journal of Surgical Research. 2018;229:1-8. doi:10.1016/j.jss.2018.03.025. 
  19. Seok J, Cho HM, Kim HH, et al. Chest trauma scoring systems for predicting respiratory complications in isolated rib fracture. Journal of Surgical Research. 2019;244:84-90. doi:10.1016/j.jss.2019.06.009. 
  20. Galvagno SM, Jr., Nahmias JT, Young DA. Advanced Trauma Life Support((R)) Update 2019: Management and Applications for Adults and Special Populations. Anesthesiol Clin. 2019;37(1):13-32.
  21. Roberts I, Shakur H, Coats T, et al. The crash-2 trial: A randomised controlled trial and economic evaluation of the effects of tranexamic acid on death, Vascular occlusive events and transfusion requirement in bleeding trauma patients. Health Technol Assess. 2013;17(10). doi:10.3310/hta17100. 
  1.  

Reviewed By

Picture of Arif Alper Cevik, MD, FEMAT, FIFEM

Arif Alper Cevik, MD, FEMAT, FIFEM

Prof Cevik is an Emergency Medicine academician at United Arab Emirates University, interested in international emergency medicine, emergency medicine education, medical education, point of care ultrasound and trauma. He is the founder and director of the International Emergency Medicine Education Project – iem-student.org, chair of the International Federation for Emergency Medicine (IFEM) core curriculum and education committee and board member of the Asian Society for Emergency Medicine and Emirati Board of Emergency Medicine.

Bradyarrhythmias (2024)

Bradyarrhythmias

by Hassan M. Alshaqaq & Danya Khoujah

You have a new patient!

An 80-year-old male patient was brought to the emergency department (ED) by ambulance from a long-term care facility with a chief complaint of altered mental state for 2 hours. On arrival, his vitals were as follows: heart rate= 42 beats/min, blood pressure (BP)=70/50 mmHg, SpO2=95% on room air, respiratory rate=23 breaths/min, temperature=37.6°C, glucocheck= 215 mg/dL. The patient was pale, diaphoretic, alert, and disoriented. Pupils were equal and reactive to light. No neurological deficits or lateralizing signs were identified. The patient has a history of diabetes mellitus, hypertension, and ischemic heart disease. There is no history of drug ingestion or recent symptoms of infection. The patient was given 1 liter of intravenous (IV) normal saline during transport. A 12-lead electrocardiogram (ECG) was obtained (Figure 1). What are your initial assessment steps? What are the ECG features? What is the most likely diagnosis? How will you stabilize this patient?

Figure 1. Courtesy of Dr. Ahmad Alsaif. Digitized using PMcardio app. 
Figure 1. Courtesy of Dr. Ahmad Alsaif. Digitized using PMcardio app. 

Importance and Epidemiology

Bradyarrhythmias are defined as rhythms with a ventricular rate lower than 60 beats/min in adults and lower than the age-appropriate rate in pediatrics. Bradycardia may cause symptoms at a rate of <50 beats/min, which is the functional definition that most guidelines use [1]. Bradyarrhythmias are categorized into bradycardias and atrioventricular (AV) blocks [2].

Bradycardias are characterized by a slow rate of both atria and ventricles and include sinus bradycardia, junctional rhythm, idioventricular rhythm, and hyperkalemia-related sinoventricular rhythm. Bradyarrhythmias due to AV blocks are characterized by ventricular beats slower than the atria and include second- and third-degree AV blocks. Uncommonly, atrial fibrillation or flutter may present with a slow ventricular rate secondary to either significant conduction disturbance or excessive nodal-blocking medications.

Patients with bradyarrhythmias can be either stable or unstable. Patients with hemodynamically unstable bradycardia are at a high risk of cardiovascular collapse [3]. On the other hand, stable asymptomatic bradycardia could be physiological in athletes and well-conditioned individuals [4,5].

Compromising bradycardia has an incidence of 6 per 10,000 patients presenting to the ED for any reason; 20% of those require temporary pacing and 50% require a permanent pacemaker [6]. One percent of admission to the intensive care units (ICUs) from the ED are patients with compromising bradycardia, and 10% of syncope can be attributed to bradycardia [6,7]. Due to the significant potential for instability in bradycardic patients [3], emergency clinicians play a critical role in their initial stabilization, identification of underlying etiology, and treatment.

Pathophysiology

Normal cardiac electrophysiology starts with an impulse generated in the sinoatrial (SA) node, traveling into the atria via interatrial conduction pathways leading to atrial contraction, which appears as a P wave on the ECG. The impulse is then conducted down to the AV node, then from the AV node into the bundle of His and bundle branches, and finally into the Purkinje system leading to a ventricular contraction. The AV conduction appears as the PR interval, whereas the ventricular depolarization appears as the QRS complex (Figure 2). Several electrolytes, such as potassium and calcium, play a crucial role in initiating and regulating the cardiac action potential.

Figure 2. Cardiac conduction system. Used with permission from OpenStax licensed under a Creative Commons Attribution 4.0 International License (CC BY 4.0). Access for free at https://openstax.org/books/anatomy-and-physiology/pages/1-introduction
Figure 2. Cardiac conduction system. Used with permission from OpenStax licensed under a Creative Commons Attribution 4.0 International License (CC BY 4.0). Access for free at https://openstax.org/books/anatomy-and-physiology/pages/1-introduction

There is an anatomical variance in the blood supply of the SA and AV nodes. The blood supply of the SA node arises from the right coronary artery in 63% and the left circumflex artery in 37% of the population [8]. The AV node is supplied by a branch of the right coronary artery in 90% and the left circumflex artery in 10% [8]. This is most relevant in acute myocardial infarction (MI), as different may arise from ischemia of the conduction system [9]. MI may also lead to ischemia through a vagal-mediated response.

Cardiac output is dependent on the heart rate and stroke volume. Initially, the reduced heart rate increases the diastolic filling leading to an augmentation in the stroke volume. This compensatory mechanism is eventually overwhelmed, leading to decreased cardiac output, hypoperfusion and potentially cardiogenic shock [2,10].

Blood pressure equals the cardiac output multiplied by the systemic vascular resistance. In patients with reduced cardiac output, endogenous sympathetic reflex leads to vasoconstriction and increased systemic vascular resistance. Therefore, patients could have hypoperfusion despite normal-appearing blood pressure.

The underlying cause of bradyarrhythmias could be either conditions affecting the automaticity of cardiac cells (i.e., their ability to generate impulses) or the conduction of impulses within the cardiac conduction system [11,12]. be intrinsic (due to an innate dysfunction of the heart) or extrinsic (Table 1). A practical mnemonic is DIE, which stands for Drugs, Ischemia, and Electrolytes. Half of the patients with compromising bradycardia have a treatable underlying etiology [6].

Table 1. Causes of bradyarrhythmias [7]

Intrinsic:

  • Myocardial ischemia/infarction
  • Cardiomyopathy
  • Congenital heart disease
  • Degenerative fibrosis (with aging)
  • Traumatic (e.g., Denervation post cardiac surgery, cardiac ablation)
  • Infectious/inflammatory (e.g., myocarditis, endocarditis, Lyme disease, syphilis, sarcoidosis, lupus)
  • Sepsis
  • Infiltrative disorders (amyloidosis, hemochromatosis)

Extrinsic:

  • Autonomic-mediated (e.g., athletes, carotid sinus hypersensitivity, situational)
  • Electrolyte disturbances (e.g., hyperkalemia, hypokalemia, hypocalcemia)
  • Hypothermia
  • Endocrinopathy (e.g., hypothyroidism)
  • Raised intracranial pressure
  • Medication overdose (e.g., β-Blockers, calcium channel blockers, digoxin, clonidine, opioids)
  • Toxicologic exposure (e.g., organophosphate)

Types of Bradycardia

Sinus Bradycardia

Sinus bradycardia is characterized by ECG findings of a P wave preceding each QRS, a fixed P-P interval equal to the R-R interval, and a ventricular rate lower than 60 beats/min (Figure. 3). Sinus bradycardia can be found in healthy individuals, particularly athletes with high resting vagal tone.

Figure 3. Sinus bradycardia. ECG shows sinus and regular rhythm, normal PR interval, and a rate of 46 beats/min. Courtesy of Dr. Ahmad Alsaif. ECG digitized using PMcardio app.
Figure 3. Sinus bradycardia. ECG shows sinus and regular rhythm, normal PR interval, and a rate of 46 beats/min. Courtesy of Dr Hassan Alshaqaq and Dr Danya Khoujah.

It could also be secondary to pathological conditions, such as acute MI (Figure 4) [9]. Patients with sinus bradycardia are usually asymptomatic and do not require any specific treatment. However, profound bradycardia causing hypoperfusion requires emergent treatment of the underlying cause. Atropine may be used, as well as cardiac pacing.

Figure 4. Sinus bradycardia with inferior STEMI. ECG shows sinus and regular rhythm, normal PR interval, and a rate of 58 beats/min combined with ST-segment elevation in the inferior leads (Leads III and aVF), hyperacute T waves in leads III and aVF with relative loss of R wave height, early Q-wave formation in leads II, III and aVF, and reciprocal ST depression and T wave inversion in leads I and aVL. Courtesy of Dr. Ahmad Alsaif. ECG digitized using PMcardio app.
Figure 4. Sinus bradycardia with inferior STEMI. ECG shows sinus and regular rhythm, normal PR interval, and a rate of 58 beats/min combined with ST-segment elevation in the inferior leads (Leads III and aVF), hyperacute T waves in leads III and aVF with relative loss of R wave height, early Q-wave formation in leads II, III and aVF, and reciprocal ST depression and T wave inversion in leads I and aVL. Courtesy of Dr. Hassan Alshaqaq and Dr. Danya Khoujah.

Junctional Rhythm

When the SA node fails to discharge or the discharge of the SA node fails to reach the AV node, the AV node generates “junctional escape beats” at a ventricular rate ranging between 40 and 60 beats/min (Figure. 5). These QRS complexes on the ECG are not usually preceded by P waves. However, in some cases, the junctional beats conduct in retrograde into the atria, producing a P wave before or after the QRS. If present before the QRS complex, the PR interval will be abnormally short (<120 msec).

Figure 5. Junctional rhythm. ECG shows an absence of sinus P waves, regular and narrow QRS complexes, and a ventricular rate of 54 beats/min. Also, there is evidence of 2 premature ventricular contractions (PVCs). Courtesy of Dr. Ahmad Alsaif. ECG digitized using PMcardio app.
Figure 5. Junctional rhythm. ECG shows an absence of sinus P waves, regular and narrow QRS complexes, and a ventricular rate of 54 beats/min. Also, there is evidence of 2 premature ventricular contractions (PVCs). Courtesy of Dr. Ahmad Alsaif. ECG digitized using PMcardio app.

Sustained junctional escape rhythm may be caused by inferior MI with right ventricle (RV) extension (Figure 6) [9], posterior MI, myocarditis, hypokalemia, and digitalis toxicity. Infrequent junctional escape beats do not require treatment. Treating the underlying etiology is the mainstay of treatment in symptomatic patients and atropine can be used as a bridge until then. Atropine enhances SA node discharge rate and AV nodal conduction, therefore suppressing slower pacemakers. In case of hemodynamic compromise, cardiac pacing might be necessary [9].

Figure 6. Junctional rhythm with inferior STEMI with posterior extension. ECG shows an absence of sinus P waves, regular and narrow QRS complexes, and ventricular rate of 43 beats/min combined with ST-segment elevation in the inferior leads (Leads II, III, and aVF), ST elevation in lead III > II, early Q-wave formation in lead III, horizontal ST segment depression and upright T waves in leads V2-V3 (posterior MI), and reciprocal ST depression in leads V4-V6, lead I, and aVL. Courtesy of Dr. Anas Halim. Digitized using PMcardio app.
Figure 6. Junctional rhythm with inferior STEMI with posterior extension. ECG shows an absence of sinus P waves, regular and narrow QRS complexes, and ventricular rate of 43 beats/min combined with ST-segment elevation in the inferior leads (Leads II, III, and aVF), ST elevation in lead III > II, early Q-wave formation in lead III, horizontal ST segment depression and upright T waves in leads V2-V3 (posterior MI), and reciprocal ST depression in leads V4-V6, lead I, and aVL. Courtesy of Dr. Anas Halim.

Idioventricular Rhythm

In idioventricular rhythm, beats originate from the ventricles with wide (>120 ms) and regular QRS complexes and a rate of 30-50 beats/min. In idioventricular rhythms, ECG shows no P waves (Figure 7). It is usually non-sustained; present for a short duration only. The significance of idioventricular rhythm is that it is most commonly seen in the setting of an ST-segment elevation MI (STEMI) [13].

Figure 7. Idioventricular rhythm. ECG shows regular and wide QRS complexes (>120 ms), absence of P wave, and ventricular rate of 49 beats/min. Courtesy of Dr. Anas Halim. ECG digitized using PMcardio app.
Figure 7. Idioventricular rhythm. ECG shows regular and wide QRS complexes (>120 ms), absence of P wave, and ventricular rate of 49 beats/min. Courtesy of Dr. Hassan Alshaqaq and Dr. Danya Khoujah.

Atropine may be utilized in symptomatic patients but is usually unsuccessful. If the rhythm persists and is compromising, cardiac pacing may be attempted. Antiarrhythmic agents are best avoided as these could lead to asystole by suppressing the rescue functioning pacemaker [9,14,15].

Table 2 compares ECG features of bradycardic rhythms.

Sinus bradycardia

Junctional rhythm

Idioventricular rhythm

 
  • Sinus P waves present.
  • PR interval 120-200 msec (normal).
  • Rate < 60 beats/min.
  • Absence of normal sinus P waves (may be retrograde).
  • Ventricular rate 40-60 beats/min.
  • Narrow QRS complexes.
  • Absence of P waves.
  • Ventricular rate 30-50 beats/min.
  • Regular and wide QRS complexes.
 
 
 
 

Sinus Node Dysfunction

Sinus node dysfunction is caused by the failure of the sinus node to generate or conduct appropriate cardiac potentials. It can be associated with various supraventricular rhythms, including tachycardia and bradycardia, as well as prolonged pauses (>3 secs). It is most often due to age-dependent fibrosis of the nodal tissue [16,17] or post-cardiac transplantation [18], and might also be seen in patients with myocardial ischemia [19,20], myocarditis [19], and cardiomyopathy [7, 21, 22]. On ECG, it is characterized by episodes of bradycardia and/or sinus arrest with episodes of supraventricular tachycardia (Figure 8).

Figure 8. Sinus node dysfunction. ECG shows tachycardia interspersed with long sinus pauses (absence of any electrical activity). Courtesy of Dr. Anas Halim. ECG digitized using PMcardio app.
Figure 8. Sinus node dysfunction. ECG shows tachycardia interspersed with long sinus pauses (absence of any electrical activity). Courtesy of Dr. Hassan Alshaqaq and Dr. Danya Khoujah.

First-Degree AV Block

First-degree AV block (more accurately described as AV delay) is characterized by delayed AV conduction of all atrial impulses to the ventricles at the level of the atria, AV node, or His-Purkinje system. There is no blocked atrial conduction. Therefore, the ECG reveals a 1:1 atrioventricular conduction (P wave for each QRS complex) and prolonged PR interval (>200 milliseconds) (Figure 9).

Figure 9. First-degree AV block. ECG shows sinus rhythm, 1:1 atrioventricular conduction, with a fixed prolonged PR interval (PR interval = 203 ms). Courtesy of Dr. Anas Halim. ECG digitized using PMcardio app.
Figure 9. First-degree AV block. ECG shows sinus rhythm, 1:1 atrioventricular conduction, with a fixed prolonged PR interval (PR interval = 203 ms). Courtesy of Dr. Hassan Alshaqaq and Dr. Danya Khoujah.

Table 3 illustrates a comparison between ECG features of the AV block types.

 

Type of Block

 

First-degree AV block

Second-degree AV block Mobitz type I

Second-degree AV block Mobitz type II

Third-degree AV block

PQRS

 

Sinus rhythm (P wave for each QRS complex)

 

Nonconducted atrial impulse (P wave not followed by QRS complex)

Cycle repeats after a dropped beat (atrial impulse is completely blocked)

Nonconducted atrial impulse (P wave not followed by QRS complex)

PR interval remains constant after the non-conducted atrial impulse

No association between P waves and QRS complexes (complete dissociation)

Atrial rate higher than ventricular rate

 

PR interval

PR interval >200 msec

Progressively prolonged PR interval

Fixed prolonged PR interval

PR interval remains constant after the non-conducted atrial impulse

Variable PR interval

QRS

Narrow

Regular

Narrow

Narrow

Wide

Regular

First-degree AV block could be found as a normal variant or may be secondary to increased vagal tone, medication toxicity, inferior MI (Figure 10) [9], or myocarditis [15]. Those with a new-onset first-degree AV block in the setting of ACS may be at a higher risk of progression to complete heart block [9,15]. Patients with first-degree AV block usually do not require specific treatment,1 especially if asymptomatic. Symptomatic patients should have their management focused on the underlying cause. Agents with AV nodal blocking effect (Table 4) should be avoided as they would worsen the conduction delay [23].

Figure 10. First-degree AV block with inferolateral STEMI. ECG shows sinus rhythm, 1:1 atrioventricular conduction, with a fixed prolonged PR interval (PR interval = 205 ms) combined with ST-segment elevation in the inferior leads (Leads II, III, and aVF) and lateral leads (V5-V6), ST depression in V1-V2 is suggestive of associated posterior infarction, and reciprocal ST depression in leads I and aVL. Courtesy of Dr. Anas Halim. ECG digitized using PMcardio app.
Figure 10. First-degree AV block with inferolateral STEMI. ECG shows sinus rhythm, 1:1 atrioventricular conduction, with a fixed prolonged PR interval (PR interval = 205 ms) combined with ST-segment elevation in the inferior leads (Leads II, III, and aVF) and lateral leads (V5-V6), ST depression in V1-V2 is suggestive of associated posterior infarction, and reciprocal ST depression in leads I and aVL. Courtesy of Dr. Hassan Alshaqaq and Dr. Danya Khoujah.

Table 4. List of agents with AV nodal blocking/slowing activity [24].

Agents with potent AV nodal blocking activity:

  • Beta-blockers (e.g., Esmolol, Metoprolol)
  • Calcium channel blockers (e.g., Diltiazem, Verapamil)
  • Digoxin
  • Adenosine
  • Sotalol

Other cardiovascular agents with AV nodal blocking/slowing activity:

  • Amiodarone
  • Procainamide
  • Quinidine
  • Lidocaine
  • Flecainide
  • Propafenone
  • Disopyramide
  • Dronedarone

Second-Degree AV Block

Second-degree AV block is characterized by intermittent conduction failure, where one or more atrial impulses are not conducted; some conduction is still present. The atrial rate (P waves) is <100 bpm. The ECG shows P waves that are not followed by a ventricular contraction (QRS complex). It is classified into two types based on the pathophysiology and ECG features.

Second-Degree Mobitz Type I AV Block

Mobitz type I (also known as Wenckebach’s block) is characterized by a progressive prolongation of the AV conduction with a nonconducted atrial impulse. The ECG shows progressive prolongation of the PR interval until an atrial impulse is blocked (not followed by a QRS complex) (Figure 11). These cyclic features lead to a grouped beating in a rhythm strip.  Mobitz Type I could be identified in healthy individuals and might also be seen in patients with acute or chronic heart disease such as inferior MI [9], medication toxicity, myocarditis, or in patients after cardiac surgery. In most cases, specific treatment is not required unless hypoperfusion is present, in which case atropine is the first-line therapy [9].

Figure 11. Second-degree Mobitz type I AV Block. ECG shows progressive prolongation of PR interval with a subsequent nonconducted P wave, then the cycle repeats after the dropped beat. Courtesy of Dr. Harry Patterson, FACEM. Used with permission from Life in the Fast Lane licensed under a Creative Commons Attribution-NonCommercial-ShareAlike 4.0 International License (CC BY-NC-SA 4.0). Source: https://litfl.com/. ECG digitized using PMcardio app.
Figure 11. Second-degree Mobitz type I AV Block. ECG shows progressive prolongation of PR interval with a subsequent nonconducted P wave, then the cycle repeats after the dropped beat. Courtesy of Dr. Harry Patterson, FACEM. Used with permission from Life in the Fast Lane licensed under a Creative Commons Attribution-NonCommercial-ShareAlike 4.0 International License (CC BY-NC-SA 4.0). Source: https://litfl.com/.
Second-Degree Mobitz Type II AV Block

Mobitz type II block is characterized by a fixed AV conduction delay followed by a nonconducted atrial beat. ECG shows a fixed prolonged PR interval, each P wave is followed by a QRS complex until a nonconducted P wave is noted without a QRS complex (Figure 12). If 2 or more P waves are not conducted, it is called a high-grade AV block (Figure 13). Mobitz type II block represents electrical intranodal conducting system structural damage. In acute MI (most commonly anterior), it could progress into a complete heart block [9].

Figure 12. Second-degree Mobitz II AV block. The rhythm strip shows non-conducted P waves (arrows), with constant PR interval, constant P-P interval, and the RR interval surrounding the dropped beat is multiple the preceding RR interval. Used with permission from Life in the Fast Lane licensed under a Creative Commons Attribution-NonCommercial-ShareAlike 4.0 International License (CC BY-NC-SA 4.0). Source: https://litfl.com/
Figure 12. Second-degree Mobitz II AV block. The rhythm strip shows non-conducted P waves (arrows), with constant PR interval, constant P-P interval, and the RR interval surrounding the dropped beat is multiple the preceding RR interval. Used with permission from Life in the Fast Lane licensed under a Creative Commons Attribution-NonCommercial-ShareAlike 4.0 International License (CC BY-NC-SA 4.0). Source: https://litfl.com/
Figure 13. Second-degree “high-grade” AV block. ECG shows a 4:1 conduction ratio, atrial rate is approximately 140 beats/min, and ventricular rate is approximately 35 beats/min. Used with permission from Life in the Fast Lane licensed under a Creative Commons Attribution-NonCommercial-ShareAlike 4.0 International License (CC BY-NC-SA 4.0). Source: https://litfl.com/
Figure 13. Second-degree “high-grade” AV block. ECG shows a 4:1 conduction ratio, atrial rate is approximately 140 beats/min, and ventricular rate is approximately 35 beats/min. Used with permission from Life in the Fast Lane licensed under a Creative Commons Attribution-NonCommercial-ShareAlike 4.0 International License (CC BY-NC-SA 4.0). Source: https://litfl.com/

These patients should be placed on transcutaneous pacing pads in anticipation of clinical deterioration. Although atropine is usually ineffective, it may be utilized during preparation for pacing as it is not harmful. Patients with second-degree Mobitz type II AV block and features of hypoperfusion require emergent cardiac pacing. Eventually, these patients require transvenous cardiac pacing, especially those with underlying acute MI [9].

Third-Degree AV Block

Third-degree (complete) heart block is characterized by the lack of any AV conduction between atria and ventricles, leading to AV dissociation. Atrial impulses are not conducted at all, and an escape pacemaker arises to pace the ventricles with a rate of 40-60 beats/min or lower for the intranodal level. The ECG features no association between the P waves and QRS complexes, an atrial rate higher than the ventricular rate, and wide QRS complexes (Figure 14).

Figure 14. Third-degree AV block. ECG shows complete dissociation between P waves and QRS complexes, wide QRS complexes (QRS duration=154 ms), atrial rate of ~100 beats/min, Ventricular rate of ~30 beats/min, and the rhythm is maintained by a junctional escape rhythm. Courtesy of Dr. Ahmad Alsaif. Digitized using PMcardio app.
Figure 14. Third-degree AV block. ECG shows complete dissociation between P waves and QRS complexes, wide QRS complexes (QRS duration=154 ms), atrial rate of ~100 beats/min, Ventricular rate of ~30 beats/min, and the rhythm is maintained by a junctional escape rhythm. Courtesy of Dr. Ahmad Alsaif. Digitized using PMcardio app.

Depending on the block’s location, it may be termed “nodal” (such as in inferior MI) or “infranodal” (such as in anterior MI) [9]. Patients with third-degree AV block are usually unstable due to the inadequate cardiac output generated by the ventricular escape pacemaker. Atropine might work in cases of nodal blockade; however, it is unlikely to be effective in infranodal blockade. If the response to atropine is inadequate, transcutaneous cardiac pacing should be started. In those patients, consider beta-adrenergic medications (epinephrine or dopamine). Eventually, transvenous pacing is necessary in the majority of cases.

Atrial Fibrillation With a Slow Ventricular Response

Patients with atrial fibrillation may present with a slow ventricular response in the setting of an overdose of nodal-blocking medications (e.g., beta-blockers, calcium channel antagonists, digoxin) or significant conduction disease [25]. A very low ventricular rate may lead to hemodynamic compromise [26]. The hallmark ECG feature in these patients is an irregularly irregular ventricular rhythm without discernible P waves, either chaotic or flat isoelectric baseline (Figure 15). Patients with an irreversible cause of slow ventricular response (such as those with intrinsic conduction system disease) may require a permanent pacemaker [27].

Figure 15. Atrial fibrillation with a slow ventricular response. ECG shows irregular rhythm with no evidence of organized atrial activity, fine fibrillatory waves, with a ventricular rate of 60 beats/min. Courtesy of Dr. Ahmad Alsaif. ECG digitized using PMcardio app.
Figure 15. Atrial fibrillation with a slow ventricular response. ECG shows irregular rhythm with no evidence of organized atrial activity, fine fibrillatory waves, with a ventricular rate of 60 beats/min. Courtesy of Dr. Hassan Alshaqaq and Dr. Danya Khoujah.

Hyperkalemia-induced Bradycardia

Patients with hyperkalemia demonstrate several features on the ECG, including peaked T waves, PR prolongation, QRS widening, and sine-wave morphology [28]. Junctional rhythm is the most common bradycardic rhythm in patients with severe hyperkalemia [29], and patients may show other types of bradydysrhythmias [29]. Obtaining a serum potassium level is critical in any bradydysrhythmia, even without other ECG changes suggestive of hyperkalemia [29].

BRASH Syndrome

BRASH syndrome (Bradycardia, Renal failure, AV blockade, Shock, and Hyperkalemia) is caused by a synergistic effect of hyperkalemia and AV-blocking medications that produce dramatic bradycardia [30]. The marked bradycardia leads to poor renal perfusion, exacerbating hyperkalemia and leading to a vicious cycle, with resulting hemodynamic instability and multiorgan failure [30]. Precipitating factors identified include nephrotoxins, potassium-sparing diuretics, angiotensin-converting enzyme inhibitors, angiotensin-receptor blockers, and digitalis [30].

Medical History

After stabilizing the patient, a comprehensive medical history often helps identify the underlying etiology. In patients unable to provide history (e.g., altered mentation), collateral history from family, emergency medical service (EMS) personnel, or nursing facility staff can be helpful.

Determining the stability of patients starts with their medical history. Symptoms of end-organ hypoperfusion, such as ischemic chest pain, dyspnea, altered mental state, and syncope,  may indicate hemodynamic instability [3,6]. Lack of symptoms, or only mild symptoms such as palpitations, lightheadedness, nausea, generalized fatigability, or mild anxiety [31], may indicate hemodynamic stability. It is essential to determine whether the presenting symptoms in patients with bradycardia are secondary to the bradycardia itself or whether the presence of bradycardia is coincidental.

Analyzing the characteristics of presenting symptoms is vital, including onset, progression (gradual vs. abrupt), duration, precipitating factors, preceding events, and any associated symptoms. Specific precipitating events include emotional distress, positional changes, urination, defecation, cough, prolonged standing, shaving, and head-turning [7]. Additionally, ask about symptoms of ACS, infectious diseases, and hypothyroidism.

A past medical history of rhythm disturbance, ischemic heart disease, structural heart disease, pacemaker placement, or coronary artery bypass graft (CABG) is relevant. Patients post transcatheter aortic valve replacement are at high risk for conduction system abnormalities [7]. A history of renal failure increases the risk of hyperkalemia.

Reviewing the medication list, as well as herbal substances and recreational drugs, including timing, doses, recent changes in patterns, compliance, and the possibility of overdose, is essential.

Travel history to areas endemic for infectious diseases, such as Chagas or Lyme, is relevant. Sexual history, specifically for history of or risk factor for syphilis, is relevant. Occupational history and chemical exposures (e.g., organophosphate) may offer clues for a toxicologic etiology.

Physical Examination

Physical examination of patients with rhythm disturbance primarily focuses on clues of end-organ hypoperfusion and underlying etiology. Assess the level of consciousness, and confirm adequate perfusion by examining the capillary refill, peripheral pulses, and temperature of extremities; cool extremities and delayed capillary refill (>3 sec) indicate hypoperfusion.

The cardiovascular examination includes palpating the radial pulse to determine the rate and rhythm, auscultating the heart sounds for the presence of murmurs, as well as looking for signs of heart failure (jugular venous distention, rales, edema of the extremities). Chest wall inspection for a midline sternotomy scar adds important information regarding the medical history. Palpating the skin overlying an implanted pacemaker may uncover lead abnormalities [32].

Look for signs of underlying pathology, including toxidromes (e.g., cholinergic, opioid-like), an arteriovenous fistula or dialysis catheter (indicates renal failure), head trauma, and signs of hypothyroidism. Core temperature measurement is vital in patients suspected to have hypothermia. In patients with head trauma, look for Cushing’s triad (hypertension, bradycardia, and irregular respirations). Table 5 illustrates a constellation of signs and symptoms indicating a possible underlying cause.

Table 5. Signs and symptoms suggestive of the underlying cause of bradycardia.

Possible underlying etiology

History

Physical examination

Myocardial infarction

  • Chest pain
  • Dyspnea
  • Anginal equivalent symptoms
  • Hypotension
  • Rales
  • New murmur

Myopericarditis

  • Fever
  • Dyspnea
  • Pleuritic chest pain
  • Recent respiratory or gastrointestinal infection
  • Malaise
  • Myalgias
  • Pericardial friction rub

Hyperkalemia

  • History of renal failure / dialysis
  • Arteriovenous fistula or dialysis catheter

Pacemaker malfunction

  • History of implanted cardiac device
  • History of cardiac conduction abnormality
  • Pacemaker identification card
  • Obvious device or lead abnormalities by palpation of the overlying skin

Increased ICP

  • History of head trauma
  • Headache
  • Vomiting
  • Decreased level of consciousness
  • Signs of basilar fracture
  • Depressed skull fracture
  • Focal neurologic deficit
  • Decreased level of consciousness

Beta blocker toxicity

  • Confusion
  • History of seizure
  • History of drug ingestion
  • History of underlying cardiac disease
  • Presence of cardioactive agents at home
  • Empty medication bottles
  • Depressed mental state
  • Hypoglycemia
  • Bronchospasm
  • Hypotension
  • Shock

Calcium channel blocker toxicity

  • History of drug ingestion
  • History of underlying cardiac disease
  • Presence of cardioactive agents at home
  • Empty medication bottles
  • Hyperglycemia
  • Hypotension
  • Shock

Digoxin toxicity

  • GI symptoms (abdominal pain, nausea, vomiting, anorexia)
  • Fatigue
  • Visual disturbance
  • Lethargy
  • Confusion
  • History of drug ingestion
  • History of underlying cardiac disease
  • Presence of cardioactive agents at home
  • Empty medication bottles
  • Altered mental status
  • Seizures
  • Hypotension
  • Shock

Local anesthetics toxicity

  • Perioral numbness
  • Visual disturbance
  • Confusion
  • Dizziness
  • Seizure
  • History of regional or topical anesthesia procedure
  • Altered mental status (ranging from agitation to unresponsiveness)
  • Weakness
  • Slurred speech

Clonidine overdose

  • History of drug ingestion
  • Miosis
  • Respiratory depression

Hypothyroidism

  • History of thyroid disease
  • Cold intolerance
  • Weight gain
  • Fatigability
  • Hypothermia
  • Constipation
  • Skin and hair changes
  • Thyroidectomy scar
  • Goiter
  • Pretibial myxedema
  • Coarse skin
  • Muscle hypertrophy
  • Hypothermia
  • Delayed relaxation phase of reflexes

Acing Diagnostic Testing

A 12-lead ECG is diagnostic for bradyarrhythmia. A single lead, most commonly lead II, is often adequate to make the diagnosis. ECG reading requires noting the rate, regularity, P waves, P-R interval, the relation between P waves and QRS complexes, and QRS width.

Furthermore, ECG may identify the underlying cause, such as acute ischemic changes (e.g., hyperacute T waves, ST-segment elevation/depression, T waves inversions, Q-waves), hyperkalemic changes (e.g., peaked T waves, P wave flattening, PR prolongation, wide QRS), cerebral T-waves (i.e., deep, symmetric, inverted T waves) in elevated intracranial pressure (ICP), downsloping ST depression in digoxin toxicity, and Osborn wave (i.e., positive deflection at the J point) in severe hypothermia (<32˚C). In patients with a malfunctioning implanted pacemaker, the ECG may show pacing spikes not followed by a QRS complex (electrical non-capture).

Given the intermittent nature of some bradyarrhythmia, a normal ECG does not exclude the diagnosis, especially in asymptomatic patients. Moreover, bradyarrhythmia may evolve; therefore, serial ECGs are advisable [2]. 

Point-of-care ultrasound (POCUS) helps in the assessment of volume status (inferior vena cava [IVC] assessment) and pulmonary edema (B-lines: vertical comet tail artifact).

Laboratory work-up to uncover the underlying etiology should be tailored to the clinical assessment. It may include the following:

  • Cardiac biomarkers (e.g., troponin) for identifying acute MI.
  • Electrolytes profile for identifying electrolyte abnormalities (particularly potassium and calcium).
  • Creatinine and blood urea nitrogen (BUN) can help diagnose acute and chronic renal failure.
  • Brain natriuretic peptide (BNP) elevation may indicate fluid overload and heart failure in the appropriate context.
  • Infectious work-up: 3 sets of peripheral cultures for infective endocarditis, serologic testing for syphilis, Lyme serology for Lyme disease, and thick and thin blood smear for Chagas disease.
  • Thyroid function test (TSH and free T4) for suspected hypothyroidism or myxedema coma.
  • Drug levels such as digoxin.

As for imaging, a chest X-ray may show evidence of pulmonary edema in patients with heart failure. In patients with pacemakers, it can identify lead fracture and migration.

Transthoracic echocardiography is essential in evaluating patients with newly identified Mobitz type II or third-degree AV block for cardiac wall motion abnormalities (indicative of an MI), valvular abnormalities, and structural heart disease. However, in patients with asymptomatic bradycardia or first-degree AV block, routine echocardiography is usually not indicated [7].

In patients with signs of a head injury and suspected increased ICP, obtain a head computed tomography (CT) scan.

Risk Stratification

Stability of Patient

Determining the hemodynamic stability of patients with bradyarrhythmia critical to management. Unstable patients require immediate treatment. Instability indicators include hypotension (defined as systolic BP [SBP] of <90 mm Hg), signs of shock (cold and moist skin, pale skin, delayed capillary refill time), ischemic chest pain, altered mental state, and dyspnea secondary to pulmonary edema (Table 6). Patients’ condition may change during treatment or ED stay, necessitating frequent reassessment to recognize impending clinical deterioration.

Table 6. Indicators of instability in patients with bradydysrhythmias

  1. Hypotension (SBP <90 mm Hg)
  2. Signs of shock
  3. Altered mental status
  4. Ischemic chest pain
  5. Dyspnea secondary to pulmonary edema

Symptomatic versus Asymptomatic

The presence of symptoms guides management. Generally, asymptomatic patients do not require treatment and may need monitoring only. In symptomatic patients, it is important to determine whether symptoms are caused by bradycardia or if the bradycardia is incidental to the actual cause of the presenting complaint.

Management

Pre-hospital care considerations

Stabilization might be started by EMS personnel, such as administration of atropine, epinephrine/dopamine infusion, and transcutaneous cardiac pacing [11,33-37], depending on their capabilities and local protocols. Transportation to a PCI-capable center is optimal for patients suspected to have an underlying ischemic pathology.

Initial Stabilization

Initial stabilization must start with a rapid assessment of the circulation, airway, and breathing “the CABs”), as well as the vital signs and signs of instability (Table 6). Patients with symptomatic bradyarrhythmia require a monitored bed with flat positioning [7]. Perform fingerstick glucose, establish two large-bore IVs, attach the patient to a continuous cardiac rhythm monitor, obtain a 12-lead ECG, and prepare for drug and electrical therapy simultaneously. If IV access cannot be obtained, attempt intraosseous (IO) access or a central line. Patients with significant symptomatic bradyarrhythmia, those with advanced heart block, and those predicted to deteriorate should be placed on pacer pads, irrespective of stability.

Although electrical cardiac pacing is immediately indicated in unstable patients, medical management may be more immediately available, making it reasonable to administer both while simultaneously identifying and treating the underlying cause [1,11]. However, atropine administration should never delay the initiation of transcutaneous pacing in unstable patients.1

A resuscitation cart, transvenous pacer kit, and airway equipment should be available at the bedside in anticipation of deterioration of critically ill patients with bradyarrhythmia. Oxygen is indicated to target SpO2 ≥94%. In select patients with hypotension (SBP < 90 mmHg), crystalloid fluids may improve blood pressure; caution must be taken in patients with decompensated heart failure. Hypotension refractory to fluid resuscitation may require inotropic and/or vasopressor support (e.g., epinephrine).

For unstable Mobitz Type II or third-degree AV block, initiate transcutaneous pacing, as well as atropine, if immediately available.  If ineffective, administer epinephrine 2-10 µg/min IV infusion. Although atropine is considered the first line of medical therapy, epinephrine may be preferred in critically unstable patients [38,39], and may be administered short-term through a peripheral IV access [40].

In patients with a wide QRS complex, atropine is unlikely to be effective due to a block at the distal conduction system; therefore, proceed immediately to epinephrine and transcutaneous pacing [1,41].

Patients with symptomatic bradycardia, or second- or third-degree AV block should be placed on continuous ECG monitoring (Class I recommendation) [42].

Figure 16 illustrates emergency approach and management of acute bradyarrhythmia in the ED.

Figure 16. Emergency management approach of patients with bradydysrhythmia. *Atropine should not be given to patients after heart transplantation. Abbreviations: BP, blood pressure; ECG, electrocardiogram; HR, heart rate; ICP, intracranial pressure; IV, intravenous; kg, kilogram; mA, milliampere; mg, milligram; mcg, microgram; MI, myocardial infarction; PCI, percutaneous coronary intervention; RR, respiratory rate; SpO2, oxygen saturation.
Figure 16. Emergency management approach of patients with bradydyarrhythmia. *Atropine should not be given to patients after heart transplantation. Abbreviations: BP, blood pressure; ECG, electrocardiogram; HR, heart rate; ICP, intracranial pressure; IV, intravenous; kg, kilogram; mA, milliampere; mg, milligram; mcg, microgram; MI, myocardial infarction; PCI, percutaneous coronary intervention; RR, respiratory rate; SpO2, oxygen saturation. Courtesy of Dr. Hassan Alshaqaq and Dr. Danya Khoujah.

Treatment Considerations

The anatomical location causing the bradycardia may predict the management and outcomes of bradyarrhythmia. Narrow QRS complex bradycardia indicates dysfunction at the level of the SA or AV node, usually requires minimal intervention, and is rarely life-threatening. On the other hand, wide QRS complex bradycardia indicates dysfunction at the level of distal His-Purkinje, usually requires aggressive management, is unlikely to respond to atropine, may require electrical pacing, and is associated with an elevated mortality rate [3].

Medications

Atropine

Atropine is the first-line treatment to increase the heart rate in patients with symptomatic bradycardia [1,2]. Atropine is an antimuscarinic medication with direct vagolytic activity, increasing the SA node’s automaticity and potentiating the AV node’s conduction. Atropine is most effective in sinus bradycardia and junctional rhythm. It is usually not useful in infranodal blocks presenting with wide QRS bradyarrhythmia. Atropine should be used cautiously in the setting of ACS due to the potential risk of exacerbating ischemia and infarct size from the resulting tachycardia [43-48]. Additionally, atropine should not be used in patients who have undergone heart transplants due to the lack of vagal innervation, as atropine may cause paradoxical AV block and asystole [1,7,49,50]. The dose of atropine is 0.5-1 mg IV every 3-5 min until resolution or maximum dose is reached (3 mg) [1,7].

Epinephrine

Epinephrine is an alternative medical therapy for bradyarrhythmia, and is the preferred adjunct medical management to electrical therapy in unstable patients, particularly those with Mobitz Type II and third-degree AV block [7]. Epinephrine acts on β1- and β2-receptors, working on the entire myocardium to increase the heart rate (inotropic and chronotropic effects), as well as enhancing the AV nodal conduction [51-52]. The dose of epinephrine is 2-10 mcg/min IV infusion, titrated to desired heart rate [7].

In peri-arrest bradycardia, some experts recommend a temporizing bolus dose of 20-50 mcg of epinephrine as an IV push, followed by infusion and electrical pacing [38,39,53,54], despite the lack of supporting data [53,54].

Dopamine

Dopamine may be used in bradyarrhythmia refractory to atropine (Class IIb) [7]. It acts on dopaminergic α1-, β1-, and β2-receptors. Higher doses (>10 mcg/kg/min) have a vasoconstrictive effect, while lower doses (1-2 mcg/kg/min) have a selective inotropic effect on the heart rate [1]. The dose of dopamine is 2 to 20 mcg/kg/min IV infusion titrated by 5 mcg/kg/min every 2 minutes to the desired heart rate [7,33], monitoring peripheral perfusion to avoid profound vasoconstriction [7].

Dobutamine

Dobutamine is a β-agonist agent with a weak α-adrenergic activity that may be used in symptomatic bradycardia; its predominant effect is inotropic via stimulation of β1-receptors [55]. Dobutamine can lead to vasodilation (via β2-receptors) and hypotension, thus, it should not be used in hypotensive patients. It can be used in cases of bradycardia resistant to standard therapy with normal or elevated blood pressure [7,38]. Dobutamine is administered as a 2-20 µg/kg/min IV infusion, titrated to desired heart rate.

Isoproterenol

Isoproterenol infusion is indicated in post-heart transplant patients with unstable bradycardia [7]. It can also be used in refractory bradyarrhythmia and AV blocks not responding to epinephrine [38]. It is a non-selective β-agonist stimulating β1– and β2-receptors, speeding up the SA and AV nodes and enhancing cardiac contractility (chronotropic and inotropic) [56]. It does not have any vasopressor effects.  Isoproterenol is given as a 2-10 mcg/min IV infusion, titrated to effect, or can be administered as an IV bolus of 1-2 mcg [1]. Isoproterenol is contraindicated in patients with angina/active ischemia due to concerns about increasing myocardial oxygen demand (β1 effect) and decreasing coronary perfusion (β2 effect), as well as in digoxin toxicity [7,57-60].

Aminophylline and Theophylline

Aminophylline and theophylline are methylxanthines, which exert positive chronotropic effects on the myocardium, likely by inhibiting the suppression effect of the adenosine on the SA node [7]. Both are reasonable to use if clinically indicated in symptomatic post-cardiac transplant patients [7,61-63] and sinus node dysfunction secondary to spinal cord injury, based on a limited case series [7,64-67].

In addition, aminophylline (250 mg IV bolus) has been used in treating second- and third-degree AV block associated with acute inferior MI, despite the limited direct evidence [7,68-70]. Aminophylline is administered with a dose of 6 mg/kg over 20-30 minutes, followed by an infusion of 0.3-0.5 mg/kg/hour [7]. Theophylline is administered as a bolus dose of 300 mg IV, followed by an oral dose of 5–10 mg/kg/day titrated to effect [7]. Although recommended by the guidelines, evidence for the use of methylxanthines is limited [7].

Table 7. Present bradyarrhythmia medications’ mechanism of action, dosing, pharmacokinetics, contraindications, and adverse events. 

Table created by authors

Electrical Cardiac Pacing

Electrical cardiac pacing is a procedure that aims to stimulate effective cardiac depolarization. Cardiac pacing is the mainstay management of acutely symptomatic patients with bradyarrhythmia, particularly unstable bradyarrhythmia or stable symptomatic bradyarrhythmia refractory to medical therapy, including type II second-degree and third-degree AV blocks [7]. It is of little value in toxin-induced bradyarrhythmia [32]. Cardiac pacing is performed using either a transcutaneous or transvenous approach. Transcutaneous cardiac pacing is a temporary bridging treatment until transvenous pacing or resolution of symptoms [7,32,38,39]. Transvenous pacing is also temporary until the resolution of the underlying cause or placement of a permanent pacemaker [7]. 

In crashing bradycardia, transcutaneous pacing should be started immediately. It is minimally invasive, instituted rapidly, and effectively treats hemodynamically unstable bradydysrhythmia [32]. The pacing pads are placed on the patient’s chest using one of two positions, an anterolateral or anteroposterior (Figure 17); positioning placement is selected based on the patient’s habitus and clinician’s preference [32]. Sedation and analgesia should be initiated as soon as possible utilizing hemodynamically stable agents such as low-dose fentanyl and/or ketamine, keeping in mind that most patients requiring electrical pacing are hemodynamically unstable.

Figure 17. Correct placement of transcutaneous pacing pads. A, anterolateral position; the anterior adhesive pad is placed inferior to the right clavicle, and the lateral adhesive pad is placed on the left fifth intercostal space at the anterior axillary line. B, anteroposterior position; the anterior adhesive pad is placed on the sternum, and the posterior adhesive pad is placed on the left infrascapular area. “Illustration by Malak Alraygi / re-designed Arif Alper Cevik”
Figure 17. Correct placement of transcutaneous pacing pads. A, anterolateral position; the anterior adhesive pad is placed inferior to the right clavicle, and the lateral adhesive pad is placed on the left fifth intercostal space at the anterior axillary line. B, anteroposterior position; the anterior adhesive pad is placed on the sternum, and the posterior adhesive pad is placed on the left infrascapular area. “Illustration by Malak Alraygi / re-designed by Arif Alper Cevik”

Adjust the pacing setting targeting a rate of 80-100 beats/min (start at 80 mA and reduce to the lowest energy) [2,39]. If the pacing is successful, the ECG will show electrical capture, which are pacing spikes followed by wide QRS complexes. Mechanical capture is demonstrated by a palpable pulse corresponding to each paced QRS complex on the cardiac monitor, preferably the femoral pulse to avoid the muscular contractions triggered by the pacer near the carotid artery, which may be confused with a pulse [32]. POCUS may be used to confirm myocardial contractions corresponding to each pacing spike and in confirming femoral pulse. In cases of cardiac arrest during transcutaneous pacing, chest compressions can be safely performed over the pacing pads [32].

Transvenous pacing has a high success rate (>95%) and is preserved for unstable bradyarrhythmias refractory to medications and transcutaneous pacing [6]. Transvenous pacing is contraindicated in patients with severe hypothermia [32]. Transvenous pacing is performed by introducing a transvenous pacing catheter into the right ventricle through a central venous catheterization (either right internal jugular or left subclavian central line). Pacemaker wire may be advanced into the endocardial wall of the right ventricle either blindly, or under the guidance of ECG or ultrasound (four-chamber view) [32]. Observe the cardiac monitor during the advancement of the wire [32]. Capture is confirmed by pacer spikes followed by QRS and ST-segment elevation, which indicates proper positioning [32]. Set the pacer generator on full-demand mode, with an output of 5 mA and a rate of 80 beats/min or at least 10 beats/min faster than the underlying ventricular rhythm [32]. Afterward, confirm electrical and mechanical capture. Continuous electrocardiographic monitoring is recommended for all patients on pacing (both transcutaneous and transvenous) until pacing is discontinued (class I) [42].

Patients with structural or electrophysiological conduction abnormalities often require definitive management with permanent pacemaker implantation. Class I recommendations have been issued for the implantation of permanent pacemakers in patients with acquired Mobitz type II AV block, high-grade AV block, and third-degree AV block not caused by reversible or physiologic causes, irrelevant of the presence of symptoms, due to the high risk of decompensation [7]. The decision to place a permanent pacemaker in patients with symptomatic sinus node dysfunction, persistent and symptomatic sinus bradycardia, and atrial fibrillation with symptomatic bradycardia in the absence of nodal blocking medications is dependent on the presence of symptoms and its correlation with the block itself [7].

Treatment of underlying etiology

Ischemia-related bradycardia

Patients with symptomatic bradyarrhythmia secondary to acute MI require stabilization and immediate reperfusion therapy, either PCI or thrombolysis, depending on feasibility.

Toxicity-related bradycardia

Patients with toxic or metabolic causes of bradycardia respond poorly to atropine and electrical pacing, and those patients require immediate treatment of the underlying cause, by eliminating the offending agent, utilizing supportive care, and administering an agent-specific antidote, if available [7]. Consulting toxicology and/or the local poison center early is paramount. Table 8 presents specific treatment strategies for toxicities related to bradycardia. In cardiac arrest secondary to an overdose, extracorporeal membrane oxygenation (ECMO) might be indicated to maintain perfusion until the underlying agent level is reduced or eliminated [6].

Table 8. Management of toxicologic causes of bradycardia.

Toxicity

Treatment

Beta-blocker toxicity

  • High-dose insulin 1 unit/kg IV bolus, followed by an infusion of 0.5 units/kg/h co-administered with dextrose 7,24,111
  • Glucagon 3-10 mg IV bolus over 3-5 min, followed by an IV infusion of 1-5 mg/h 7,96
  • Intravenous lipid emulsion112

Calcium channel blockers toxicity

  • 10% Calcium chloride 1-2 g IV bolus every 10-20 min or an infusion of 0.2-0.4 mL/kg/h7,113]
  • 10% calcium gluconate 3-6 g IV every 10-20 min or an infusion at 0.6-1.2 mL/kg/h7
  • High-dose insulin 1 unit/kg bolus, followed by an infusion of 0.5 units/kg/h co-administered with dextrose7,24,113
  • Glucagon 3-10 mg IV bolus over 3-5 min, followed by an IV infusion of 1-5 mg/h7,96
  • Intravenous lipid emulsion113

Digoxin toxicity

  • Digoxin-specific antibody fragments (dosage is dependent on amount of ingestion or serum level and whether acute or chronic toxicity)7. It is indicated in unstable bradydysrhythmias or K+ >5.0.

Organophosphate poisoning

  • Decontamination.
  • Atropine 1-3 mg IV every 5 min (0.01-0.04 mg/kg IV in children) with doubling the dose each time, followed by IV maintenance of 0.4-4 mg/hr.
  • Pralidoxime 1-2 g IM (20-40 mg/kg in children) with normal saline infused over 5-10 min followed by IV infusion of 500 mg/h (5-10 mg/kg/h in children).

Local anesthetic systemic toxicity

  • Intravenous lipid emulsion, initial bolus of 100 mL IV over 2–3 min, followed by an infusion of 200–250 mL IV over 15–20 min (for those <70 kg, the bolus dose is 1.5 mL/kg IV over 2–3 min, followed by 0.25 mL/kg/min IV infusion)114,115

Opioids toxicity

  • Naloxone 0.4-2 mg IV/IM/SC bolus repeated every 3 min (pediatrics: 0.1 mg/kg IV/IO/ET75).

Abbreviations: ET, endotracheal; ; g, gram; h, hour; IM, intramuscular; IO, intraosseous;  IV, intravenous; kg, kilogram; mg, milligram; min, minutes; mL, milliliter; mcg, microgram; SC, subcutaneous.

Hyperkalemia-related bradycardia treatment
Hyperkalemia may lead to profound bradycardia and mimic AV blocks [116]. Treatment should include the administration of 2 g calcium gluconate IV or 1 g calcium chloride IV to stabilize the cardiac membrane, in addition to treatments that shift the potassium across the cellular membrane and enhance its elimination.
BRASH syndrome treatment

Treating patients with BRASH syndrome involves a simultaneous approach that targets all associated conditions. The treatment strategy includes usual care of bradycardia (medications [such as epinephrine infusion] and/or pacing), hyperkalemia therapy (IV calcium, IV insulin and dextrose, and/or emergent dialysis), and fluid resuscitation [30]. In addition, it may include further advanced therapies in refractory cases or patients with AV-nodal blocking medication toxicity (e.g., lipid emulsion, glucagon, high-dose insulin, and digoxin-specific antibody) [30].

Hypothermia-related bradycardia treatment

In hypothermia, the first management line is rewarming, even before pacing. Due to the arrhythmogenic effect of hypothermia, pacing severely hypothermic patients has not been recommended due to concerns of precipitating ventricular fibrillation [23, 117]; however, case reports of successful pacing have been reported [118]. ECMO might be considered for severe hypothermia (<32 °C) and cardiac instability [119,120].

Myxedema coma-related bradycardia treatment

Patients with bradycardia secondary to myxedema coma require emergent thyroid hormone replacement (Levothyroxine [T4] 200-400 mcg IV, lower dose in geriatric patients) [121,122]. Adjunctive therapy includes hydrocortisone (100 mg IV), correction of hypoglycemia and electrolyte abnormalities (such as hyponatremia) and supportive care [121,122]. Moreover, identify and treat triggers causing decompensated hypothyroidism (e.g., infection, medications, MI, heart failure, and GI bleeding) [121]. 

Elevated ICP-related bradycardia treatment
For patients with bradycardia secondary to head trauma and increased ICP, treatment should be directed to lower the ICP, such as head elevation, hyperventilation, and mannitol or hypertonic saline.

Special Patient Groups

Considerations of bradycardia in pediatrics

The initial assessment of children is unique. The Pediatric Assessment Triangle is a rapid assessment tool to identify patients with respiratory or circulatory compromise who require immediate stabilization, and stands for appearance, breathing, and circulatory status [123, 124]. This is followed by a primary assessment using the ABCDE approach, similar to adults.

Clinically significant bradycardia in pediatric patients is defined as a heart rate less than the age-appropriate rate with impaired systemic perfusion [125]. The heart rate definition of pediatrics differs based on age, and in infants, an asleep heart rate has a different cut-off than an awake heart rate [126].

In pediatrics, always consider bradycardia as secondary to a reversible cause until proven otherwise. Bradycardia in the pediatric age group is usually associated with hypoxia (the most common cause), hypotension, acidosis, hypothermia, and medications, whereas primarily cardiac causes are rare [74,126].

Bradyarrhythmia in pediatrics is commonly a pre-arrest rhythm125; therefore, an early aggressive approach in children with bradycardia with poor perfusion has been recommended [127]. Immediate evaluation of adequate oxygenation and ventilation is necessary. In patients with a persistent heart rate of <60 and poor perfusion despite adequate oxygenation and ventilation, start chest compressions and follow the pediatric advanced life support (PALS) bradycardia algorithm [74,127], even if there is a detectable pulse.

Considerations for bradycardia in geriatrics

Older adults have a relatively high incidence of symptomatic bradycardia (6%), admission rate for symptomatic bradycardia (39%), rate of unstable bradycardia (16%), and mortality (5%) [128]. In older adults, ischemia tends to present atypically; thus, clinicians should have a high index of suspicion for an underlying ischemic cause. Moreover, polypharmacy is more common in older adults, thereby increasing the risk of drug-induced bradycardia and drug interactions.

Considerations of bradycardia in pregnancy

In critically ill pregnant patients, no medication should be withheld due to concerns of fetal teratogenicity [129,130]. Atropine crosses the placenta and risks and benefits of its use in stable patients should carefully weighed [131]. In unstable patients, atropine or epinephrine may be administered [132]. Epinephrine is preferred over dopamine in pregnancy [133].

Pregnant patients with symptomatic bradycardia necessitating atropine or vasoactive agents (e.g., epinephrine or dopamine) or those with advanced heart block require a multidisciplinary team, including obstetricians and neonatologists for maternal–fetal intensive monitoring [129], which may require transfer.

Bradycardia in patients with a heart transplant

Resting heart rate in heart transplant recipients ranges from 80-110 beats/min [134,135]. Therefore, bradycardia in these patients is defined as a heart rate persistently <70-80 beats/min [7,18,136]. Post-transplant bradycardia could be attributed to several mechanisms, including sympathetic denervation, SA ischemic injury, graft ischemia, and drug-induced [137-139].

Atropine is contraindicated due to the potential risk of paradoxical AV block and asystole [7,49]. Medications used to increase the heart rate include isoproterenol, aminophylline, and theophylline [7,61,140]. Target heart rate for temporary pacing, if indicated, is over 90 beats/min [140].

Following stabilization, those patients may require transport to an advanced heart transplant center for monitoring. It is recommended (Class I) to treat sinus node dysfunction medically in the postoperative period (1-6 weeks) and observe for resolution before attempting pacemaker implantation [21], which is eventually required in 7-24% of patients [136,141,144].

Disposition

All patients with symptomatic bradyarrhythmias require cardiology consultation [23]. Patients presenting with unstable or symptomatic bradyarrhythmias, particularly Mobitz type II or third-degree heart block, require admission to a cardiac ICU for monitoring.  Patients with secondary bradycardia often require admission for definitive treatment of the underlying etiology, either to the ICU or an intermediate care unit (i.e., stepdown). On the other hand, asymptomatic patients with benign ECG features and a normal ED work-up may be followed by cardiology on an outpatient basis; ambulatory electrocardiography monitoring and/or electrophysiology studies may be considered.

Revisiting your patient

The patient was rushed into a monitored bed. IV lines were established. The patient was placed on pacer pads. Atropine 1 mg IV was given with no response. ECG confirmed evidence of third-degree heart block (complete dissociation between P waves and QRS complexes, wide QRS complexes [QRS=154 ms], atrial rate of ~100 beats/min, Ventricular rate of ~30 beats/min, rhythm is maintained by a junctional escape rhythm). Epinephrine and transcutaneous pacing were initiated. Electrical capture was demonstrated on the ECG. The pacing rate was 85 beats/min.

The patient returned to his baseline mental state. Capillary refill improved, extremities became warm, the peripheral pulse became strong, and lactate clearance was appropriate.  Central line access was obtained to transition the patient into transvenous pacing. Cardiac biomarkers resulted, revealing a significantly elevated troponin level at seven times the upper limit of normal. The patient was diagnosed with non-ST elevation occlusive MI. Cardiology was consulted, and they admitted the patient to the cardiac ICU. The patient was taken to the cardiac catheterization laboratory for PCI the following day.

Authors

Picture of Hassan M. Alshaqaq, MBBS

Hassan M. Alshaqaq, MBBS

Hassan Alshaqaq is an Emergency Medicine PGY1 Resident at King Saud University Medical City. He is passionate about EM, research, medical education, resuscitation, and critical care. His research work has appeared in various medical journals and has been awarded by EM and critical care societies. He has been involved with several medical societies in different leadership and educational roles. He is interested in developing clinical practice guidelines and has contributed to the Saudi Critical Care Society guidelines. He is the student club president of the Saudi Society of EM. He is an enthusiast in contributing to EM education, particularly FOAMed.

Picture of Danya Khoujah, MBBS, MEHP

Danya Khoujah, MBBS, MEHP

Dr. Danya Khoujah is an American board-certified Emergency Physician with a keen interest in medical education. She completed her emergency medicine residency and faculty development fellowship at the University of Maryland in Baltimore and a Master of Education in Health Professions from Johns Hopkins University. She has developed over 120 lectures, 75 podcasts, and 50 publications on various emergency medicine and medical education topics that have been well received. She is most passionate about simplifying the science to allow healthcare practitioners to better care for their patients, whether seasoned physicians, resident physicians-in-training, medical students, or allied health professionals.

Listen to the chapter

References

  1. Neumar RW, Otto CW, Link MS, et al. Part 8: Adult Advanced Cardiovascular Life Support. Circulation. 2010;122(18_suppl_3). doi:10.1161/CIRCULATIONAHA.110.970988
  2. Deal N. Evaluation and management of bradydysrhythmias in the emergency department. Emerg Med Pract. 2013;15(9):1-15.
  3. Schwartz B, Vermeulen MJ, Idestrup C, Datta P. Clinical variables associated with mortality in out-of-hospital patients with hemodynamically significant bradycardia. Academic emergency medicine. 2004;11(6):656-661.
  4. Northcote RJ, Canning GP, Ballantyne D. Electrocardiographic findings in male veteran endurance athletes. Heart. 1989;61(2):155-160. doi:10.1136/hrt.61.2.155
  5. Talan DA, Bauernfeind RA, Ashley WW, Kanakis C, Rosen KM. Twenty-Four Hour Continuous ECG Recordings in Long-Distance Runners. Chest. 1982;82(1):19-24. doi:10.1378/chest.82.1.19
  6. Sodeck GH, Domanovits H, Meron G, et al. Compromising bradycardia: Management in the emergency department. Resuscitation. 2007;73(1):96-102. doi:10.1016/j.resuscitation.2006.08.006
  7. Kusumoto FM, Schoenfeld MH, Barrett C, et al. 2018 ACC/AHA/HRS Guideline on the Evaluation and Management of Patients With Bradycardia and Cardiac Conduction Delay: A Report of the American College of Cardiology/American Heart Association Task Force on Clinical Practice Guidelines and the Heart Rhythm Society. Circulation. 2019;140(8):382-482. doi:10.1161/CIR.0000000000000628
  8. Pejković B, Krajnc I, Anderhuber F, Košutić D. Anatomical Aspects of the Arterial Blood Supply to the Sinoatrial and Atrioventricular Nodes of the Human Heart. Journal of International Medical Research. 2008;36(4):691-698. doi:10.1177/147323000803600410
  9. Perron AD, Sweeney T. Arrhythmic Complications of Acute Coronary Syndromes. Emerg Med Clin North Am. 2005;23(4):1065-1082. doi:10.1016/j.emc.2005.07.002
  10. Wung SF. Bradyarrhythmias. Crit Care Nurs Clin North Am. 2016;28(3):297-308. doi:10.1016/j.cnc.2016.04.003
  11. Brady WJ, Swart G, DeBehnke DJ, Ma OJ, Aufderheide TP. The efficacy of atropine in the treatment of hemodynamically unstable bradycardia and atrioventricular block: prehospital and emergency department considerations. Resuscitation. 1999;41(1):47-55. doi:10.1016/S0300-9572(99)00032-5
  12. Swart G, Brady WJ, DeBehnke DJ, John O, Aufderheide TP. Acute myocardial infarction complicated by hemodynamically unstable bradyarrhythmia: Prehospital and ED treatment with atropine. Am J Emerg Med. 1999;17(7):647-652. doi:10.1016/S0735-6757(99)90151-1
  13. Norris RM, Mercer CJ. Significance of idioventricular rhythms in acute myocardial infarction. Prog Cardiovasc Dis. 1974;16(5):455-468. doi:10.1016/0033-0620(74)90006-1
  14. Antman EM, Anbe DT, Armstrong PW, et al. ACC/AHA Guidelines for the Management of Patients With ST-Elevation Myocardial Infarction—Executive Summary. Circulation. 2004;110(5):588-636. doi:10.1161/01.CIR.0000134791.68010.FA
  15. Tintinalli JE, Ma OJ, Yealy DM, et al. Tintinalli’s Emergency Medicine. Ninth. McGraw-Hill Education; 2019.
  16. Song Y, Laaksonen H, Saukko P, Toivonen S, Zhu J. Histopathological findings of cardiac conduction system of 150 Finns. Forensic Sci Int. 2001;119(3):310-317. doi:10.1016/S0379-0738(00)00457-6
  17. Bharati S, Lev M. The pathologic changes in the conduction system beyond the age of ninety. Am Heart J. 1992;124(2):486-496. doi:10.1016/0002-8703(92)90615-3
  18. HEINZ G, HIRSCHL M, BUXBAUM P, LAUFER G, GASIC S, LACZKOVICS A. Sinus Node Dysfunction After Orthotopic Cardiac Transplantation: Postoperative Incidence and Long-Term Implications. Pacing and Clinical Electrophysiology. 1992;15(5):731-737. doi:10.1111/j.1540-8159.1992.tb06838.x
  19. Demoulin JC, Kulbertus HE. Histopathological correlates of sinoatrial disease. Heart. 1978;40(12):1384-1389. doi:10.1136/hrt.40.12.1384
  20. Shaw DB, Linker NJ, Heaver PA, Evans R. Chronic sinoatrial disorder (sick sinus syndrome): a possible result of cardiac ischaemia. Heart. 1987;58(6):598-607. doi:10.1136/hrt.58.6.598
  21. Brignole M, Auricchio A, Baron-Esquivias G, et al. 2013 ESC Guidelines on cardiac pacing and cardiac resynchronization therapy: The Task Force on cardiac pacing and resynchronization therapy of the European Society of Cardiology (ESC). Developed in collaboration with the European Heart Rhythm Association (EHRA). Europace. 2013;15(8):1070-1118. doi:10.1093/europace/eut206
  22. Sathnur N, Ebin E, Benditt DG. Sinus Node Dysfunction. Card Electrophysiol Clin. 2021;13(4):641-659. doi:10.1016/j.ccep.2021.06.006
  23. Walls RM, Hockberger RS, Gausche-Hill M, Erickson TB, Wilcox SR. Rosen’s Emergency Medicine. Tenth. Philadelphia: Elsevier; 2022.
  24. Tisdale JE, Chung MK, Campbell KB, et al. Drug-Induced Arrhythmias: A Scientific Statement From the American Heart Association. Circulation. 2020;142(15):214-233. doi:10.1161/CIR.0000000000000905
  25. Fuster V, Rydén LE, Asinger RW, et al. ACC/AHA/ESC Guidelines for the Management of Patients With Atrial Fibrillation: Executive Summary A Report of the American College of Cardiology/American Heart Association Task Force on Practice Guidelines and the European Society of Cardiology Committee for Practice Guidelines and Policy Conferences (Committee to Develop Guidelines for the Management of Patients With Atrial Fibrillation) Developed in Collaboration With the North American Society of Pacing and Electrophysiology.Circulation. 2001;104(17):2118-2150.
  26. January CT, Wann LS, Alpert JS, et al. 2014 AHA/ACC/HRS Guideline for the Management of Patients With Atrial Fibrillation. J Am Coll Cardiol. 2014;64(21):e1-e76. doi:10.1016/j.jacc.2014.03.022
  27. Gilligan DM, Ellenbogen KA, Epstein AE. The management of atrial fibrillation. Am J Med. 1996;101(4):413-421. doi:10.1016/S0002-9343(96)00194-5
  28. Littmann L, Gibbs MA. Electrocardiographic manifestations of severe hyperkalemia. J Electrocardiol. 2018;51(5):814-817. doi:10.1016/j.jelectrocard.2018.06.018
  29. Drumheller BC, Tuffy E, Gibney F, Stallard S, Siewers C, Korvek S. Severe bradycardia from severe hyperkalemia: Patient characteristics, outcomes and factors associated with hemodynamic support. Am J Emerg Med. 2022;55:117-125. doi:10.1016/j.ajem.2022.03.007
  30. Farkas JD, Long B, Koyfman A, Menson K. BRASH Syndrome: Bradycardia, Renal Failure, AV Blockade, Shock, and Hyperkalemia. J Emerg Med. 2020;59(2):216-223. doi:10.1016/j.jemermed.2020.05.001
  31. Brady WJ, Harrigan RA. Evaluation and management of bradyarrhythmias in the emergency department. Emerg Med Clin North Am. 1998;16(2):361-388. doi:10.1016/S0733-8627(05)70007-9
  32. Roberts JR. Roberts and Hedges’ Clinical Procedures in Emergency Medicine and Acute Care. Seventh. Philadelphia: Elsevier; 2018.
  33. Morrison LJ, Long J, Vermeulen M, et al. A randomized controlled feasibility trial comparing safety and effectiveness of prehospital pacing versus conventional treatment: ‘PrePACE.’ Resuscitation. 2008;76(3):341-349. doi:10.1016/j.resuscitation.2007.08.008
  34. Sherbino J, Verbeek PR, MacDonald RD, Sawadsky B V., McDonald AC, Morrison LJ. Prehospital transcutaneous cardiac pacing for symptomatic bradycardia or bradyasystolic cardiac arrest: A systematic review. Resuscitation. 2006;70(2):193-200. doi:10.1016/j.resuscitation.2005.11.019
  35. Barthell E, Troiano P, Olson D, Stueven HA, Hendley G. Prehospital external cardiac pacing: A prospective, controlled clinical trial. Ann Emerg Med. 1988;17(11):1221-1226. doi:10.1016/S0196-0644(88)80074-X
  36. Hedges JR, Feero S, Shultz B, Easter R, Syverud SA, Dalsey WC. Prehospital Transcutaneous Cardiac Pacing for Symptomatic Bradycardia. Pacing and Clinical Electrophysiology. 1991;14(10):1473-1478. doi:10.1111/j.1540-8159.1991.tb04068.x
  37. Hedges JR, Syverud SA, Dalsey WC, Feero S, Easter R, Shultz B. Prehospital trial of emergency transcutaneous cardiac pacing. Circulation. 1987;76(6):1337-1343. doi:10.1161/01.CIR.76.6.1337
  38. Farkas J. Bradycardia. EMCrit. https://emcrit.org/ibcc/bradycardia/. Published November 20, 2021. Accessed April 1, 2023.
  39. Alblaihed L, Tewelde S. Bradydysrhythmias. CorePendium. https://www.emrap.org/corependium/chapter/recSdHpVvdD2oAbVe/Bradydysrhythmias. Published July 14, 2021. Accessed April 1, 2023.
  40. Tian DH, Smyth C, Keijzers G, et al. Safety of peripheral administration of vasopressor medications: A systematic review. Emergency Medicine Australasia. 2020;32(2):220-227. doi:10.1111/1742-6723.13406
  41. Helman A, Hedayati T, Dorian P. 4-Step Approach to Bradycardia and Bradydysrhythmias. Emergency Medicine Cases. https://emergencymedicinecases.com/approach-bradycardia-bradydysrhythmias/. Published 2018. Accessed April 1, 2023.
  42. Sandau KE, Funk M, Auerbach A, et al. Update to Practice Standards for Electrocardiographic Monitoring in Hospital Settings: A Scientific Statement From the American Heart Association. Circulation. 2017;136(19):e273-e344. doi:10.1161/CIR.0000000000000527
  43. Scheinman MM, Thorburn D, Abbott JA. Use of atropine in patients with acute myocardial infarction and sinus bradycardia. Circulation. 1975;52(4):627-633. doi:10.1161/01.CIR.52.4.627
  44. Richman S. Adverse effect of atropine during myocardial infarction. Enchancement of ischemia following intravenously administered atropine. JAMA. 1974;228(11):1414-1416.
  45. Dauchot P, Gravenstein JS. Bradycardia after Myocardial Ischemia and Its Treatment with Atropine. Anesthesiology. 1976;44(6):501-518. doi:10.1097/00000542-197606000-00008
  46. Massumi RA, Mason DT, Amsterdam EA, et al. Ventricular Fibrillation and Tachycardia after Intravenous Atropine for Treatment of Bradycardias. New England Journal of Medicine. 1972;287(7):336-338. doi:10.1056/NEJM197208172870706
  47. Pentecost BL, Bennett MA, George CF. Bradyarrhythmia Complicating Myocardial Infarction. The Lancet. 1968;292(7581):1300-1301. doi:10.1016/S0140-6736(68)91793-5
  48. Maroko PR, Kjekshus JK, Sobel BE, et al. Factors Influencing Infarct Size Following Experimental Coronary Artery Occlusions. Circulation. 1971;43(1):67-82. doi:10.1161/01.CIR.43.1.67
  49. Bernheim A, Fatio R, Kiowski W, Weilenmann D, Rickli H, Rocca HPBL. Atropine often results in complete atrioventricular block or sinus arrest after cardiac transplantation: an unpredictable and dose-independent phenomenon. Transplantation. 2004;77(8):1181-1185. doi:10.1097/01.TP.0000122416.70287.D9
  50. Rocca HPBL, Kiowski W, Bracht C, Weilenmann D, Follath F. Atrioventricular block after administration of atropine in patients following cardiac transplantation. Transplantation. 1997;63(12):1838-1839. doi:10.1097/00007890-199706270-00023
  51. Morady F, Nelson SD, Kou WH, et al. Electrophysiologic effects of epinephrine in humans. J Am Coll Cardiol. 1988;11(6):1235-1244. doi:10.1016/0735-1097(88)90287-2
  52. Tisdale JE, Patel R V., Webb CR, Borzak S, Zarowitz BJ. Proarrhythmic Effects of Intravenous Vasopressors. Annals of Pharmacotherapy. 1995;29(3):269-281. doi:10.1177/106002809502900309
  53. Panchal AR, Bartos JA, Cabañas JG, et al. Part 3: Adult Basic and Advanced Life Support: 2020 American Heart Association Guidelines for Cardiopulmonary Resuscitation and Emergency Cardiovascular Care.Circulation. 2020;142(16_suppl_2):S366-S468. doi:10.1161/CIR.0000000000000916
  54. Holden D, Ramich J, Timm E, Pauze D, Lesar T. Safety Considerations and Guideline-Based Safe Use Recommendations for “Bolus-Dose” Vasopressors in the Emergency Department. Ann Emerg Med. 2018;71(1):83-92. doi:10.1016/j.annemergmed.2017.04.021
  55. Hollenberg SM. Vasoactive Drugs in Circulatory Shock. Am J Respir Crit Care Med. 2011;183(7):847-855. doi:10.1164/rccm.201006-0972CI
  56. Cossú SF, Rothman SA, Chmielewski IL, et al. The Effects of Isoproterenol on the Cardiac Conduction System. J Cardiovasc Electrophysiol. 1997;8(8):847-853. doi:10.1111/j.1540-8167.1997.tb00845.x
  57. Duong H, Masarweh OM, Campbell G, Win TT, Joolhar F. Isoproterenol Causing Coronary Vasospasm and ST Elevations During Tilt Table Testing. J Investig Med High Impact Case Rep. 2020;8. doi:10.1177/2324709620966862
  58. Ferrans VJ, Hibbs RG, Black WC, Weilbaecher DG. Isoproterenol-induced myocardial necrosis. A histochemical and electron microscopic study. Am Heart J. 1964;68(1):71-90. doi:10.1016/0002-8703(64)90242-X
  59. Kurland G, Williams J, Lewiston N. Fatal myocardial toxicity during continuous infusion intravenous isoproterenol therapy of asthma. Journal of Allergy and Clinical Immunology. 1979;63(6):407-411. doi:10.1016/0091-6749(79)90214-8
  60. Becker DJ, Nonkin PM, Bennett LD, Kimball SG, Sternberg MS, Wasserman F. Effect of isoproterenol in digitalis cardiotoxicity. Am J Cardiol. 1962;10(2):242-247. doi:10.1016/0002-9149(62)90302-8
  61. Bertolet BD, Eagle DA, Conti JB, Mills RM, Belardinelli L. Bradycardia after heart transplantation: Reversal with theophylline. J Am Coll Cardiol. 1996;28(2):396-399. doi:10.1016/0735-1097(96)00162-3
  62. Heinz G, Kratochwill C, Buxbaum P, et al. Immediate normalization of profound sinus node dysfunction by aminophylline after cardiac transplantation. Am J Cardiol. 1993;71(4):346-349. doi:10.1016/0002-9149(93)90805-M
  63. Redmond JM, Zehr KJ, Gillinov MA, et al. Use of theophylline for treatment of prolonged sinus node dysfunction in human orthotopic heart transplantation. J Heart Lung Transplant. 1993;12(1):133-139.
  64. Sadaka F, Naydenov SK, Ponzillo JJ. Theophylline for Bradycardia Secondary to Cervical Spinal Cord Injury. Neurocrit Care. 2010;13(3):389-392. doi:10.1007/s12028-010-9454-y
  65. Sakamoto T, Sadanaga T, Okazaki T. Sequential use of aminophylline and theophylline for the treatment of atropine-resistant bradycardia after spinal cord injury: a case report. J Cardiol. 2007;49(2):91-96.
  66. Pasnoori VR, Leesar MA. Use of Aminophylline in the Treatment of Severe Symptomatic Bradycardia Resistant to Atropine. Cardiol Rev. 2004;12(2):65-68. doi:10.1097/01.crd.0000096418.72821.fa
  67. Weant KA, Kilpatrick M, Jaikumar S. Aminophylline for the treatment of symptomatic bradycardia and asystole secondary to cervical spine injury. Neurocrit Care. 2007;7(3):250-252. doi:10.1007/s12028-007-0067-z
  68. Bertolet BD, McMurtrie E, Hill J, Belardinelli L. Theophylline for the Treatment of Atrioventricular Block after Myocardial Infarction. Ann Intern Med. 1995;123(7):509-511. doi:10.7326/0003-4819-123-7-199510010-00006
  69. Goodfellow J, Walker PR. Reversal of atropine-resistant atrioventricular block with intravenous aminophylline in the early phase of inferior wall acute myocardial infarction following treatment with streptokinase. Eur Heart J. 1995;16(6):862-865. doi:10.1093/oxfordjournals.eurheartj.a061008
  70. Altun A, Kirdar C, Özbay G. Effect of aminophylline in patients with atropine-resistant late advanced atrioventricular block during acute inferior myocardial infarction. Clin Cardiol. 1998;21(10):759-762. doi:10.1002/clc.4960211012
  71. Gorten R, Gunnells JC, Weissler AM, Stead EA. Effects of Atropine and Isoproterenol on Cardiac Output, Central Venous Pressure, and Mean Transit Time of Indicators Placed at Three Different Sites in the Venous System. Circ Res. 1961;9(5):979-983. doi:10.1161/01.RES.9.5.979
  72. Hinderling PH, Gundert-Remy U, Schmidliny O, Heinzel G. Integrated Pharmacokinetics and Pharmacodynamics of Atropine in Healthy Humans II: Pharmacodynamics. J Pharm Sci. 1985;74(7):711-717. doi:10.1002/jps.2600740703
  73. Berry JN, Thompson HK, Miller DE, McIntosh HD. Changes in cardiac output, stroke volume, and central venous pressure induced by atropine in man. Am Heart J. 1959;58(2):204-213. doi:10.1016/0002-8703(59)90337-0
  74. Topjian AA, Raymond TT, Atkins D, et al. Part 4: Pediatric Basic and Advanced Life Support: 2020 American Heart Association Guidelines for Cardiopulmonary Resuscitation and Emergency Cardiovascular Care. Circulation. 2020;142(16_suppl_2):S469-S523. doi:10.1161/CIR.0000000000000901
  75. Kleinman ME, Chameides L, Schexnayder SM, et al. Part 14: Pediatric Advanced Life Support. Circulation. 2010;122(18_suppl_3):S876-908. doi:10.1161/CIRCULATIONAHA.110.971101
  76. Lönnerholm G, Widerlöv E. Effect of intravenous atropine and methylatropine on heart rate and secretion of saliva in man. Eur J Clin Pharmacol. 1975;8(3-4):233-240. doi:10.1007/BF00567121
  77. Santini M, Ammirati F, Colivicchi F, Gentilucci G, Guido V. The effect of atropine in vasovagal syncope induced by head-up tilt testing. Eur Heart J. 1999;20(23):1745-1751. doi:10.1053/euhj.1999.1697
  78. Kentala E, Kaila T, Iisalo E, Kanto J. Intramuscular atropine in healthy volunteers: a pharmacokinetic and pharmacodynamic study. Int J Clin Pharmacol. 1990;28(9):399-404.
  79. Volz-Zang C, Waldhäuser T, Schulte B, Palm D. Comparison of the effects of atropine in vivo and ex vivo (radioreceptor assay) after oral and intramuscular administration to man. Eur J Clin Pharmacol. 1995;49(1-2):45-49. doi:10.1007/BF00192357
  80. Hinderling PH, Gundert-Remy U, Schmidlin O. Integrated pharmacokinetics and pharmacodynamics of atropine in healthy humans. I: Pharmacokinetics. J Pharm Sci. 1985;74(7):703-710. doi:10.1002/jps.2600740702
  81. Adams RG, Verma P, Jackson AJ, Miller RL. Plasma pharmacokinetics of intravenously administered atropine in normal human subjects. J Clin Pharmacol. 1982;22(10):477-481. doi:10.1002/j.1552-4604.1982.tb02638.x
  82. Van der Meer MJ, Hundt HK, Müller FO. The metabolism of atropine in man. J Pharm Pharmacol. 1986;38(10):781-784. doi:10.1111/j.2042-7158.1986.tb04494.x
  83. Kalser SC, McLain PL. Atropine metabolism in man. Clin Pharmacol Ther. 1970;11(2):214-227. doi:10.1002/cpt1970112214
  84. MacGregor DA, Smith TE, Prielipp RC, Butterworth JF, James RL, Scuderi PE. Pharmacokinetics of dopamine in healthy male subjects. Anesthesiology. 2000;92(2):338-346. doi:10.1097/00000542-200002000-00013
  85. Horwitz D, SM FD, Goldberg L. Effects of Dopamine in man. Circ Res. 1962;10:237-243. doi:10.1161/01.res.10.2.237
  86. Gundert-Remy U, Penzien J, Hildebrandt R, Mäurer W, Weber E. Correlation between the pharmacokinetics and pharmacodynamics of dopamine in healthy subjects. Eur J Clin Pharmacol. 1984;26(2):163-169. doi:10.1007/BF00630281
  87. Mueller HS, Evans R, Ayres SM. Effect of dopamine on hemodynamics and myocardial metabolism in shock following acute myocardial infarction in man. Circulation. 1978;57(2):361-365. doi:10.1161/01.cir.57.2.361
  88. Guidelines 2000 for Cardiopulmonary Resuscitation and Emergency Cardiovascular Care. Part 6: advanced cardiovascular life support: section 6: pharmacology II: agents to optimize cardiac output and blood pressure. The American Heart Association in collaboration with the International Liaison Committee on Resuscitation. Circulation. 2000;102(8 Suppl):I129-135.
  89. Juste RN, Moran L, Hooper J, Soni N. Dopamine clearance in critically ill patients. Intensive Care Med. 1998;24(11):1217-1220. doi:10.1007/S001340050747
  90. Borne P, Oren R, Somers VK. Dopamine depresses minute ventilation in patients with heart failure. Circulation. 1998;98(2):126-131. doi:10.1161/01.cir.98.2.126
  91. Fellows IW, Bennett T, MacDonald IA. The effect of adrenaline upon cardiovascular and metabolic functions in man. Clin Sci (Lond). 1985;69(2):215-222. doi:10.1042/cs0690215
  92. Stratton JR, Pfeifer MA, Ritchie JL, Halter JB. Hemodynamic effects of epinephrine: concentration-effect study in humans. J Appl Physiol (1985). 1985;58(4):1199-1206. doi:10.1152/jappl.1985.58.4.1199
  93. Abboud I, Lerolle N, Urien S, et al. Pharmacokinetics of epinephrine in patients with septic shock: modelization and interaction with endogenous neurohormonal status. Crit Care. 2009;13(4):R120. doi:10.1186/cc7972
  94. Parmley WW, Glick G, Sonnenblick EH. Cardiovascular effects of glucagon in man. N Engl J Med. 1968;279(1):12-17. doi:10.1056/NEJM196807042790103
  95. Murtagh JG, Binnion PF, Lal S, Hutchison KJ, Fletcher E. Haemodynamic effects of glucagon. Br Heart J. 1970;32(3):307-315. doi:10.1136/hrt.32.3.307
  96. Bailey B. Glucagon in beta-blocker and calcium channel blocker overdoses: a systematic review. J Toxicol Clin Toxicol. 2003;41(5):595-602. doi:10.1081/clt-120023761
  97. Lavonas EJ, Drennan IR, Gabrielli A, et al. Part 10: Special Circumstances of Resuscitation: 2015 American Heart Association Guidelines Update for Cardiopulmonary Resuscitation and Emergency Cardiovascular Care. Circulation. 2015;132(18 Suppl 2):S501-18. doi:10.1161/CIR.0000000000000264
  98. Lvoff R, Wilcken DE. Glucagon in heart failure and in cardiogenic shock. Experience in 50 patients. Circulation. 1972;45(3):534-542. doi:10.1161/01.cir.45.3.534
  99. Love JN, Howell JM. Glucagon therapy in the treatment of symptomatic bradycardia. Ann Emerg Med. 1997;29(1):181-183. doi:10.1016/s0196-0644(97)70327-5
  100. Mansell PI, Fellows IW, Birmingham AT, Macdonald IA. Metabolic and cardiovascular effects of infusions of low doses of isoprenaline in man. Clin Sci (Lond). 1988;75(3):285-291. doi:10.1042/cs0750285
  101. Matsubara S, Morimatsu Y, Shiraishi H, et al. Fetus with heart failure due to congenital atrioventricular block treated by maternally administered ritodrine. Arch Gynecol Obstet. 2008;278(1):85-88. doi:10.1007/s00404-007-0516-0
  102. Castilla M, Jerez M, Llácer M, Martinez S. Anaesthetic management in a neonate with congenital complete heart block. Paediatr Anaesth. 2004;14(2):172-175. doi:10.1046/j.1460-9592.2003.01180.x
  103. Kadar D, Tang HY, Conn AW. Isoproterenol metabolism in children after intravenous administration. Clin Pharmacol Ther. 1974;16(5 Part 1):789-795. doi:10.1002/cpt1974165part1789
  104. Reyes G, Schwartz PH, Newth CJ, Eldadah MK. The pharmacokinetics of isoproterenol in critically ill pediatric patients. J Clin Pharmacol. 1993;33(1):29-34. doi:10.1002/j.1552-4604.1993.tb03899.x
  105. Okuya Y, Park JY, Garg A, Moussa I. Coronary Artery Spasm During Catheter Ablation Caused by the Intravenous Infusion of Isoproterenol. Intern Med. 2021;60(8):1221-1224. doi:10.2169/internalmedicine.6130-20
  106. McMurray JJ V, Adamopoulos S, Anker SD, et al. ESC Guidelines for the diagnosis and treatment of acute and chronic heart failure 2012: The Task Force for the Diagnosis and Treatment of Acute and Chronic Heart Failure 2012 of the European Society of Cardiology. Developed in collaboration with the Heart Failure Association (HFA) of the ESC. Eur Heart J. 2012;33(14):1787-1847. doi:10.1093/eurheartj/ehs104
  107. Ahonen J, Aranko K, Iivanainen A, Maunuksela EL, Paloheimo M, Olkkola KT. Pharmacokinetic-pharmacodynamic relationship of dobutamine and heart rate, stroke volume and cardiac output in healthy volunteers. Clin Drug Investig. 2008;28(2):121-127. doi:10.2165/00044011-200828020-00006
  108. Pentel P, Benowitz N. Pharmacokinetic and pharmacodynamic considerations in drug therapy of cardiac emergencies. Clin Pharmacokinet. 1984;9(4):273-308. doi:10.2165/00003088-198409040-00001
  109. Dellinger RP, Levy MM, Rhodes A, et al. Surviving sepsis campaign: international guidelines for management of severe sepsis and septic shock: 2012. Crit Care Med. 2013;41(2):580-637. doi:10.1097/CCM.0b013e31827e83af
  110. Tisdale JE, Patel R, Webb CR, Borzak S, Zarowitz BJ. Electrophysiologic and proarrhythmic effects of intravenous inotropic agents. Prog Cardiovasc Dis. 1995;38(2):167-180. doi:10.1016/s0033-0620(05)80005-2
  111. Dalsey WC, Syverud SA, Hedges JR. Emergency department use of transcutaneous pacing for cardiac arrests. Crit Care Med. 1985;13(5):399-401. doi:10.1097/00003246-198505000-00006
  112. Rotella JA, Greene SL, Koutsogiannis Z, et al. Treatment for beta-blocker poisoning: a systematic review. Clin Toxicol (Phila). 2020;58(10):943-983. doi:10.1080/15563650.2020.1752918
  113. St-Onge M, Anseeuw K, Cantrell FL, et al. Experts Consensus Recommendations for the Management of Calcium Channel Blocker Poisoning in Adults. Crit Care Med. 2017;45(3):e306-e315. doi:10.1097/CCM.0000000000002087
  114. Neal JM, Neal EJ, Weinberg GL. American Society of Regional Anesthesia and Pain Medicine Local Anesthetic Systemic Toxicity checklist: 2020 version. Reg Anesth Pain Med. 2021;46(1):81-82. doi:10.1136/rapm-2020-101986
  115. Long B, Chavez S, Gottlieb M, Montrief T, Brady WJ. Local anesthetic systemic toxicity: A narrative review for emergency clinicians. Am J Emerg Med. 2022;59:42-48. doi:10.1016/j.ajem.2022.06.017
  116. Noble K, Isles C. Hyperkalaemia causing profound bradycardia. Heart. 2006;92(8):1063. doi:10.1136/hrt.2005.071803
  117. Helman A, Dorian P, Hedayati T. Treatment of Bradycardia and Bradydysrhythmias. Emergency Medicine Cases. https://emergencymedicinecases.com/treatment-bradycardia-bradydysrhythmias/. Published 2021. Accessed April 1, 2023.
  118. Ho JD, Heegaard WG, Brunette DD. Successful transcutaneous pacing in 2 severely hypothermic patients. Ann Emerg Med. 2007;49(5):678-681. doi:10.1016/j.annemergmed.2006.05.014
  119. Brugger H, Durrer B, Elsensohn F, et al. Resuscitation of avalanche victims: Evidence-based guidelines of the international commission for mountain emergency medicine (ICAR MEDCOM): intended for physicians and other advanced life support personnel. Resuscitation. 2013;84(5):539-546. doi:10.1016/j.resuscitation.2012.10.020
  120. Truhlář A, Deakin CD, Soar J, et al. European Resuscitation Council Guidelines for Resuscitation 2015: Section 4. Cardiac arrest in special circumstances. Resuscitation. 2015;95:148-201. doi:10.1016/j.resuscitation.2015.07.017
  121. Bridwell RE, Willis GC, Gottlieb M, Koyfman A, Long B. Decompensated hypothyroidism: A review for the emergency clinician. Am J Emerg Med. 2021;39:207-212. doi:10.1016/j.ajem.2020.09.062
  122. Jonklaas J, Bianco AC, Bauer AJ, et al. Guidelines for the treatment of hypothyroidism: prepared by the american thyroid association task force on thyroid hormone replacement. Thyroid. 2014;24(12):1670-1751. doi:10.1089/thy.2014.0028
  123. Dieckmann RA, Brownstein D, Gausche-Hill M. The pediatric assessment triangle: a novel approach for the rapid evaluation of children. Pediatr Emerg Care. 2010;26(4):312-315. doi:10.1097/PEC.0b013e3181d6db37
  124. Fernández A, Ares MI, Garcia S, Martinez-Indart L, Mintegi S, Benito J. The Validity of the Pediatric Assessment Triangle as the First Step in the Triage Process in a Pediatric Emergency Department. Pediatr Emerg Care. 2017;33(4):234-238. doi:10.1097/PEC.0000000000000717
  125. Jat KR, Lodha R, Kabra SK. Arrhythmias in children. Indian J Pediatr. 2011;78(2):211-218. doi:10.1007/s12098-010-0276-x
  126. Baruteau AE, Perry JC, Sanatani S, Horie M, Dubin AM. Evaluation and management of bradycardia in neonates and children. Eur J Pediatr. 2016;175(2):151-161. doi:10.1007/s00431-015-2689-z
  127. Van de Voorde P, Turner NM, Djakow J, et al. European Resuscitation Council Guidelines 2021: Paediatric Life Support. Resuscitation. 2021;161:327-387. doi:10.1016/j.resuscitation.2021.02.015
  128. Rujichanuntagul S, Sri-On J, Traiwanatham M, et al. Bradycardia in Older Patients in a Single-Center Emergency Department: Incidence, Characteristics and Outcomes. Open Access Emerg Med. 2022;14:147-153. doi:10.2147/OAEM.S351548
  129. ACOG Practice Bulletin No. 211: Critical Care in Pregnancy. Obstetrics and gynecology. 2019;133(5):e303-e319. doi:10.1097/AOG.0000000000003241
  130. Jeejeebhoy FM, Zelop CM, Lipman S, et al. Cardiac Arrest in Pregnancy: A Scientific Statement From the American Heart Association. Circulation. 2015;132(18):1747-1773. doi:10.1161/CIR.0000000000000300
  131. Bailey B. Are there teratogenic risks associated with antidotes used in the acute management of poisoned pregnant women? Birth Defects Res A Clin Mol Teratol. 2003;67(2):133-140. doi:10.1002/bdra.10007
  132. SANDLER M, RUTHREN CR, WOOD C. METABOLISM OF C14-NOREPINEPHRINE AND C14-EPINEPHRINE AND THEIR TRANSMISSION ACROSS THE HUMAN PLACENTA. Int J Neuropharmacol. 1964;3:123-128. doi:10.1016/0028-3908(64)90055-3
  133. Newell J. Dopamine. CorePendium. https://www.emrap.org/corependium/drug/recZPuXuOq4bUMquk/Dopamine#h.t44uk7at4g7p. Published May 12, 2023. Accessed April 1, 2023.
  134. Alexopoulos D, Yusuf S, Johnston JA, Bostock J, Sleight P, Yacoub MH. The 24-hour heart rate behavior in long-term survivors of cardiac transplantation. Am J Cardiol. 1988;61(11):880-884. doi:10.1016/0002-9149(88)90363-3
  135. Banner NR, Patel N, Cox AP, Patton HE, Lachno DR, Yacoub MH. Altered sympathoadrenal response to dynamic exercise in cardiac transplant recipients. Cardiovasc Res. 1989;23(11):965-972. doi:10.1093/cvr/23.11.965
  136. Woo GW, Schofield RS, Pauly DF, et al. Incidence, predictors, and outcomes of cardiac pacing after cardiac transplantation: an 11-year retrospective analysis. Transplantation. 2008;85(8):1216-1218. doi:10.1097/TP.0b013e31816b677c
  137. Thajudeen A, Stecker EC, Shehata M, et al. Arrhythmias after heart transplantation: mechanisms and management. J Am Heart Assoc. 2012;1(2):e001461. doi:10.1161/JAHA.112.001461
  138. Jacquet L, Ziady G, Stein K, et al. Cardiac rhythm disturbances early after orthotopic heart transplantation: prevalence and clinical importance of the observed abnormalities. J Am Coll Cardiol. 1990;16(4):832-837. doi:10.1016/s0735-1097(10)80330-4
  139. DiBiase A, Tse TM, Schnittger I, Wexler L, Stinson EB, Valantine HA. Frequency and mechanism of bradycardia in cardiac transplant recipients and need for pacemakers. Am J Cardiol. 1991;67(16):1385-1389. doi:10.1016/0002-9149(91)90469-2
  140. Costanzo MR, Dipchand A, Starling R, et al. The International Society of Heart and Lung Transplantation Guidelines for the care of heart transplant recipients. J Heart Lung Transplant. 2010;29(8):914-956. doi:10.1016/j.healun.2010.05.034
  141. Cantillon DJ, Tarakji KG, Hu T, et al. Long-term outcomes and clinical predictors for pacemaker-requiring bradyarrhythmias after cardiac transplantation: analysis of the UNOS/OPTN cardiac transplant database. Heart Rhythm. 2010;7(11):1567-1571. doi:10.1016/j.hrthm.2010.06.026
  142. Cantillon DJ, Gorodeski EZ, Caccamo M, et al. Long-term outcomes and clinical predictors for pacing after cardiac transplantation. J Heart Lung Transplant. 2009;28(8):791-798. doi:10.1016/J.HEALUN.2009.04.034
  143. Jones DG, Mortsell DH, Rajaruthnam D, et al. Permanent pacemaker implantation early and late after heart transplantation: clinical indication, risk factors and prognostic implications. J Heart Lung Transplant. 2011;30(11):1257-1265. doi:10.1016/j.healun.2011.05.010
  144. Wellmann P, Herrmann FEM, Hagl C, Juchem G. A Single Center Study of 1,179 Heart Transplant Patients-Factors Affecting Pacemaker Implantation. Pacing Clin Electrophysiol. 2017;40(3):247-254. doi:10.1111/pace.13021

Reviewed By

Picture of Arif Alper Cevik, MD, FEMAT, FIFEM

Arif Alper Cevik, MD, FEMAT, FIFEM

Prof Cevik is an Emergency Medicine academician at United Arab Emirates University, interested in international emergency medicine, emergency medicine education, medical education, point of care ultrasound and trauma. He is the founder and director of the International Emergency Medicine Education Project – iem-student.org, chair of the International Federation for Emergency Medicine (IFEM) core curriculum and education committee and board member of the Asian Society for Emergency Medicine and Emirati Board of Emergency Medicine.

Question Of The Day #90

question of the day
366 - pneumonia-middle lobe

Which of the following is the most likely cause of this patient’s condition?

Shortness of breath, also known as dyspnea, is a common reason for patients to visit the Emergency Department.  Dyspnea is often caused by a pulmonary or cardiovascular condition, but it is important to remember that dyspnea can be due to endocrine conditions, toxicologic conditions, neurologic conditions, hematologic conditions, musculoskeletal conditions, and psychiatric conditions. 

The initial approach to all patients with shortness of breath involves the primary survey, or “ABCs” (Airway, Breathing, Circulation).  This first involves checking the patient for a patent airway.  A simple method to assess the airway is to ask the patient to speak and listen for the voice.  A muffled voice, the presence of stridor, hematemesis, or a lethargic patient are clues that a patent airway may not be present.  Problems with the airway, such as an obstructing foreign body, inflammation (i.e., epiglottitis, anaphylactic shock), or vocal cord dysfunction can certainly cause shortness of breath.  Endotracheal intubation may need to be performed before moving forward.  Breathing is assessed by evaluating the function of the lungs.  Steps include looking at how the patient is breathing (fast or slow), measurement of an SpO2 level, and auscultation of both lungs for wheezing, crackles, rhonchi, or distant or absent sounds.  A low oxygen level should be immediately addressed with supplemental oxygen before moving forward.  The patient’s breathing rate and lung sounds can be very helpful in discovering the diagnosis and guiding treatment.  Lastly, circulation should be assessed.  Check the heart rate, blood pressure, peripheral pulses, skin color and temperature, and evaluate for any sites of hemorrhage.  The presence of hypotension or tachycardia should be addressed appropriately based on the presumed cause.  After the primary assessment (“ABCs”) and initial treatment actions, a more detailed history and physical exam should be conducted. 

Pertinent causes of shortness of breath for the emergency practitioner to know are outlined in the chart below. 

 

Select Causes of Shortness of Breath (Dyspnea)

Pulmonary

 

Tension pneumothorax, pneumonia, empyema, pleural effusion, pulmonary edema, asthma, COPD

Cardiovascular

 

Acute coronary syndrome (i.e., STEMI), pulmonary embolism, cardiac tamponade, Decompensated Congestive Heart Failure (acute pulmonary edema)

Endocrine

 

Diabetic ketoacidosis (Kussmaul breathing)

Toxicologic

 

Salicylate overdose, or any ingestion that causes a severe metabolic acidosis

Neurologic

 

Intracranial hemorrhage, Stroke, Spinal cord injury, Guillain-Barre syndrome, Myasthenia Gravis crisis (myasthenic crisis)

Hematologic

 

Severe anemia (i.e., GI bleeding, trauma, miscarriage, post-partum hemorrhage, ruptured ectopic pregnancy)

Musculoskeletal

 

Rib fracture, flail chest

Psychiatric

 

Anxiety, Panic attack

Airway Problem

Foreign body, epiglottitis, anaphylactic shock (laryngeal swelling), expanding neck hematoma

This patient arrives to the Emergency department with shortness of breath, productive cough, and fever for 5 days.  On exam, the patient is febrile, tachycardic, and has a low SpO2 on room air.  The lung exam demonstrates focal rhonchi at the right base.  The chest X-ray demonstrates a consolidation at the right middle lobe that obscures the right heart boarder.  The consolidation is highlighted with a red star in the patient’s X-ray below.

Lung consolidations have multiple causes, including pneumonia, malignancy, heart failure, pulmonary emboli, and septic emboli from endocarditis.  Septic pulmonary emboli (Choice A) can present with cough, fever, and difficulty breathing, but often have multiple foci of consolidations on chest X-ray.  This patient has a single area of consolidation.  This patient also lacks the typical risk factors for septic emboli, like IV drug use, recent dental procedures, structural heart disease, or prosthetic heart valves.  An infected pleural effusion (Choice B), also known as an empyema, is shown as a blunted or hazy right costo-diaphragmatic angle.  This patient’s X-ray shows no fluid in both costo-diaphragmatic recesses to indicate the presence of a pleural effusion.  A pulmonary embolism (Choice D) often presents with clear lungs on auscultation and a normal chest X-ray.  However, if the pulmonary embolism progresses to a pulmonary infarct, a wedge-shaped opacity can be seen on the X-ray.  This patient’s X-ray lacks this finding.  The most likely cause for this patient’s symptoms is a right middle lobe pneumonia (Choice C).  She should receive IV fluids, antipyretics, supplemental oxygen, and IV antibiotics.

References

[cite]

Question Of The Day #89

question of the day

Which of the following is the most appropriate next step in management?

Shortness of breath, also known as dyspnea, is a common reason for patients to visit the Emergency Department.  Dyspnea is often caused by a pulmonary or cardiovascular condition, but it is important to remember that dyspnea can be due to endocrine conditions, toxicologic conditions, neurologic conditions, hematologic conditions, musculoskeletal conditions, and psychiatric conditions. 

The initial approach to all patients with shortness of breath involves the primary survey, or “ABCs” (Airway, Breathing, Circulation).  This first involves checking the patient for a patent airway.  A simple method to assess the airway is to ask the patient to speak and listen for the voice.  A muffled voice, the presence of stridor, hematemesis, or a lethargic patient are clues that a patent airway may not be present.  Problems with the airway, such as an obstructing foreign body, inflammation (i.e., epiglottitis, anaphylactic shock), or vocal cord dysfunction can certainly cause shortness of breath.  Endotracheal intubation may need to be performed before moving forward.  Breathing is assessed by evaluating the function of the lungs.  Steps include looking at how the patient is breathing (fast or slow), measurement of an SpO2 level, and auscultation of both lungs for wheezing, crackles, rhonchi, or distant or absent sounds.  A low oxygen level should be immediately addressed with supplemental oxygen before moving forward.  The patient’s breathing rate and lung sounds can be very helpful in discovering the diagnosis and guiding treatment.  Lastly, circulation should be assessed.  Check the heart rate, blood pressure, peripheral pulses, skin color and temperature, and evaluate for any sites of hemorrhage.  The presence of hypotension or tachycardia should be addressed appropriately based on the presumed cause.  After the primary assessment (“ABCs”) and initial treatment actions, a more detailed history and physical exam should be conducted. 

Pertinent causes of shortness of breath for the emergency practitioner to know are outlined in the chart below. 

 

Select Causes of Shortness of Breath (Dyspnea)

Pulmonary

 

Tension pneumothorax, pneumonia, empyema, pleural effusion, pulmonary edema, asthma, COPD

Cardiovascular

 

Acute coronary syndrome (i.e., STEMI), pulmonary embolism, cardiac tamponade, Decompensated Congestive Heart Failure (acute pulmonary edema)

Endocrine

 

Diabetic ketoacidosis (Kussmaul breathing)

Toxicologic

 

Salicylate overdose, or any ingestion that causes a severe metabolic acidosis

Neurologic

 

Intracranial hemorrhage, Stroke, Spinal cord injury, Guillain-Barre syndrome, Myasthenia Gravis crisis (myasthenic crisis)

Hematologic

 

Severe anemia (i.e., GI bleeding, trauma, miscarriage, post-partum hemorrhage, ruptured ectopic pregnancy)

Musculoskeletal

 

Rib fracture, flail chest

Psychiatric

 

Anxiety, Panic attack

Airway Problem

Foreign body, epiglottitis, anaphylactic shock (laryngeal swelling), expanding neck hematoma

This patient arrives to the Emergency department with acute shortness of breath, an urticarial rash, hypotension, tachycardia, swelling of the lips and tongue, and wheezing on lung exam.  This patient is in anaphylactic shock and requires prompt treatment with epinephrine.  Anaphylaxis is an IgE-mediated life-threatening allergic reaction that by definition affects two or more body systems (i.e., skin/mucosa, pulmonary, cardiovascular, gastrointestinal, etc.).  This patient has involvement of the skin (urticarial rash, mucosal swelling), cardiovascular system (hypotension and tachycardia), and pulmonary system (wheezing).  Symptoms of anaphylaxis may include urticaria, shortness of breath, wheezing, facial or airway swelling, vomiting or diarrhea, and abdominal pain.  Anaphylaxis is a clinical diagnosis and does not require vital signs to be unstable in order to be diagnosed.  Once diagnosed, the most time sensitive and lifesaving treatment is epinephrine.  The recommended initial dose for epinephrine is 0.3-0.5mg intramuscularly in the thigh for adults.  Epinephrine doses can be repeated every 5-15 minutes if there is no improvement after the initial dose. Antihistamines, like Diphenhydramine (Choice D) or famotidine may be helpful as adjunctive treatments, but they are not lifesaving.  Steroids, like Dexamethasone (Choice C), are also routinely given in anaphylaxis with the theory that they can prevent “rebound” allergic reactions.  Again, steroids are not acutely lifesaving and should be given after IM epinephrine.  IV epinephrine can be given in a patient unresponsive to IM epinephrine at a dose of 1-5mcg/min.  A dose of IV Epinephrine 1mg (1000mcg) (Choice A) is the dose of Epinephrine used during cardiac arrest and is too high of a dose to use in anaphylaxis.  The best initial step in management is IM Epinephrine 0.3mg (Choice B).  

References

[cite]

Question Of The Day #88

question of the day
Which of the following is the most appropriate next step in management?

Shortness of breath, also known as dyspnea, is a common reason for patients to visit the Emergency Department.  Dyspnea is often caused by a pulmonary or cardiovascular condition, but it is important to remember that dyspnea can be due to endocrine conditions, toxicologic conditions, neurologic conditions, hematologic conditions, musculoskeletal conditions, and psychiatric conditions. 

The initial approach to all patients with shortness of breath involves the primary survey, or “ABCs” (Airway, Breathing, Circulation).  This first involves checking the patient for a patent airway.  A simple method to assess the airway is to ask the patient to speak and listen for the voice.  A muffled voice, the presence of stridor, hematemesis, or a lethargic patient are clues that a patent airway may not be present.  Problems with the airway, such as an obstructing foreign body, inflammation (i.e., epiglottitis, anaphylactic shock), or vocal cord dysfunction can certainly cause shortness of breath.  Endotracheal intubation may need to be performed before moving forward.  Breathing is assessed by evaluating the function of the lungs.  Steps include looking at how the patient is breathing (fast or slow), measurement of an SpO2 level, and auscultation of both lungs for wheezing, crackles, rhonchi, or distant or absent sounds.  A low oxygen level should be immediately addressed with supplemental oxygen before moving forward.  The patient’s breathing rate and lung sounds can be very helpful in discovering the diagnosis and guiding treatment.  Lastly, circulation should be assessed.  Check the heart rate, blood pressure, peripheral pulses, skin color and temperature, and evaluate for any sites of hemorrhage.  The presence of hypotension or tachycardia should be addressed appropriately based on the presumed cause.  After the primary assessment (“ABCs”) and initial treatment actions, a more detailed history and physical exam should be conducted. 

Pertinent causes of shortness of breath for the emergency practitioner to know are outlined in the chart below. 

 

Select Causes of Shortness of Breath (Dyspnea)

Pulmonary

 

Tension pneumothorax, pneumonia, empyema, pleural effusion, pulmonary edema, asthma, COPD

Cardiovascular

 

Acute coronary syndrome (i.e., STEMI), pulmonary embolism, cardiac tamponade, Decompensated Congestive Heart Failure (acute pulmonary edema)

Endocrine

 

Diabetic ketoacidosis (Kussmaul breathing)

Toxicologic

 

Salicylate overdose, or any ingestion that causes a severe metabolic acidosis

Neurologic

 

Intracranial hemorrhage, Stroke, Spinal cord injury, Guillain-Barre syndrome, Myasthenia Gravis crisis (myasthenic crisis)

Hematologic

 

Severe anemia (i.e., GI bleeding, trauma, miscarriage, post-partum hemorrhage, ruptured ectopic pregnancy)

Musculoskeletal

 

Rib fracture, flail chest

Psychiatric

 

Anxiety, Panic attack

Airway Problem

Foreign body, epiglottitis, anaphylactic shock (laryngeal swelling), expanding neck hematoma

This patient arrives to the Emergency department with shortness of breath and abdominal discomfort for 1 day.  On exam, she is hypotensive, tachycardic, and tachypneic.  Her lungs are clear, the abdomen is tender and distended, and the pregnancy test is positive.  This patient has a ruptured ectopic pregnancy until proven otherwise and requires prompt surgical management.  Once diagnosed by the Emergency clinician, ectopic pregnancy can be managed medically or surgically.  See the chart below for more details.

Treatment options for ectopic pregnancy

 

Medical Management (Methotrexate) Indicated:

Surgical Management

Indicated:

Patient hemodynamically stable

Patient hemodynamically unstable

HCG <5,000

HCG >5,000

Able to comply with Methotrexate treatment and follow up

Unable to comply with Methotrexate treatment and/or follow up

No fetal cardiac activity on ultrasound

Fetal cardiac activity present on ultrasound

   

This patient has an assumed ectopic pregnancy due to the positive pregnancy test and presence of hemodynamic instability.  A transvaginal ultrasound (Choice C) would help definitively diagnose the patient with a ruptured ectopic pregnancy, but this should not delay consultation with the OBGYN team for definitive surgical management.  Methotrexate (Choice A) is a medical treatment for ectopic pregnancy, but Methotrexate is contraindicated in ruptured ectopic due to the need for surgical treatment and intra-abdominal hemorrhage control.  IV antibiotics (Choice B) are often given preoperatively for infection prophylaxis (prevention), but this is not a crucial next step.  This patient is in shock and needs operative management. The best next step is OBGYN consultation for operative management (Choice D).

References

[cite]

Question Of The Day #87

question of the day

 

Test Value

Reference Range

BUN

14

6 – 24 mg/dL

Creatinine

0.87

0.59 – 1.04 mg/dL

Hemoglobin

5.5

12.0 – 15.0 g/dL

WBC count

5.2

4.5 to 11.0 × 109/L

HCG quantitative

0

<5 mIU/mL

Which of the following is the most like cause for this patient’s condition?

Shortness of breath, also known as dyspnea, is a common reason for patients to visit the Emergency Department.  Dyspnea is often caused by a pulmonary or cardiovascular condition, but it is important to remember that dyspnea can be due to endocrine conditions, toxicologic conditions, neurologic conditions, hematologic conditions, musculoskeletal conditions, and psychiatric conditions. 

The initial approach to all patients with shortness of breath involves the primary survey, or “ABCs” (Airway, Breathing, Circulation).  This first involves checking the patient for a patent airway.  A simple method to assess the airway is to ask the patient to speak and listen for the voice.  A muffled voice, the presence of stridor, hematemesis, or a lethargic patient are clues that a patent airway may not be present.  Problems with the airway, such as an obstructing foreign body, inflammation (i.e., epiglottitis, anaphylactic shock), or vocal cord dysfunction can certainly cause shortness of breath.  Endotracheal intubation may need to be performed before moving forward.  Breathing is assessed by evaluating the function of the lungs.  Steps include looking at how the patient is breathing (fast or slow), measurement of an SpO2 level, and auscultation of both lungs for wheezing, crackles, rhonchi, or distant or absent sounds.  A low oxygen level should be immediately addressed with supplemental oxygen before moving forward.  The patient’s breathing rate and lung sounds can be very helpful in discovering the diagnosis and guiding treatment.  Lastly, circulation should be assessed.  Check the heart rate, blood pressure, peripheral pulses, skin color and temperature, and evaluate for any sites of hemorrhage.  The presence of hypotension or tachycardia should be addressed appropriately based on the presumed cause.  After the primary assessment (“ABCs”) and initial treatment actions, a more detailed history and physical exam should be conducted. 

Pertinent causes of shortness of breath for the emergency practitioner to know are outlined in the chart below. 

 

Select Causes of Shortness of Breath (Dyspnea)

Pulmonary

 

Tension pneumothorax, pneumonia, empyema, pleural effusion, pulmonary edema, asthma, COPD

Cardiovascular

 

Acute coronary syndrome (i.e., STEMI), pulmonary embolism, cardiac tamponade, Decompensated Congestive Heart Failure (acute pulmonary edema)

Endocrine

 

Diabetic ketoacidosis (Kussmaul breathing)

Toxicologic

 

Salicylate overdose, or any ingestion that causes a severe metabolic acidosis

Neurologic

 

Intracranial hemorrhage, Stroke, Spinal cord injury, Guillain-Barre syndrome, Myasthenia Gravis crisis (myasthenic crisis)

Hematologic

 

Severe anemia (i.e., GI bleeding, trauma, miscarriage, post-partum hemorrhage, ruptured ectopic pregnancy)

Musculoskeletal

 

Rib fracture, flail chest

Psychiatric

 

Anxiety, Panic attack

Airway Problem

Foreign body, epiglottitis, anaphylactic shock (laryngeal swelling), expanding neck hematoma

This patient arrives to the Emergency department with shortness of breath with deceased exercise tolerance or 5 days.  Her vital signs are normal and lungs are clear, but she appears pale.  The laboratory test provided shows normal kidney function, a negative serum pregnancy test, and a markedly low hemoglobin level.  A ruptured ectopic pregnancy (Choice B) can cause shortness of breath due to anemia and hemorrhagic shock, but this patient has a negative pregnancy test.  Asthma (Choice A) is unlikely given the patient’s normal lung exam without wheezing and no mention of cough.  A pulmonary embolism (Choice D) is possible due to the tachycardia, but the patient lacks other risk factors as stated in the question stem.  A D-Dimer test could help further evaluate if this patient has a pulmonary embolism, but the low hemoglobin likely explains the patient’s symptoms.  The patient’s history of menorrhagia, also known as heavy menses (Choice C), is a common cause of anemia in women of childbearing age.  Even though this patient is not currently menstruating, her heavy menses are the most likely cause for her shortness of breath.  Choice C is the best answer.

References

[cite]

Question Of The Day #86

question of the day
420 - right pneumothorax1
Which of the following is the most appropriate next step in management for this patient’s condition?

Shortness of breath, also known as dyspnea, is a common reason for patients to visit the Emergency Department.  Dyspnea is often caused by a pulmonary or cardiovascular condition, but it is important to remember that dyspnea can be due to endocrine conditions, toxicologic conditions, neurologic conditions, hematologic conditions, musculoskeletal conditions, and psychiatric conditions. 

The initial approach to all patients with shortness of breath involves the primary survey, or “ABCs” (Airway, Breathing, Circulation).  This first involves checking the patient for a patent airway.  A simple method to assess the airway is to ask the patient to speak and listen for the voice.  A muffled voice, the presence of stridor, hematemesis, or a lethargic patient are clues that a patent airway may not be present.  Problems with the airway, such as an obstructing foreign body, inflammation (i.e., epiglottitis, anaphylactic shock), or vocal cord dysfunction can certainly cause shortness of breath.  Endotracheal intubation may need to be performed before moving forward.  Breathing is assessed by evaluating the function of the lungs.  Steps include looking at how the patient is breathing (fast or slow), measurement of an SpO2 level, and auscultation of both lungs for wheezing, crackles, rhonchi, or distant or absent sounds.  A low oxygen level should be immediately addressed with supplemental oxygen before moving forward.  The patient’s breathing rate and lung sounds can be very helpful in discovering the diagnosis and guiding treatment.  Lastly, circulation should be assessed.  Check the heart rate, blood pressure, peripheral pulses, skin color and temperature, and evaluate for any sites of hemorrhage.  The presence of hypotension or tachycardia should be addressed appropriately based on the presumed cause.  After the primary assessment (“ABCs”) and initial treatment actions, a more detailed history and physical exam should be conducted. 

Pertinent causes of shortness of breath for the emergency practitioner to know are outlined in the chart below. 

 

Select Causes of Shortness of Breath (Dyspnea)

Pulmonary

 

Tension pneumothorax, pneumonia, empyema, pleural effusion, pulmonary edema, asthma, COPD

Cardiovascular

 

Acute coronary syndrome (i.e., STEMI), pulmonary embolism, cardiac tamponade, Decompensated Congestive Heart Failure (acute pulmonary edema)

Endocrine

 

Diabetic ketoacidosis (Kussmaul breathing)

Toxicologic

 

Salicylate overdose, or any ingestion that causes a severe metabolic acidosis

Neurologic

 

Intracranial hemorrhage, Stroke, Spinal cord injury, Guillain-Barre syndrome, Myasthenia Gravis crisis (myasthenic crisis)

Hematologic

 

Severe anemia (i.e., GI bleeding, trauma, miscarriage, post-partum hemorrhage, ruptured ectopic pregnancy)

Musculoskeletal

 

Rib fracture, flail chest

Psychiatric

 

Anxiety, Panic attack

Airway Problem

Foreign body, epiglottitis, anaphylactic shock (laryngeal swelling), expanding neck hematoma

This patient arrives to the Emergency department with acute onset shortness of breath with pleuritic right sided chest pain.  On exam, there is mild tachypnea and a borderline low SpO2 of 95% on room air.  The chest X-ray demonstrates a small right sided pneumothorax (see location of red stars below).

Needle decompression to the right chest (Choice C) would be the right choice if the patient had a right sided tension pneumothorax.  Signs of a tension pneumothorax are hypotension, tachycardia, tracheal deviation, and mediastinal shift on Chest X-ray.  Tension pneumothorax should be diagnosed clinically without a chest X-ray and promptly treated with needle decompression with a 14-16 gauge needle at the 2nd intercostal space in the mid clavicular line.  Needle decompression can also be performed at the 4th or 5th intercostal space in the anterior axillary line. Needle decompression is always followed by placement of a formal chest tube.  This patient does not have the hemodynamic instability or chest X-ray findings of a classic tension pneumothorax. IV Azithromycin (Choice D) would be appropriate for a COPD exacerbation or for community-acquired pneumonia.  This patient does have a cough, but lacks fever, sputum production, and also has a pneumothorax on X-ray that can explain his symptoms.  An IV Heparin bolus and infusion (Choice A) would be the ideal treatment for a pulmonary embolism or acute coronary syndrome.  Again, the Chest X-ray provided shows support for an alternative cause for the patient’s symptoms.  The best next step is supplemental oxygen (Choice B).  100% supplemental oxygen helps decrease the time to lung expansion in patients with pneumothoraces.   A nonrebreather mask at 15L/min is the ideal method to providing this level of oxygen.

This patient has a small pneumothorax (<3cm between lung margin and chest wall).  Small primary pneumothoraces have two treatment options.  The first option is to administer 100% oxygen and place a pigtail catheter for rapid lung re-expansion.  The second option is to only administer 100% oxygen administration for a period of 4-6 hours followed by a repeat chest X-ray to evaluate for improvement of the pneumothorax.   If the pneumothorax is improving and symptoms are improving (less shortness of breath and chest pain), the patient can be discharged home with close outpatient follow up and no chest tube placement.  Deciding which treatment option is best should depend on the patient’s ability to follow up with a doctor, patient reliability, and resource availability.  This patient does have a small pneumothorax by measurement, but he likely has a secondary pneumothorax from his COPD.  Secondary pneumothoraces have a higher rate of recurrence and almost always require chest tube placement.  Regardless, the best initial step in treatment is supplemental oxygen (Choice B).

References

[cite]